Med Surg 101!!!!

अब Quizwiz के साथ अपने होमवर्क और परीक्षाओं को एस करें!

A client diagnosed with meningitis says, "Im just so thirsty. I keep drinking and drinking but I just cant seem to get enough. Ive been urinating a lot too." The nurse checks the clients urine specific gravity and finds it to be very dilute. The nurse suspects that the client may be developing diabetes insipidus. Which assessment finding would support the nurses suspicion? select all that apply. A) weight gain B) decreased heart rate C) hypotension D) poor skin turgor E) dry mucous membranes

C) hypotension D) poor skin turgor E) dry mucous membranes

The nurses musculoskeletal assessment of a patient reveals involuntary twitching of muscle groups. How would the nurse document this observation in the patients chart? A) Tetany B) Atony C) Clonus D) Fasciculations

D

A 6-year-old child is brought to the pediatric clinic for the assessment of redness and discharge from the eye and is diagnosed with viral conjunctivitis. What is the most important information to discuss with the parents and child? A) Handwashing can prevent the spread of the disease to others. B) The importance of compliance with antibiotic therapy C) Signs and symptoms of complications, such as meningitis and septicemia D) The likely need for surgery to prevent scarring of the conjunctiva

A

A child is growing at a rate appropriate for his age. What cells are responsible for the secretion of bone matrix that eventually results in bone growth? A) Osteoblasts B) Osteocytes C) Osteoclasts D) Lamellae

A

A medical patients CA 19-9 levels have become available and they are significantly elevated. How should the nurse best interpret this diagnostic finding? A) The patient may have cancer, but other GI disease must be ruled out. B) The patient most likely has early-stage colorectal cancer. C) The patient has a genetic predisposition to gastric cancer. D) The patient has cancer, but the site is unknown.

A

A nurse is promoting increased protein intake to enhance a patients wound healing. The nurse knows that enzymes are essential in the digestion of nutrients such as protein. What is the enzyme that initiates the digestion of protein? A) Pepsin B) Intrinsic factor C) Lipase D) Amylase

A

A patient has come to the outpatient radiology department for diagnostic testing. Which of the following diagnostic procedures will allow the care team to evaluate and remove polyps? A) Colonoscopy B) Barium enema C) ERCP D) Upper gastrointestinal fibroscopy

A

A patient has had a sudden loss of vision after head trauma. How should the nurse best describe the placement of items on the dinner tray? A) Explain the location of items using clock cues. B) Explain that each of the items on the tray is clearly separated. C) Describe the location of items from the bottom of the plate to the top. D) Ask the patient to describe the location of items before confirming their location.

A

A patient has just been diagnosed with psoriasis and frequently has lesions around his right eye. What should the nurse teach the patient about topical corticosteroid use on these lesions? A) Cataract development is possible. B) The ointment is likely to cause weeping. C) Corticosteroid use is contraindicated on these lesions. D) The patient may develop glaucoma.

A

A patient is postoperative day 6 following tympanoplasty and mastoidectomy. The patient has phoned the surgical unit and states that she is experiencing occasional sharp, shooting pains in her affected ear. How should the nurse best interpret this patient's complaint? A) These pains are an expected finding during the first few weeks of recovery. B) The patient's complaints are suggestive of a postoperative infection. C) The patient may have experienced a spontaneous rupture of the tympanic membrane. D) The patient's surgery may have been unsuccessful.

A

A patient presents at the free clinic with a black, wart-like lesion on his face, stating, Ive done some research, and Im pretty sure I have malignant melanoma. Subsequent diagnostic testing results in a diagnosis of seborrheic keratosis. The nurse should recognize what significance of this diagnosis? A) The patient requires no treatment unless he finds the lesion to be cosmetically unacceptable. B) The patients lesion will be closely observed for 6 months before a plan of treatment is chosen. C) The patient has one of the few dermatologic malignancies that respond to chemotherapy. D) The patient will likely require wide excision.

A

A patient presents at the walk-in clinic complaining of recurrent sharp stomach pain that is relieved by eating. The nurse suspects that the patient may have an ulcer. How would the nurse explain the formation and role of acid in the stomach to the patient? A) Hydrochloric acid is secreted by glands in the stomach in response to the actual or anticipated presence of food. B) As digestion occurs in the stomach, the stomach combines free hydrogen ions from the food to form acid. C) The body requires an acidic environment in order to synthesize pancreatic digestive enzymes; the stomach provides this environment. D) The acidic environment in the stomach exists to buffer the highly alkaline environment in the esophagus.

A

A patient will be undergoing abdominal computed tomography (CT) with contrast. The nurse has administered IV sodium bicarbonate and oral acetylcysteine (Mucomyst) before the study as ordered. What would indicate that these medications have had the desired therapeutic effect? A) The patients BUN and creatinine levels are within reference range following the CT. B) The CT yields high-quality images. C) The patients electrolytes are stable in the 48 hours following the CT. D) The patients intake and output are in balance on the day after the CT.

A

An advanced practice nurse is assessing the size and density of a patients abdominal organs. If the results of palpation are unclear to the nurse, what assessment technique should be implemented? A) Percussion B) Auscultation C) Inspection D) Rectal examination

A

The nurse is performing an assessment of a patients musculoskeletal system and is appraising the patients bone integrity. What action should the nurse perform during this phase of assessment? A) Compare parts of the body symmetrically. B) Assess extremities when in motion rather than at rest. C) Percuss as many joints as are accessible. D) Administer analgesia 30 to 60 minutes before assessment.

A

The nurse is preparing to perform a patients abdominal assessment. What examination sequence should the nurse follow? A) Inspection, auscultation, percussion, and palpation B) Inspection, palpation, auscultation, and percussion C) Inspection, percussion, palpation, and auscultation D) Inspection, palpation, percussion, and auscultation

A

The nurse is providing discharge education for a patient with a new diagnosis of Ménière's disease. What food should the patient be instructed to limit or avoid? A) Sweet pickles B) Frozen yogurt C) Shellfish D) Red meat

A

patient has been experiencing an unexplained decline in knee function and has consequently been scheduled for arthrography. The nurse should teach the patient about what process? A) Injection of a contrast agent into the knee joint prior to ROM exercises B) Aspiration of synovial fluid for serologic testing C) Injection of corticosteroids into the patients knee joint to facilitate ROM D) Replacement of the patients synovial fluid with a synthetic substitute

A

A patient with thyroid cancer has undergone surgery and a significant amount of parathyroid tissue has been removed. The nurse caring for the patient should prioritize what question when addressing potential complications? A) "Do you feel any muscle twitches or spasms?" B) "Do you feel flushed or sweaty?" C) "Are you experiencing any dizziness or lightheadedness?" D) "Are you having any pain that seems to be radiating from your bones?"

A) "Do you feel any muscle twitches or spasms?"

The outer layer of the epidermis provides the most effective barrier to penetration of the skin by environmental factors. Which of the following is an example of penetration by an environmental factor? A) An insect bite B) Dehydration C) Sunburn D) Excessive perspiration

A) An insect bite The stratum corneum, the outer layer of the epidermis, provides the most effective barrier to both epidermal water loss and penetration of environmental factors, such as chemicals, microbes, insect bites, and other trauma. Dehydration, sunburn, and excessive perspiration are not examples of penetration of an environmental factor.

Following a thyroidectomy, a client exhibits signs of tetany. The nurse anticipates administering which medication? A) IV calcium gluconate B) methimazole C) propylthiouracil D) potassium iodide

A) IV calcium gluconate

While waiting to see the physician, a patient shows the nurse skin areas that are flat, nonpalpable, and have had a change of color. The nurse recognizes that the patient is demonstrating what? A) Macules B) Papules C) Vesicles D) Pustules

A) Macules A macule is a flat, nonpalpable skin color change, while a papule is an elevated, solid, palpable mass. A vesicle is a circumscribed, elevated, palpable mass containing serous fluid, while a pustule is a pus-filled vesicle.

A patient is undergoing testing for suspected adrenocortical insufficiency. The care team should ensure that the patient has been assessed for the most common cause of adrenocortical insufficiency. What is the most common cause of this health problem? A) Therapeutic use of corticosteroids B) Pheochromocytoma C) Inadequate secretion of ACTH D) Adrenal tumor

A) Therapeutic use of corticosteroids

The nurse is performing a shift assessment of a patient with aldosteronism. What assessments should the nurse include? Select all that apply. A) Urine output B) Signs or symptoms of venous thromoembolism C) Peripheral pulses D) Blood pressure E) Skin integrity

A) Urine output D) Blood pressure

The nurse educator is reviewing the blood supply of the GI tract with a group of medical nurses. The nurse is explaining the fact that the veins that return blood from the digestive organs and the spleen form the portal venous system. What large veins will the nurse list when describing this system? Select all that apply. A) Splenic vein B) Inferior mesenteric vein C) Gastric vein D) Inferior vena cava E) Saphenous vein

A,B,E

A patient who has experienced an ischemic stroke has been admitted to the medical unit. The patient's family in adamant that she remain on bed rest to hasten her recovery and to conserve energy. What principle of care should inform the nurse's response to the family? A) The patient should mobilize as soon as she is physically able. B) To prevent contractures and muscle atrophy, bed rest should not exceed 4 weeks. C) The patient should remain on bed rest until she expresses a desire to mobilize. D) Lack of mobility will greatly increase the patient's risk of stroke recurrence.

Ans: A Feedback: As soon as possible, the patient is assisted out of bed and an active rehabilitation program is started. Delaying mobility causes complications, but not necessarily stroke recurrence. Mobility should not be withheld until the patient initiates.

The nurse is caring for a patient whose spinal cord injury has caused recent muscle spasticity. What medication should the nurse expect to be ordered to control this? A) Baclofen (Lioresal) B) Dexamethasone (Decadron) C) Mannitol (Osmitrol) D) Phenobarbital (Luminal)

Ans: A Feedback: Baclofen is classified as an antispasmodic agent in the treatment of muscles spasms related to spinal cord injury. Decadron is an anti-inflammatory medication used to decrease inflammation in both SCI and head injury. Mannitol is used to decrease cerebral edema in patients with head injury. Phenobarbital is an anticonvulsant that is used in the treatment of seizure activity.

A preceptor is discussing stroke with a new nurse on the unit. The preceptor would tell the new nurse which cardiac dysrhythmia is associated with cardiogenic embolic strokes? A) Ventricular tachycardia B) Atrial fibrillation C) Supraventricular tachycardia D) Bundle branch block

Ans: B Feedback: Cardiogenic embolic strokes are associated with cardiac dysrhythmias, usually atrial fibrillation. The other listed dysrhythmias are less commonly associated with this type of stroke.

A patient has recently begun mobilizing during the recovery from an ischemic stroke. To protect the patient's safety during mobilization, the nurse should perform what action? A) Support the patient's full body weight with a waist belt during ambulation. B) Have a colleague follow the patient closely with a wheelchair. C) Avoid mobilizing the patient in the early morning or late evening. D) Ensure that the patient's family members do not participate in mobilization.

Ans: B Feedback: During mobilization, a chair or wheelchair should be readily available in case the patient suddenly becomes fatigued or feels dizzy. The family should be encouraged to participate, as appropriate, and the nurse should not have to support the patient's full body weight. Morning and evening activity are not necessarily problematic.

A nurse is caring for a patient diagnosed with a hemorrhagic stroke. When creating this patient's plan of care, what goal should be prioritized? A) Prevent complications of immobility. B) Maintain and improve cerebral tissue perfusion. C) Relieve anxiety and pain. D) Relieve sensory deprivation.

Ans: B Feedback: Each of the listed goals is appropriate in the care of a patient recovering from a stroke. However, promoting cerebral perfusion is a priority physiologic need, on which the patient's survival depends.

When caring for a patient who has had a stroke, a priority is reduction of ICP. What patient position is most consistent with this goal? A) Head turned slightly to the right side B) Elevation of the head of the bed C) Position changes every 15 minutes while awake D) Extension of the neck

Ans: B Feedback: Elevation of the head of the bed promotes venous drainage and lowers ICP; the nurse should avoid flexing or extending the neck or turning the head side to side. The head should be in a neutral midline position. Excessively frequent position changes are unnecessary.

A patient with a new diagnosis of ischemic stroke is deemed to be a candidate for treatment with tissue plasminogen activator (t-PA) and has been admitted to the ICU. In addition to closely monitoring the patient's cardiac and neurologic status, the nurse monitors the patient for signs of what complication? A) Acute pain B) Septicemia C) Bleeding D) Seizures

Ans: C Feedback: Bleeding is the most common side effect of t-PA administration, and the patient is closely monitored for any bleeding. Septicemia, pain, and seizures are much less likely to result from thrombolytic therapy.

28. A patient with a brain tumor has begun to exhibit signs of cachexia. What subsequent assessment should the nurse prioritize? A) Assessment of peripheral nervous function B) Assessment of cranial nerve function C) Assessment of nutritional status D) Assessment of respiratory status

Ans: C- assessment of nutritional status Feedback: Cachexia is a wasting syndrome of weight loss, muscle atrophy, fatigue, weakness, and significant loss of appetite. Consequently, nutritional assessment is paramount.

The public health nurse is planning a health promotion campaign that reflects current epidemiologic trends. The nurse should know that hemorrhagic stroke currently accounts for what percentage of total strokes in the United States? A) 43% B) 33% C) 23% D) 13%

Ans: D Feedback: Strokes can be divided into two major categories: ischemic (87%), in which vascular occlusion and significant hypoperfusion occur, and hemorrhagic (13%), in which there is extravasation of blood into the brain or subarachnoid space.

A diabetic nurse is working for the summer at a camp for adolescents with diabetes. When providing information on the prevention and management of hypoglycemia, what action should the nurse promote? A) Always carry a form of fast-acting sugar. B) Perform exercise prior to eating whenever possible. C) Eat a meal or snack every 8 hours. D) Check blood sugar at least every 24 hours.

Ans: A Feedback: The following teaching points should be included in information provided to the patient on how to prevent hypoglycemia: Always carry a form of fast-acting sugar, increase food prior to exercise, eat a meal or snack every 4 to 5 hours, and check blood sugar regularly.

A nurse is caring for a patient who has suffered an unstable thoracolumbar fracture. Which of the following is the priority during nursing care? A) Preventing infection B) Maintaining spinal alignment C) Maximizing function D) Preventing increased intracranial pressure

Ans: B Feedback: Patients with an unstable fracture must have their spine in alignment at all times in order to prevent neurologic damage. This is a greater threat, and higher priority, than promoting function and preventing infection, even though these are both valid considerations. Increased ICP is not a high risk.

A bone biopsy has just been completed on a patient with suspected bone metastases. What assessment should the nurse prioritize in the immediate recovery period? A) Assessment for dehiscence at the biopsy site B) Assessment for pain C) Assessment for hematoma formation D) Assessment for infection

B

While reviewing the health history of an older adult experiencing hearing loss the nurse notes the patient has had no trauma or loss of balance. What aspect of this patient's health history is most likely to be linked to the patient's hearing deficit? A) Recent completion of radiation therapy for treatment of thyroid cancer B) Routine use of quinine for management of leg cramps C) Allergy to hair coloring and hair spray D) Previous perforation of the eardrum

B

A nurse is explaining the importance of sunlight on the skin to a woman with decreased mobility who rarely leaves her house. The nurse would emphasize that ultraviolet light helps to synthesize what vitamin? A) E B) D C) A D) C

B) D Skin exposed to ultraviolet light can convert substances necessary for synthesizing vitamin D (cholecalciferol). Vitamin D is essential for preventing rickets, a condition that causes bone deformities and results from a deficiency of vitamin D, calcium, and phosphorus.

A young student is brought to the school nurse after falling off a swing. The nurse is documenting that the child has bruising on the lateral aspect of the right arm. What term will the nurse use to describe bruising on the skin in documentation? A) Telangiectasias B) Ecchymoses C) Purpura D) Urticaria

B) Ecchymoses Telangiectasias consists of red marks on the skin caused by stretching of superficial blood vessels. Ecchymoses are bruises, and purpura consists of pinpoint hemorrhages into the skin. Urticariais wheals or hives.

The nurse is planning the care of a patient with hyperthyroidism. What should the nurse specify in the patient's meal plan? A) A clear thin liquid diet, high in nutrients B) Small frequent meals, high in protein and calories C) Three large, bland meals a day D) A diet high in fiber and plant-sourced fat

B) Small frequent meals, high in protein and calories

A patient injured in a motor vehicle accident has sustained a fracture to the diaphysis of the right femur. Of what is the diaphysis of the femur mainly constructed? A) Epiphyses B) Cartilage C) Cortical bone D) Cancellous bone

C

The nurses comprehensive assessment of an older adult involves the assessment of the patients gait. How should the nurse best perform this assessment? A) Instruct the patient to walk heel-to-toe for 15 to 20 steps. B) Instruct the patient to walk in a straight line while not looking at the floor. C) Instruct the patient to walk away from the nurse for a short distance and then toward the nurse. D) Instruct the patient to balance on one foot for as long as possible and then walk in a circle around the room.

C

The results of a nurses musculoskeletal examination show an increase in the lumbar curvature of the spine. The nurse should recognize the presence of what health problem? A) Osteoporosis B) Kyphosis C) Lordosis D) Scoliosis

C

The nurse is assessing a patient diagnosed with Graves' disease. What physical characteristics of Graves' disease would the nurse expect to find? A) hair loss B) moon face C) bulging eyes D) fatigue

C) bulging eyes

A patient with liver disease has developed jaundice; the nurse is collaborating with the patient to develop a nutritional plan. The nurse should prioritize which of the following in the patient's plan? A. Increased potassium intake B. Fluid restriction to 2L per day C. Reduction in sodium intake D. High-protein, low-fat diet

C. Reduction in sodium intake Rationale: Patient with ascites require a sharp reduction in sodium intake. Potassium intake should not be correspondingly increased. There is no need for fluid restriction or increased protein intake.

A patient with liver cancer is being discharged home with a biliary drainage system in place. The nurse should teach the patient's family how to safely perform which of the following actions? A. Aspirating bile from the catheter using a syringe B. Removing the catheter when output is 15 mL in 24 hours C. Instilling antibiotics into the catheter D. Assessing the patency of the drainage catheter

D. Assessing the patency of the drainage catheter Rationale: Families should be taught to provide basic catheter care, including assessment of patency. Antibiotics are not instilled into the catheter and aspiration using a syringe is contraindicated. The family would not independently remove the catheter; this would be done by a member of the care team when deemed necessary.

A client is diagnosed with a brain tumor of the parietal lobe. Based on the tumors location, which assessment finding would the nurse most likely note? select all that apply. A- difficulty with reading B- problems with mathematical calculations C- impaired reasoning D- memory changes E- changing moods

a- difficulty with reading B- problems with mathematical calculations

A client develops a perforated eardrum. when teaching the client about this condition, the nurse would identify which condition as a most likely cause? a- infection b- otosclerosis c- Meniere's disease d- cholesteatoma

a- infection

A client is diagnosed with an acoustic neuroma. when assessing this client, which manifestation would the nurse expect to find? select all that apply. A- tinnitus B- vertigo C- staggering gait D- seizures E- headache

a- tinnitus b- vertigo c- staggering gait

A client with hearing loss is scheduled to undergo aural rehabilitation. when describing this therapy, the nurse would include which information as the primary purpose? a- increase hearing ability b- maximize ability to communicate c- facilitate use of a hearing aid d- limit extraneous noise

b- maximize ability to communicate

A client diagnosed with benign paroxysmal positional vertigo is experiencing an acute attack. The client is prescribed a vestibular suppressant. Which medication would the nurse anticipate being used? a- scopolamine b- meclizine c- dimenhydrinate d- promethazine

b- meclizine

when administering a patients eye drops, the nurse recognizes the need to prevent absorption by the nasolacrimal duct. How can the nurse best achieve this goal? A- Ensure that the patient is well hydrated at all times B- Encourage self-administration of eye drops C- occlude the puncta after applying the medication D- position the patient supine before administering eye drops

c

A nurse is reading a journal article about brain tumors and the various types that can occur. The nurse demonstrates understanding of the article by identifying which type as being classified as an intracerebral tumor? select all that apply. A- meningioma B- schwannoma C- glioblastoma D- astrocytoma E- medulloblastoma

c- glioblastoma D- astrocytoma E- medulloblastoma

When writing a plan of care for a patient with psoriasis, the nurse would know that an appropriate nursing diagnosis for this patient would be what? A) Impaired Skin Integrity Related to Scaly Lesions B) Acute Pain Related to Blistering and Erosions of the Oral Cavity C) Impaired Tissue Integrity Related to Epidermal Shedding D) Anxiety Related to Risk for Melanoma

A

A nurse is conducting an assessment of a client who is suspected of having a brain tumor. Assessment reveals reports of a headache, for which the nurse gathers additional information. The nurse determines that these reports support the suspicion of a brain tumor when the client reports that the headache occurs: A- early in the morning B- around lunchtime C- in the middle of the afternoon D- at bedtime

A- early in the morning

A patient is admitted to the medical unit with an exacerbation of multiple sclerosis. When assessing this patient, the nurse has the patient stick out her tongue and move it back and forth. What is the nurse assessing? A- function of the hypoglossal nerve B- function of the vagus nerve C- function of the spinal nerve D- function of the trochlear nerve

A- function of the hypoglossal nerve

A gerontologic nurse educator is providing practice guidelines to unlicensed care providers. Because reaction to painful stimuli is sometimes blunted in older adults, what must be used with caution? A- hot or cold packs B- analgesics C- anti inflammatory medications D- whirlpool baths

A- hold or cold packs

A client is exhibiting late signs of increased intracranial pressure. Which finding would the nurse most likely assess? select all that apply. A- hypertension B- bradycardia C- respiratory depression D- headache E- papilledema

A- hypertension B- bradycardia C- respiratory depression

A patient who is being treated in the hospital for a spinal cord injury is advocating for the removal of his urinary catheter, stating that he wants to try to resume normal elimination. What principle should guide the care team's decision regarding this intervention? A) Urinary retention can have serious consequences in patients with SCIs. B) Urinary function is permanently lost following an SCI. C) Urinary catheters should not remain in place for more than 7 days. D) Overuse of urinary catheters can exacerbate nerve damage.

Ans: A Feedback: Bladder distention, a major cause of autonomic dysreflexia, can also cause trauma. For this reason, removal of a urinary catheter must be considered with caution. Extended use of urinary catheterization is often necessary following SCI. The effect of a spinal cord lesion on urinary function depends on the level of the injury. Catheter use does not cause nerve damage, although it is a major risk factor for UTIs.

A female patient has presented to the emergency department with right upper quadrant pain; the physician has ordered abdominal ultrasound to rule out cholecystitis (gallbladder infection). The patient expresses concern to the nurse about the safety of this diagnostic procedure. How should the nurse best respond? A) Abdominal ultrasound is very safe, but it cant be performed if youre pregnant. B) Abdominal ultrasound poses no known safety risks of any kind. C) Current guidelines state that a person can have up to 3 ultrasounds per year. D) Current guidelines state that a person can have up to 6 ultrasounds per year.

B

A nurse is caring for a newly admitted patient with a suspected GI bleed. The nurse assesses the patients stool after a bowel movement and notes it to be a tarry-black color. This finding is suggestive of bleeding from what location? A) Sigmoid colon B) Upper GI tract C) Large intestine D) Anus or rectum

B

A nurse is caring for an older adult who has been diagnosed with geriatric failure to thrive. This patients prolonged immobility creates a risk for what complication? A) Muscle clonus B) Muscle atrophy C) Rheumatoid arthritis D) Muscle fasciculations

B

The nurse is providing health education to a patient newly diagnosed with glaucoma. The nurse teaches the patient that this disease has a familial tendency. The nurse should encourage the patient's immediate family members to undergo clinical examinations how often? A) At least monthly B) At least once every 2 years C) At least once every 5 years D) At least once every 10 years

B

The nurse on the medical-surgical unit is reviewing discharge instructions with a patient who has a history of glaucoma. The nurse should anticipate the use of what medications? A) Potassium-sparing diuretics B) Cholinergics C) Antibiotics D) Loop diuretics

B

The nurse's assessment of a patient with significant visual losses reveals that the patient cannot count fingers. How should the nurse proceed with assessment of the patient's visual acuity? A) Assess the patient's vision using a Snellen chart. B) Determine whether the patient is able to see the nurse's hand motion. C) Perform a detailed examination of the patient's external eye structures. D) Palpate the patient's periocular regions.

B

When assessing a patients peripheral nerve function, the nurse uses an instrument to prick the fat pad at the top of the patients small finger. This action will assess which of the following nerves? A) Radial B) Ulnar C) Median D) Tibial

B

The nurse is preparing to insert a patient's ordered NG tube. What factor should the nurse recognize as a risk for incorrect placement? A) The patient is obese and has a "short neck." B) The patient is agitated. C) The patient has a history of gastroesophageal reflux disease (GERD). D) The patient is being treated for pneumonia.

B Feedback: Inappropriate placement may occur in patients with decreased levels of consciousness, confused mental states, poor or absent cough and gag reflexes, or agitation during insertion. A "short neck," GERD, and pneumonia are not linked to incorrect placement.

A nurses assessment of a teenage girl reveals that her shoulders are not level and that she has one prominent scapula that is accentuated by bending forward. The nurse should expect to read about what health problem in the patients electronic health record? A) Lordosis B) Kyphosis C) Scoliosis D) Muscular dystrophy

C

A patient has had a cast placed for the treatment of a humeral fracture. The nurses most recent assessment shows signs and symptoms of compartment syndrome. What is the nurses most appropriate action? A) Arrange for a STAT assessment of the patients serum calcium levels. B) Perform active range of motion exercises. C) Assess the patients joint function symmetrically. D) Contact the primary care provider immediately.

D

The registered nurse taking shift report learns that an assigned patient is blind. How should the nurse best communicate with this patient? A) Provide instructions in simple, clear terms. B) Introduce herself in a firm, loud voice at the doorway of the room. C) Lightly touch the patient's arm and then introduce herself. D) State her name and role immediately after entering the patient's room.

D

Which of the following nurse's actions carries the greatest potential to prevent hearing loss due to ototoxicity? A) Ensure that patients understand the differences between sensory hearing loss and conductive hearing loss. B) Educate patients about expected age-related changes in hearing perception. C) Educate patients about the risks associated with prolonged exposure to environmental noise. D) Be aware of patients' medication regimens and collaborate with other professionals accordingly.

D

While assessing a patient, the patient tells the nurse that she is experiencing rhythmic muscle contractions when the nurse performs passive extension of her wrist. What is this pattern of muscle contraction referred to as? A) Fasciculations B) Contractures C) Effusion D) Clonus

D

A patient with a recent diagnosis of hypothyroidism is being treated for an unrelated injury. When administering medications to the patient, the nurse should know that the patient's diminished thyroid function may have what effect? A) anaphylaxis B) nausea and vomiting C) increased risk of drug interactions D) prolonged duration of effect

D) prolonged duration of effect

The nurse is admitting a patient to the unit who is diagnosed with a lower motor neuron lesion. What entry in the patient's electronic record is most consistent with this diagnosis? A- "Patient exhibits increased muscle tone" B- "Patient demonstrates normal muscle structure with no evidence of atrophy" C- "Patient demonstrates hyperactive deep tendon reflexes" D- "patient demonstrates an absence of deep tendon reflexes"

D- "Patient demonstrates an absence of deep tendon reflexes"

An elderly patient is being discharged home. The patient lives alone and has atrophy of his olfactory organs. The nurse tells the patient's family that it is essential that the patient have what installed in the home? A- grab bars B- nonslip mats C- baseboard heaters D- a smoke detector

D- a smoke detector

When administering a patient's eye drops, the nurse recognizes the need to prevent absorption by the nasolacrimal duct. How can the nurse best achieve this goal? A) Ensure that the patient is well hydrated at all times. B) Encourage self-administration of eye drops. C) Occlude the puncta after applying the medication. D) Position the patient supine before administering eye drops.

C

A nurse is assessing a child who has a diagnosis of muscular dystrophy. Assessment reveals that the childs muscles have greater-than-normal tone. The nurse should document the presence of which of the following? A) Tonus B) Flaccidity C) Atony D) Spasticity

D

A nurse is caring for a patient who has an MRI scheduled. What is the priority safety action prior to this diagnostic procedure? A) Assessing the patient for signs and symptoms of active infection B) Ensuring that the patient can remain immobile for up to 3 hours C) Assessing the patient for a history of nut allergies D) Ensuring that there are no metal objects on or in the patient

D

A nurse is planning the care of a patient with herpes zoster. What medication, if administered within the first 24 hours of the initial eruption, can arrest herpes zoster? A) Prednisone (Deltasone) B) Azanthioprine (Imuran) C) Triamcinolone (Kenalog) D) Acyclovir (Zovirax)

D

A patient comes to the clinic complaining of a red rash of small, fluid-filled blisters and is suspected of having herpes zoster. What presentation is most consistent with herpes zoster? A) Grouped vesicles occurring on lips and oral mucous membranes B) Grouped vesicles occurring on the genitalia C) Rough, fresh, or gray skin protrusions D) Grouped vesicles in linear patches along a dermatome

D

A hearing-impaired patient is scheduled to have an MRI. What would be important for the nurse to remember when caring for this patient? A) Patient is likely unable to hear the nurse during test. B) A person adept in sign language must be present during test. C) Lip reading will be the method of communication that is necessary. D) The nurse should interact with the patient like any other patient.

A

A nurse is caring for a patient who has been diagnosed with psoriasis. The nurse is creating an education plan for the patient. What information should be included in this plan? A) Use caution when taking nonprescription medications. B) Avoid public places until symptoms subside. C) Wash skin frequently to prevent infection. D) Liberally apply corticosteroids as needed.

A

A nurse is caring for a patient who has just had an arthroscopy as an outpatient and is getting ready to go home. The nurse should teach the patient to monitor closely for what postprocedure complication? A) Fever B) Crepitus C) Fasciculations D) Synovial fluid leakage

A

A nurse is caring for a patient who is scheduled for a colonoscopy and whose bowel preparation will include polyethylene glycol electrolyte lavage prior to the procedure. The presence of what health problem would contraindicate the use of this form of bowel preparation? A) Inflammatory bowel disease B) Intestinal polyps C) Diverticulitis D) Colon cancer

A

A nurse is caring for a patient whose cancer metastasis has resulted in bone pain. Which of the following are typical characteristics of bone pain? A) A dull, deep ache that is boring in nature B) Soreness or aching that may include cramping C) Sharp, piercing pain that is relieved by immobilization D) Spastic or sharp pain that radiates

A

A nurse is caring for an 83-year-old patient who is being assessed for recurrent and intractable nausea. What age-related change to the GI system may be a contributor to the patients health complaint? A) Stomach emptying takes place more slowly. B) The villi and epithelium of the small intestine become thinner. C) The esophageal sphincter becomes incompetent. D) Saliva production decreases.

A

A nurse is providing care for a patient whose pattern of laboratory testing reveals longstanding hypocalcemia. What other laboratory result is most consistent with this finding? A) An elevated parathyroid hormone level B) An increased calcitonin level C) An elevated potassium level D) A decreased vitamin D level

A

A nurse is taking a health history on a patient with musculoskeletal dysfunction. What is the primary focus of this phase of the nurses assessment? A) Evaluating the effects of the musculoskeletal disorder on the patients function B) Evaluating the patients adherence to the existing treatment regimen C) Evaluating the presence of genetic risk factors for further musculoskeletal disorders D) Evaluating the patients active and passive range of motion

A

A nurse is teaching a patient with glaucoma how to administer eye drops to achieve maximum absorption. The nurse should teach the patient to perform what action? A) Instill the medication in the conjunctival sac. B) Maintain a supine position for 10 minutes after administration. C) Keep the eyes closed for 1 to 2 minutes after administration. D) Apply the medication evenly to the sclera

A

A nurse on the orthopedic unit is assessing a patients peroneal nerve. The nurse will perform this assessment by doing which of the following actions? A) Pricking the skin between the great and second toe B) Stroking the skin on the sole of the patients foot C) Pinching the skin between the thumb and index finger D) Stroking the distal fat pad of the small finger

A

A patient comes to the ophthalmology clinic for an eye examination. The patient tells the nurse that he often sees floaters in his vision. How should the nurse best interpret this subjective assessment finding? A) This is a normal aging process of the eye. B) Glasses will minimize this phenomenon. C) The patient may be exhibiting signs of glaucoma. D) This may be a result of weakened ciliary muscles.

A

A patient diagnosed with arthritis has been taking aspirin and now reports experiencing tinnitus and hearing loss. What should the nurse teach this patient? A) The hearing loss will likely resolve with time after the drug is discontinued. B) The patient's hearing loss and tinnitus are irreversible at this point. C) The patient's tinnitus is likely multifactorial, and not directly related to aspirin use. D) The patient's tinnitus will abate as tolerance to aspirin develops.

A

A patient got a sliver of glass in his eye when a glass container at work fell and shattered. The glass had to be surgically removed and the patient is about to be discharged home. The patient asks the nurse for a topical anesthetic for the pain in his eye. What should the nurse respond? A) "Overuse of these drops could soften your cornea and damage your eye." B) "You could lose the peripheral vision in your eye if you used these drops too much." C) "I'm sorry, this medication is considered a controlled substance and patients cannot take it home." D) "I know these drops will make your eye feel better, but I can't let you take them home."

A

A patient has been brought to the emergency department with abdominal pain and is subsequently diagnosed with appendicitis. The patient is scheduled for an appendectomy but questions the nurse about how his health will be affected by the absence of an appendix. How should the nurse best respond? A) Your appendix doesn't play a major role, so you wont notice any difference after you recovery from surgery. B) The surgeon will encourage you to limit your fat intake for a few weeks after the surgery, but your body will then begin to compensate. C) Your body will absorb slightly fewer nutrients from the food you eat, but you wont be aware of this. D) Your large intestine will adapt over time to the absence of your appendix.

A

A patient has just had an arthroscopy performed to assess a knee injury. What nursing intervention should the nurse implement following this procedure? A) Wrap the joint in a compression dressing. B) Perform passive range of motion exercises. C) Maintain the knee in flexion for up to 30 minutes. D) Apply heat to the knee.

A

A patient has sought care because of recent dark-colored stools. As a result, a fecal occult blood test has been ordered. The nurse should instruct the patient to avoid which of the following prior to collecting a stool sample? A) NSAIDs B) Acetaminophen C) OTC vitamin D supplements D) Fiber supplements

A

A patient with glaucoma has presented for a scheduled clinic visit and tells the nurse that she has begun taking an herbal remedy for her condition that was recommended by a work colleague. What instruction should the nurse provide to the patient? A) The patient should discuss this new remedy with her ophthalmologist promptly. B) The patient should monitor her IOP closely for the next several weeks. C) The patient should do further research on the herbal remedy. D) The patient should report any adverse effects to her pharmacist.

A

A patient with otosclerosis has significant hearing loss. What should the nurse do to best facilitate communication with the patient? A) Sit or stand in front of the patient when speaking. B) Use exaggerated lip and mouth movements when talking. C) Stand in front of a light or window when speaking. D) Say the patient's name loudly before starting to talk

A

A patient's ocular tumor has necessitated enucleation and the patient will be fitted with a prosthesis. The nurse should address what nursing diagnosis when planning the patient's discharge education? A) Disturbed body image B) Chronic pain C) Ineffective protection D) Unilateral neglect

A

A patients fracture is healing and callus is being deposited in the bone matrix. This process characterizes what phase of the bone healing process? A) The reparative phase B) The reactive phase C) The remodeling phase D) The revascularization phase

A

An older adult patient has symptoms of osteoporosis and is being assessed during her annual physical examination. The assessment shows that the patient will require further testing related to a possible exacerbation of her osteoporosis. The nurse should anticipate what diagnostic test? A) Bone densitometry B) Hip bone radiography C) Computed tomography (CT) D) Magnetic resonance imaging (MRI)

A

Diagnostic tests show that a patients bone density has decreased over the past several years. The patient asks the nurse what factors contribute to bone density decreasing. What would be the nurses best response? A) For many people, lack of nutrition can cause a loss of bone density. B) Progressive loss of bone density is mostly related to your genes. C) Stress is known to have many unhealthy effects, including reduced bone density. D) Bone density decreases with age, but scientists are not exactly sure why this is the case.

A

The nurse has taken shift report on her patients and has been told that one patient has an ocular condition that has primarily affected the rods in his eyes. Considering this information, what should the nurse do while caring for the patient? A) Ensure adequate lighting in the patient's room. B) Provide a dimly lit room to aid vision by limiting contrast. C) Carefully point out color differences for the patient. D) Carefully point out fine details for the patient.

A

The nurse is admitting a 55-year-old male patient diagnosed with a retinal detachment in his left eye. While assessing this patient, what characteristic symptom would the nurse expect to find? A) Flashing lights in the visual field B) Sudden eye pain C) Loss of color vision D) Colored halos around lights

A

The nurse should recognize the greatest risk for the development of blindness in which of the following patients? A) A 58-year-old Caucasian woman with macular degeneration B) A 28-year-old Caucasian man with astigmatism C) A 58-year-old African American woman with hyperopia D) A 28-year-old African American man with myopia

A

A nurse is admitting a patient to the postsurgical unit following a gastrostomy. When planning assessments, the nurse should be aware of what potential postoperative complication of a gastrostomy? A) Premature removal of the G tube B) Bowel perforation C) Constipation D) Development of peptic ulcer disease (PUD)

A Feedback: A significant postoperative complication of a gastrostomy is premature removal of the G tube. Constipation is a less immediate threat and bowel perforation and PUD are not noted to be likely complications.

A nurse is caring for a patient who has an order to discontinue the administration of parenteral nutrition. What should the nurse do to prevent the occurrence of rebound hypoglycemia in the patient? A) Administer an isotonic dextrose solution for 1 to 2 hours after discontinuing the PN. B) Administer a hypertonic dextrose solution for 1 to 2 hours after discontinuing the PN. C) Administer 3 ampules of dextrose 50% immediately prior to discontinuing the PN. D) Administer 3 ampules of dextrose 50% 1 hour after discontinuing the PN.

A Feedback: After administration of the PN solution is gradually discontinued, an isotonic dextrose solution is administered for 1 to 2 hours to protect against rebound hypoglycemia. The other listed actions would likely cause hyperglycemia.

A patient's new onset of dysphagia has required insertion of an NG tube for feeding; the nurse has modified the patient's care plan accordingly. What intervention should the nurse include in the patient's plan of care? A) Confirm placement of the tube prior to each medication administration. B) Have the patient sip cool water to stimulate saliva production. C) Keep the patient in a low Fowler's position when at rest. D) Connect the tube to continuous wall suction when not in use.

A Feedback: Each time liquids or medications are administered, and once a shift for continuous feedings, the tube must be checked to ensure that it remains properly placed. If the NG tube is used for decompression, it is attached to intermittent low suction. During the placement of a nasogastric tube the patient should be positioned in a Fowler's position. Oral fluid administration is contraindicated by the patient's dysphagia.

A patient's health decline necessitates the use of total parenteral nutrition. The patient has questioned the need for insertion of a central venous catheter, expressing a preference for a "normal IV." The nurse should know that peripheral administration of high-concentration PN formulas is contraindicated because of the risk for what complication? A) Chemical phlebitis B) Hyperglycemia C) Dumping syndrome D) Line sepsis

A Feedback: Formulations with dextrose concentrations of more than 10% should not be administered through peripheral veins because they irritate the intima (innermost walls) of small veins, causing chemical phlebitis. Hyperglycemia and line sepsis are risks with both peripheral and central administration of PN. PN is not associated with dumping syndrome.

A nurse is preparing to administer a patient's intravenous fat emulsion simultaneously with parenteral nutrition (PN). Which of the following principles should guide the nurse's action? A) Intravenous fat emulsions may be infused simultaneously with PN through a Y-connector close to the infusion site and should not be filtered. B) The nurse should prepare for placement of another intravenous line, as intravenous fat emulsions may not be infused simultaneously through the line used for PN. C) Intravenous fat emulsions may be infused simultaneously with PN through a Y-connector close to the infusion site after running the emulsion through a filter. D) The intravenous fat emulsions can be piggy-backed into any existing IV solution that is infusing.

A Feedback: Intravenous fat emulsions may be infused simultaneously with PN through a Y-connector close to the infusion site and should not be filtered. The patient does not need another intravenous line for the fat emulsion. The IVFE cannot be piggy-backed into any existing IV solution that is infusing.

The nurse is administering total parenteral nutrition (TPN) to a client who underwent surgery for gastric cancer. Which of the nurse's assessments most directly addresses a major complication of TPN? A) Checking the patient's capillary blood glucose levels regularly B) Having the patient frequently rate his or her hunger on a 10-point scale C) Measuring the patient's heart rhythm at least every 6 hours D) Monitoring the patient's level of consciousness each shift

A Feedback: The solution, used as a base for most TPN, consists of a high dextrose concentration and may raise blood glucose levels significantly, resulting in hyperglycemia. This is a more salient threat than hunger, though this should be addressed. Dysrhythmias and decreased LOC are not among the most common complications.

A patient has a gastrostomy tube that has been placed to drain stomach contents by low intermittent suction. What is the nurse's priority during this aspect of the patient's care? A) Measure and record drainage. B) Monitor drainage for change in color. C) Titrate the suction every hour. D) Feed the patient via the G tube as ordered.

A Feedback: This drainage should be measured and recorded because it is a significant indicator of GI function. The nurse should indeed monitor the color of the output, but fluid balance is normally the priority. Frequent titration of the suction should not be necessary and feeding is contraindicated if the G tube is in place for drainage.

A nurse is preparing to place a patient's ordered nasogastric tube. How should the nurse best determine the correct length of the nasogastric tube? A) Place distal tip to nose, then ear tip and end of xiphoid process. B) Instruct the patient to lie prone and measure tip of nose to umbilical area. C) Insert the tube into the patient's nose until secretions can be aspirated. D) Obtain an order from the physician for the length of tube to insert.

A Feedback: Tube length is traditionally determined by (1) measuring the distance from the tip of the nose to the earlobe and from the earlobe to the xiphoid process, and (2) adding up to 6 inches for NG placement or at least 8 to 10 inches or more for intestinal placement, although studies do not necessarily confirm that this is a reliable technique. The physician would not prescribe a specific length and the umbilicus is not a landmark for this process. Length is not determined by aspirating from the tube.

A patient has been experiencing significant psychosocial stress in recent weeks. The nurse is aware of the hormonal effects of stress, including norepinephrine release. Release of this substance would have what effect on the patients gastrointestinal function? Select all that apply. A) Decreased motility B) Increased sphincter tone C) Increased enzyme release D) Inhibition of secretions E) Increased peristalsis

A Feedback: Norepinephrine generally decreases GI motility and secretions, but increases muscle tone of sphincters. Norepinephrine does not increase the release of enzyme

A patient with Cushing's syndrome as a result of a pituitary tumor has been admitted for a transsphenoidal hypophysectomy. What would be most important for the nurse to monitor before, during, and after surgery? A) Blood glucose B) Assessment of urine for blood C) Weight D) Oral temperature

A) Blood glucose

An unresponsive Caucasian patient has been brought to the emergency room by EMS. While assessing this patient, the nurse notes that the patients face is a cherry-red color. What should the nurse suspect? A) Carbon monoxide poisoning B) Anemia C) Jaundice D) Uremia

A) Carbon monoxide poisoning Carbon monoxide poisoning causes a bright cherry red color in the face and upper torso in light-skinned persons. In dark-skinned persons, there will be a cherry red color to nail beds, lips, and oral mucosa. When anemia occurs in light-skinned persons, the skin has generalized pallor. Anemia in dark-skinned persons manifests as a yellow-brown coloration. Jaundice appears as a yellow coloration of the sclerae. Uremia gives a yellow-orange tinge to the skin.

The nurse in an ambulatory care center is admitting an older adult patient who has bright red moles on the skin. Benign changes in elderly skin that appear as bright red moles are termed what? A) Cherry angiomas B) Solar lentigo C) Seborrheickeratoses D) Xanthelasma

A) Cherry angiomas Cherry angiomas appear as bright red moles, while solar lentigo are commonly called liver spots. Seborrheickeratoses are described as crusty brown stuck on patches, while xanthelasma appears as yellowish, waxy deposits on the upper eyelids.

A nurse is conducting a health interview and is assessing for integumentary conditions that are known to have a genetic component. What assessment question is most appropriate? A) Does anyone in your family have eczema or psoriasis? B) Have any of your family members been diagnosed with malignant melanoma? C) Do you have a family history of vitiligo or port-wine stains? D) Does any member of your family have a history of keloid scarring?

A) Does anyone in your family have eczema or psoriasis?

Following an addisonian crisis, a patient's adrenal function has been gradually regained. The nurse should ensure that the patient knows about the need for supplementary glucocorticoid therapy in which of the following circumstances? A) Episodes of high psychosocial stress B) Periods of dehydration C) Episodes of physical exertion D) Administration of a vaccine

A) Episodes of high psychosocial stress

The nurse is providing care for a patient with Chushing's syndrome has identified the nursing diagnosis of risk for injury r/t weakness. How should the nurse best reduce this risk? A) Establish falls prevention B) Encourage bed rest whenever possible C) Encourage the use of assistive devices D) Provide constant supervision

A) Establish falls prevention

The nurse is caring for a patient diagnosed with hypothyroidism secondary to Hashimoto's thyroiditis. When assessing this patient, what sign or symptom would the nurse expect? A) Fatigue B) Bulging eyes C) Palpitations D) Flushed skin

A) Fatigue

A patient with Cushing's syndrome has been hospitalized after a fall. The dietitian consulted works with the patient to improve the patient's nutritional intake. What foods should a patient with Cushing's syndrome eat to optimize health? Select all that apply. A) Foods high in vitamin D B) Foods high in calories C) Foods high in proteins D) Foods high in calcium E) Foods high in sodium

A) Foods high in vitamin D C) Foods high in proteins D) Foods high in calcium

A nurse in the emergency department (ED) is triaging a 5-year-old who has been brought to the ED by her parents for an outbreak of urticaria. What would be the most appropriate question to ask this patient and her family? A) Has she eaten any new foods today? B) Has she bathed in the past 24 hours? C) Did she go to a friends house today? D) Was she digging in the dirt today?

A) Has she eaten any new foods today? Foods can cause skin reactions, especially in children. In most cases, this is a more plausible cause of urticaria than bathing, contact with other children, or soil-borne pathogens.

A patient presents at the walk-in clinic complaining of diarrhea and vomiting. The patient has a documented history of adrenal insufficiency. Considering the patient's history and current symptoms, the nurse should anticipate that the patient will be instructed to do which of the following? A) Increase his intake of sodium until the GI symptoms improve. B) Increase his intake of potassium until the GI symptoms improve. C) Increase his intake of glucose until the GI symptoms improve. D) Increase his intake of calcium until the GI symptoms improve.

A) Increase his intake of sodium until the GI symptoms improve.

You are developing a care plan for a patient with Cushing's syndrome. What nursing diagnosis would have the highest priority in this care plan? A) Risk for injury r/t weakness B) Ineffective breathing pattern r/t muscle weakneess C) Risk for loneliness r/t disturbed body image D) Autonomic dysreflexia r/t neurologic changes

A) Risk for injury r/t weakness

An older adult patient is diagnosed with a vitamin D deficiency. What would be an appropriate recommendation by the nurse? A) Spend time outdoors at least twice per week B) Increase intake of leafy green vegetables C) Start taking a multivitamin each morning D) Eat red meat at least once per week

A) Spend time outdoors at least twice per week Skin exposed to ultraviolet light can convert substances necessary for synthesizing vitamin D (cholecalciferol). It is estimated that most people need five to thirty minutes of sun exposure twice a week in order for this synthesis to occur. Multivitamins may not resolve a specific vitamin D deficiency. Vitamin D is unrelated to meat and vegetable intake.

An emergency department nurse has just received a patient with burn injuries brought in by ambulance. The paramedics have started a large-bore IV and covered the burn in cool towels. The burn is estimated as covering 24% of the patients body. How should the nurse best address the pathophysiologic changes resulting from major burns during the initial burn shock period? A) administer IV fluids B) administer broad-spectrum antibiotics C) administer IV potassium chloride D) administer packed red blood cells

A) administer IV fluids

A client comes to the clinic for an evaluation. During the health history, the client reports feeling tired all the time and just being overall sluggish. The nurse suspects that the client may be experiencing hypothyroidism. What assessment findings would the nurse expect to identify to confirm this suspicion? select all that apply. A) coarse, dry hair B) brittle nails C) diarrhea D) peripheral edema E) irritability

A) coarse, dry hair B) brittle nails D) peripheral edema

A nurse is assessing a client with acromegaly. Which finding would the nurse most likely assess? select all that apply. A) enlarged feet B) height greater than 7 feet C) broad nose D) enlarged tongue E) carpal tunnel syndrome

A) enlarged feet C) broad nose D) enlarged tongue E) carpal tunnel syndrome

A client diagnosed with thyroid cancer has undergone a thyroidectomy and is returning to the unit. When developing the clients plan of care, which action would be a priority? A) having a tracheostomy tray at the bedside B) ensuring adequate IV fluid replacement C) performing frequent vital sign checks D) administering analgesics

A) having a tracheostomy tray at the bedside

A client is suspected of having central diabetes insipidus and is scheduled to undergo a vasopressin challenge test. When preparing the client for this test, the nurse anticipates that the test would be done: A) in the morning after fasting B) just after breakfast C) in the middle of the afternoon D) immediately before bedtime

A) in the morning after fasting

A nurse is providing care to a client who undergone a transphenoidal hypophysectomy. The nurse checks the clients nasal packing for drainage. Which finding would confirm the presence of CSF in the drainage. select all that apply. A) positive halo ring B) positive beta-2 transferrin C) positive glucose D) positive total protein E) positive chloride

A) positive halo ring B) positive beta-2 transferrin

While reviewing a client's medical record, the nurse notes that the client has hypothyroidism resulting from a dysfunction of the thyroid gland itself. The nurse identifies this as which type of hypothyroidism? A) primary B) central C) secondary D) tertiary

A) primary

A patient with suspected adrenal insufficiency has been ordered an adrenocorticotropic hormone (ACTH) stimulation test. Administration of ACTH caused a marked increase in cortisol levels. How should the nurse interpret this finding? A) the patients pituitary function is compromised B) the patients adrenal insufficiency is not treatable C) the patient has insufficient hypothalamic function D) the patient would benefit from surgery

A) the patients pituitary function is compromised

A nurse is explaining a patients decreasing bone density in terms of the balance between bone resorption and formation. What dietary nutrients and hormones play a role in the resorption and formation of adult bones? Select all that apply. A) Thyroid hormone B) Growth hormone C) Estrogen D) Vitamin B12 E) Luteinizing hormone

A, B, C

A nurse is preparing to discharge a patient home on parenteral nutrition. What should an effective home care teaching program address? Select all that apply. A) Preparing the patient to troubleshoot for problems B) Teaching the patient and family strict aseptic technique C) Teaching the patient and family how to set up the infusion D) Teaching the patient to flush the line with sterile water E) Teaching the patient when it is safe to leave the access site open to air

A, B, C Feedback: An effective home care teaching program prepares the patient to store solutions, set up the infusion, flush the line with heparin, change the dressings, and troubleshoot for problems. The most common complication is sepsis. Strict aseptic technique is taught for hand hygiene, handling equipment, changing the dressing, and preparing the solution. Sterile water is never used for flushes and the access site must never be left open to air.

A nurse is caring for a patient admitted to the medical unit with a diagnosis of pemphigus vulgaris. When writing the care plan for this patient, what nursing diagnoses should be included? Select all that apply. A) Risk for Infection Related to Lesions B) Impaired Skin Integrity Related to Epidermal Blisters C) Disturbed Body Image Related to Presence of Skin Lesions D) Acute Pain Related to Disruption in Skin Integrity E) Hyperthermia Related to Disruptions in Thermoregulation

A, B, C, D

A patient has been discharged home on parenteral nutrition (PN). Much of the nurse's discharge education focused on coping. What must a patient on PN likely learn to cope with? Select all that apply. A) Changes in lifestyle B) Loss of eating as a social behavior C) Chronic bowel incontinence from GI changes D) Sleep disturbances related to frequent urination during nighttime infusions E) Stress of choosing the correct PN formulation

A, B, D Feedback: Patients must cope with the loss of eating as a social behavior and with changes in lifestyle brought on by sleep disturbances related to frequent urination during night time infusions. PN is not associated with bowel incontinence and the patient does not select or adjust the formulation of PN.

A nurse educator is teaching a group of medical nurses about Kaposis sarcoma. What would the educator identify as characteristics of endemic Kaposis sarcoma? Select all that apply. A) Affects people predominantly in the eastern half of Africa B) Affects men more than women C) Does not affect children D) Cannot infiltrate E) Can progress to lymphadenopathic forms

A, B, E

The nurse is assessing a patient for dietary factors that may influence her risk for osteoporosis. The nurse should question the patient about her intake of what nutrients? Select all that apply. A) Calcium B) Simple carbohydrates C) Vitamin D D) Protein E) Soluble fiber

A, C

A nurse is caring for a patient who has a gastrointestinal tube in place. Which of the following are indications for gastrointestinal intubation? Select all that apply. A) To remove gas from the stomach B) To administer clotting factors to treat a GI bleed C) To remove toxins from the stomach D) To open sphincters that are closed E) To diagnose GI motility disorders

A, C, E Feedback: GI intubation may be performed to decompress the stomach and remove gas and fluid, lavage (flush with water or other fluids) the stomach and remove ingested toxins or other harmful materials, diagnose disorders of GI motility and other disorders, administer medications and feedings, compress a bleeding site, and aspirate gastric contents for analysis. GI intubation is not used for opening sphincters that are not functional or for administering clotting factors.

The nurse is providing care for a patient who has benefited from a cochlear implant. The nurse should understand that this patient's health history likely includes which of the following? Select all that apply. A) The patient was diagnosed with sensorineural hearing loss. B) The patient's hearing did not improve appreciably with the use of hearing aids. C) The patient has deficits in peripheral nervous function. D) The patient's hearing deficit is likely accompanied by a cognitive deficit. E) The patient is unable to lip-read.

A,B

A public health nurse is teaching a health promotion workshop that focuses on vision and eye health. What should this nurse cite as the most common causes of blindness and visual impairment among adults over the age of 40? Select all that apply. A) Diabetic retinopathy B) Trauma C) Macular degeneration D) Cytomegalovirus E) Glaucoma

A,C,E

A patient is scheduled for enucleation and the nurse is providing anticipatory guidance about postoperative care. What aspects of care should the nurse describe to the patient? Select all that apply. A) Application of topical antibiotic ointment B) Maintenance of a supine position for the first 48 hours postoperative C) Fluid restriction to prevent orbital edema D) Administration of loop diuretics to prevent orbital edema E) Use of an ocular pressure dressing

A,E

A patient exhibiting an uncoordinated gait has presented at the clinic. Which of the following is the most plausible cause of this patient's health problem? A- cerebellar dysfunction B- a lesion in the pons C- dysfunction of the medulla D- a hemorrhage in the midbrain

A- cerebellar dysfunction

A patient is being given a medication that stimulates her parasympathetic system. Following administration of this medication, the nurse should anticipate what effect? A- constricted pupils B- dilated bronchioles C- decreased peristaltic movement D- relaxed muscular walls of the urinary bladder

A- constricted pupils

A patient is scheduled for a myelogram and the nurse explains to the patient that this is an invasive procedure, which assesses for any lesion in the spinal cord. The nurse should explain that the preparation is similar to which of the following neurologic tests? A- lumbar puncture B- MRI C- cerebral angiography D- EEG

A- lumbar puncture

A female client is admitted to the medical unit for evaluation of cerebral metastasis from a primary site. when reviewing the clients history, the nurse would most likely find which site as being the primary site? A- lungs B- prostate C- renal D- uterus

A- lungs

A nursing student is writing a care plan for a newly admitted patient who has been diagnosed with a stroke. What major nursing diagnosis should most likely be included in the patient's plan of care? A) Adult failure to thrive B) Post-trauma syndrome C) Hyperthermia D) Disturbed sensory perception

Ans: D Feedback: The patient who has experienced a stroke is at a high risk for disturbed sensory perception. Stroke is associated with multiple other nursing diagnoses, but hyperthermia, adult failure to thrive, and post-trauma syndrome are not among these.

A patient with end-stage liver disease has developed hypervolemia. What nursing interventions would be most appropriate when addressing the patient's fluid volume excess? Select all that apply. A. Administering diuretics B. Administering calcium channel blockers C. Implementing fluid restrictions D. Implementing a 1500 kcal/day restriction E. Enhancing patient positioning

A. Administering diuretics B. Administering calcium channel blockers E. Enhancing patient positioning Rationale: Administering diuretics, implementing fluid restrictions, and enhancing patient positioning can optimize the management of fluid volume excess. Calcium channel blockers and calorie restriction do not address this problem.

A patient's physician has ordered a liver panel in response to the patient's development of jaundice. When reviewing the results of this laboratory testing, the nurse should expect to review what blood tests? Select all that apply. A. Alanine aminotransferase (ALT) B. C-reactive protein (CRP) C. Gamma-glutamyl transferase (GGT) D. Aspartate aminotransferase (AST) E. B-type natriuretic peptide (BNP)

A. Alanine aminotransferase (ALT) B. C-reactive protein (CRP) D. Aspartate aminotransferase (AST) Rationale: Liver function testing includes GGT, ALT, and AST. CRP addresses the presence of generalized inflammation and BNP is relevant to heart failure; neither is included in a liver panel.

A nurse is caring for a patient with hepatic encephalopathy. While making the initial shift assessment, the nurse notes that the patient has a flapping tremor of the hands. The nurse should document the presence of what sign of liver disease? A. Asterixis B. Constructional apraxia C. Fector hepaticus D. Palmar erythema

A. Asterixis Rationale: The nurse will document that a patient exhibiting a flapping tremor of the hands is demonstrating asterixis. While constructional apraxia is a motor disturbance, it is the inability to reproduce a simple figure. Fector hepaticus is a sweet, slightly fecal odor to the breath and not associated with a motor disturbance. Skin changes associated with liver dysfunction may include palmar erythema, which is a reddening of the palms, but is not a flapping tremor.

A 55-year-old female patient with hepatocellular carcinoma (HCC) is undergoing radiofrequency ablation. The nurse should recognize what goal of this treatment? A. Destruction of the patient's liver tumor B. Restoration of portal vein patency C. Destruction of a liver abscess D. Reversal of metastasis

A. Destruction of the patient's liver tumor Rationale: Using radiofrequency ablation, a tumor up to 5 cm in size can be destroyed in one treatment session. This technique does not address circulatory function or abscess formation. It does not allow for the reversal of metastasis.

A group of nurses have attended an inservice on the prevention of occupationally acquired diseases that affect healthcare providers. What action has the greatest potential to reduce a nurse's risk of acquiring hepatitis C in the workplace? A. Disposing of sharps appropriately and not recapping needles B. Performing meticulous hand hygiene at the appropriate moments in care C. Adhering to the recommended schedule of immunizations D. Wearing an N95 mask when providing care for patients on airborne precautions

A. Disposing of sharps appropriately and not recapping needles Rationale: HCV is blood-borne. Consequently, prevention of needle stick injuries are paramount. Hand hygiene, immunizations and appropriate use of masks are important aspects of overall infection control, but these actions do not directly mitigate the risk of HCV.

A nurse is caring for a patient with cancer of the liver whose condition has required the insertion of a percutaneous biliary drainage system. The nurse's most recent assessment reveals the presence of dark green fluid in the collection container. What is the nurse's best response to this assessment finding? A. Document the presence of normal bile output. B. Irrigate the drainage system with normal saline as ordered. C. Aspirate a sample of the drainage for culture. D. Promptly report this assessment finding to the primary care provider.

A. Document the presence of normal bile output. Rationale: Bile is usually a dark green or brownish-yellow color, so this would constitute an expected assessment finding, with no other action necessary.

During a health education session, a participant has asked about the hepatitis E virus. What prevention measure should the nurse recommend for preventing infection with this virus? A. Following proper hand-washing techniques B. Avoiding chemicals that are toxic to the liver C. Wearing a condom during sexual contact D. Limiting alcohol intake

A. Following proper hand-washing techniques Rationale: Avoiding contact with the hepatitis E virus through good hygiene, including hand-washing, is the major method of prevention. Hepatitis E is transmitted by the fecaloral route, principally through contaminated water in areas with poor sanitation. Consequently, none of the other listed preventative measure is indicated.

A triage nurse in the emergency department is assessing a patient who presented with complaints of general malaise. Assessment reveals the presence of jaundice and increased abdominal girth. What assessment question best addresses the possible etiology of this patient's presentation? A. How many alcoholic drinks do you typically consume in a week? B. To the best of your knowledge, are your immunizations up to date? C. Have you ever worked in an occupation where you might have been exposed to toxins? D. Has anyone in your family ever experienced symptoms similar to yours?

A. How many alcoholic drinks do you typically consume in a week? Rationale: Signs or symptoms of hepatic dysfunction indicate a need to assess for alcohol use. Immunization status, occupational risks, and family history are also relevant considerations, but alcohol use is a more common etiologic factor in liver disease.

A nurse educate is teaching a group of recent nursing graduates about their occupational risks for contracting hepatitis B. What preventative measures should the educator promote? Select all that apply. A. Immunizations B. Use of standard precautions C. Consumption of vitamin-rich diet D. Annual vitamin K injections E. Annual vitamin B12 injections

A. Immunizations B. Use of standard precautions Rationale: People who are at high risk, including nurses and other health care personnel exposed to blood or blood products, should receive active immunization. The consistent use of standard precautions is also highly beneficial. Vitamin supplementation is unrelated to an individual's risk of HBV.

A nurse on a solid organ transplant unit is planning the care of a patient who will soon be admitted upon immediate recovery following liver transplantation. What aspect of nursing care is the nurse's priority? A. Implementation of infection-control measures B. Close monitoring of skin integrity and color C. Frequent assessment of the patient's psychosocial status D. Administration of antiretroviral medications

A. Implementation of infection-control measures Rationale: Infection control is paramount following liver transplantation. This is a priority over skin integrity and psychosocial status, even though these are valid areas of assessment and intervention. Antiretrovirals are not indicated.

A nurse is presenting an educational event to a local community group. When speaking about colorectal cancer, what risk factor should the nurse cite? • High levels of alcohol consumption • History of bowel obstruction • History of diverticulitis • Longstanding psychosocial stress

Ans: A Feedback: Risk factors include high alcohol intake; cigarette smoking; and high-fact, high-protein, low-fiber diet. Diverticulitis, obstruction, and stress are not noted as risk factors for colorectal cancer.

A patient with a liver mass is undergoing a percutaneous liver biopsy. What action should the nurse perform when assisting with this procedure? A. Position the patient on the right side with a pillow under the costal margin after the procedure B. Administer 1 unit of albumin 90 minutes before the procedure as ordered C. Administer at least 1 unit of packed red blood cells as ordered the day before the scheduled procedure D. Confirm that the patient's electrolyte levels have been assessed prior to the procedure

A. Position the patient on the right side with a pillow under the costal margin after the procedure. Rationale: Immediately after a percutaneous liver biopsy, assist the patient to turn onto the right side and place a pillow under the costal margin. Prior administration of albumin or PRBC's is unnecessary. Coagulation tests should be performed, but electrolyte analysis is not necessary.

A patient is being discharged after a liver transplant and the nurse is performing discharge education. When planning the patient's continuing care, the nurse should prioritize which of the following risk diagnoses? A. Risk for infection related to immunosuppressant use B. Risk for injury related to decreased hemostasis C. Risk for unstable blood glucose related to impaired gluconeogensis D. Risk for contamination related to accumulation of ammonia

A. Risk for infection related to immunosuppressant use Rationale: Infection is the leading cause of death after liver transplantation. Pulmonary and fungal infections are common; susceptibility to infection is increased by the immunosuppressive therapy that is needed to prevent rejection. This risk exceeds the threats of injury and unstable blood glucose. The diagnosis of risk for contamination related to environmental toxin exposure.

A patient has developed hepatic encephalopathy secondary to cirrhosis and is receiving care on the medical unit. The patient's current medication regimen includes lactulose (Cephulac) four times daily. What desired outcome should the nurse relate to this pharmacologic intervention? A. Two to 3 soft bowel movements daily B. Significant increase in appetite and food intake C. Absence of nausea and vomiting D. Absence of blood or mucus in stool

A. Two to 3 soft bowel movements daily Rationale: Lactulose (Cephulac) is administered to reduce serum ammonia levels. Two or three soft stools per day are desirable; this indicates that lactulose is performing as intended. Lactulose does not address the patient's appetite, symptoms of nausea and vomiting, or the development of blood and mucus in the stool.

A patient who just suffered a suspected ischemic stroke is brought to the ED by ambulance. On what should the nurse's primary assessment focus? A) Cardiac and respiratory status B) Seizure activity C) Pain D) Fluid and electrolyte balance

Ans: A Feedback: Acute care begins with managing ABCs. Patients may have difficulty keeping an open and clear airway secondary to decreased LOC. Neurologic assessment with close monitoring for signs of increased neurologic deficit and seizure activity occurs next. Fluid and electrolyte balance must be controlled carefully with the goal of adequate hydration to promote perfusion and decrease further brain activity.

A patient is admitted to the neurologic ICU with a suspected diffuse axonal injury. What would be the primary neuroimaging diagnostic tool used on this patient to evaluate the brain structure? A) MRI B) PET scan C) X-ray D) Ultrasound

Ans: A Feedback: CT and MRI scans, the primary neuroimaging diagnostic tools, are useful in evaluating the brain structure. Ultrasound would not show the brain nor would an x-ray. A PET scan shows brain function, not brain structure.

A patient with a T2 injury is in spinal shock. The nurse will expect to observe what assessment finding? A) Absence of reflexes along with flaccid extremities B) Positive Babinski's reflex along with spastic extremities C) Hyperreflexia along with spastic extremities D) Spasticity of all four extremities

Ans: A Feedback: During the period immediately following a spinal cord injury, spinal shock occurs. In spinal shock, all reflexes are absent and the extremities are flaccid. When spinal shock subsides, the patient demonstrates a positive Babinski's reflex, hyperreflexia, and spasticity of all four extremities

When preparing to discharge a patient home, the nurse has met with the family and warned them that the patient may exhibit unexpected emotional responses. The nurse should teach the family that these responses are typically a result of what cause? A) Frustration around changes in function and communication B) Unmet physiologic needs C) Changes in brain activity during sleep and wakefulness D) Temporary changes in metabolism

Ans: A Feedback: Emotional problems associated with stroke are often related to the new challenges around ADLs and communication. These challenges are more likely than metabolic changes, unmet physiologic needs, or changes in brain activity, each of which should be ruled out.

The nurse is assessing a patient with a suspected stroke. What assessment finding is most suggestive of a stroke? A) Facial droop B) Dysrhythmias C) Periorbital edema D) Projectile vomiting

Ans: A Feedback: Facial drooping or asymmetry is a classic abnormal finding on a physical assessment that may be associated with a stroke. Facial edema is not suggestive of a stroke and patients less commonly experience dysrhythmias or vomiting.

The nurse is planning the care of a patient with a T1 spinal cord injury. The nurse has identified the diagnosis of "risk for impaired skin integrity." How can the nurse best address this risk? A) Change the patient's position frequently. B) Provide a high-protein diet. C) Provide light massage at least daily. D) Teach the patient deep breathing and coughing exercises.

Ans: A Feedback: Frequent position changes are among the best preventative measures against pressure ulcers. A high-protein diet can benefit wound healing, but does not necessarily prevent skin breakdown. Light massage and deep breathing do not protect or restore skin integrity.

After a major ischemic stroke, a possible complication is cerebral edema. Nursing care during the immediate recovery period from an ischemic stroke should include which of the following? A) Positioning to avoid hypoxia B) Maximizing PaCO2 C) Administering hypertonic IV solution D) Initiating early mobilization

Ans: A Feedback: Interventions during this period include measures to reduce ICP, such as administering an osmotic diuretic (e.g., mannitol), maintaining the partial pressure of carbon dioxide (PaCO2) within the range of 30 to 35 mm Hg, and positioning to avoid hypoxia. Hypertonic IV solutions are not used unless sodium depletion is evident. Mobilization would take place after the immediate threat of increased ICP has past.

A patient who suffered an ischemic stroke now has disturbed sensory perception. What principle should guide the nurse's care of this patient? A) The patient should be approached on the side where visual perception is intact. B) Attention to the affected side should be minimized in order to decrease anxiety. C) The patient should avoid turning in the direction of the defective visual field to minimize shoulder subluxation. D) The patient should be approached on the opposite side of where the visual perception is intact to promote recovery.

Ans: A Feedback: Patients with decreased field of vision should first be approached on the side where visual perception is intact. All visual stimuli should be placed on this side. The patient can and should be taught to turn the head in the direction of the defective visual field to compensate for this loss. The nurse should constantly remind the patient of the other side of the body and should later stand at a position that encourages the patient to move or turn to visualize who and what is in the room.

A nurse is assessing a patient with an acoustic neuroma who has been recently admitted to an oncology unit. What symptoms is the nurse likely to find during the initial assessment? A) Loss of hearing, tinnitus, and vertigo B) Loss of vision, change in mental status, and hyperthermia C) Loss of hearing, increased sodium retention, and hypertension D) Loss of vision, headache, and tachycardia

Ans: A-loss of hearing, tinnitus and vertigo Feedback: An acoustic neuroma is a tumor of the eighth cranial nerve, the cranial nerve most responsible for hearing and balance. The patient with an acoustic neuroma usually experiences loss of hearing, tinnitus, and episodes of vertigo and staggering gait. Acoustic neuromas do not cause loss of vision, increased sodium retention, or tachycardia.

The patient has been diagnosed with aphasia after suffering a stroke. What can the nurse do to best make the patient's atmosphere more conducive to communication? A) Provide a board of commonly used needs and phrases. B) Have the patient speak to loved ones on the phone daily. C) Help the patient complete his or her sentences. D) Speak in a loud and deliberate voice to the patient.

Ans: A Feedback: The inability to talk on the telephone or answer a question or exclusion from conversation causes anger, frustration, fear of the future, and hopelessness. A common pitfall is for the nurse or other health care team member to complete the thoughts or sentences of the patient. This should be avoided because it may cause the patient to feel more frustrated at not being allowed to speak and may deter efforts to practice putting thoughts together and completing a sentence. The patient may also benefit from a communication board, which has pictures of commonly requested needs and phrases. The board may be translated into several languages.

Following diagnostic testing, a patient has been admitted to the ICU and placed on cerebral aneurysm precautions. What nursing action should be included in patient's plan of care? A) Supervise the patient's activities of daily living closely. B) Initiate early ambulation to prevent complications of immobility. C) Provide a high-calorie, low-protein diet. D) Perform all of the patient's hygiene and feeding.

Ans: A Feedback: The patient is placed on immediate and absolute bed rest in a quiet, nonstressful environment, because activity, pain, and anxiety elevate BP, which increases the risk for bleeding. As such, independent ADLs and ambulation are contraindicated. There is no need for a high-calorie or low-protein diet.

The nurse is providing health education to a patient who has a C6 spinal cord injury. The patient asks why autonomic dysreflexia is considered an emergency. What would be the nurse's best answer? A) "The sudden increase in BP can raise the ICP or rupture a cerebral blood vessel." B) "The suddenness of the onset of the syndrome tells us the body is struggling to maintain its normal state." C) "Autonomic dysreflexia causes permanent damage to delicate nerve fibers that are healing." D) "The sudden, severe headache increases muscle tone and can cause further nerve damage."

Ans: A Feedback: The sudden increase in BP may cause a rupture of one or more cerebral blood vessels or lead to increased ICP. Autonomic dysreflexia does not directly cause nerve damage.

A patient is brought by ambulance to the ED after suffering what the family thinks is a stroke. The nurse caring for this patient is aware that an absolute contraindication for thrombolytic therapy is what? A) Evidence of hemorrhagic stroke B) Blood pressure of ³ 180/110 mm Hg C) Evidence of stroke evolution D) Previous thrombolytic therapy within the past 12 months

Ans: A Feedback: Thrombolytic therapy would exacerbate a hemorrhagic stroke with potentially fatal consequences. Stroke evolution, high BP, or previous thrombolytic therapy does not contraindicate its safe and effective use.

During a patient's recovery from stroke, the nurse should be aware of predictors of stroke outcome in order to help patients and families set realistic goals. What are the predictors of stroke outcome? Select all that apply. A) National Institutes of Health Stroke Scale (NIHSS) score B) Race C) LOC at time of admission D) Gender E) Age

Ans: A, C, E Feedback: It is helpful for clinicians to be knowledgeable about the relative importance of predictors of stroke outcome (age, NIHSS score, and LOC at time of admission) to provide stroke survivors and their families with realistic goals. Race and gender are not predictors of stroke outcome.

The school nurse is giving a presentation on preventing spinal cord injuries (SCI). What should the nurse identify as prominent risk factors for SCI? Select all that apply. A) Young age B) Frequent travel C) African American race D) Male gender E) Alcohol or drug use

Ans: A, D, E Feedback: The predominant risk factors for SCI include young age, male gender, and alcohol and drug use. Ethnicity and travel are not risk factors.

26. The nurse is planning the care of a patient who has been recently diagnosed with a cerebellar tumor. Due to the location of this patient's tumor, the nurse should implement measures to prevent what complication? A) Falls B) Audio hallucinations C) Respiratory depression D) Labile BP

Ans: A- falls Feedback: A cerebellar tumor causes dizziness, an ataxic or staggering gait with a tendency to fall toward the side of the lesion, and marked muscle incoordination. Because of this, the patient faces a high risk of falls. Hallucinations and unstable vital signs are not closely associated with cerebellar tumors.

27. A patient has been admitted to the neurologic ICU with a diagnosis of a brain tumor. The patient is scheduled to have a tumor resection/removal in the morning. Which of the following assessment parameters should the nurse include in the initial assessment? A) Gag reflex B) Deep tendon reflexes C) Abdominal girth D) Hearing acuity

Ans: A- gag reflex Feedback: Preoperatively, the gag reflex and ability to swallow are evaluated. In patients with diminished gag response, care includes teaching the patient to direct food and fluids toward the unaffected side, having the patient sit upright to eat, offering a semisoft diet, and having suction readily available. Deep tendon reflexes, abdominal girth, and hearing acuity are less commonly affected by brain tumors and do not affect the risk for aspiration.

29. A patient with an inoperable brain tumor has been told that he has a short life expectancy. On what aspects of assessment and care should the home health nurse focus? Select all that apply. A) Pain control B) Management of treatment complications C) Interpretation of diagnostic tests D) Assistance with self-care E) Administration of treatments

Ans: A- pain control B- management of treatment complications D- assistance with self care E- administration of treatments Feedback: Home care needs and interventions focus on four major areas: palliation of symptoms and pain control, assistance in self-care, control of treatment complications, and administration of specific forms of treatment, such as parenteral nutrition. Interpretation of diagnostic tests is normally beyond the purview of the nurse.

25. A male patient presents at the free clinic with complaints of impotency. Upon physical examination, the nurse practitioner notes the presence of hypogonadism. What diagnosis should the nurse suspect? A) Prolactinoma B) Angioma C) Glioma D) Adrenocorticotropic hormone (ACTH)-producing adenoma

Ans: A- prolactinoma Feedback: Male patients with prolactinomas may present with impotence and hypogonadism. An ACTH-producing adenoma would cause acromegaly. The scenario contains insufficient information to know if the tumor is an angioma, glioma, or neuroma.

When caring for a patient who had a hemorrhagic stroke, close monitoring of vital signs and neurologic changes is imperative. What is the earliest sign of deterioration in a patient with a hemorrhagic stroke of which the nurse should be aware? A) Generalized pain B) Alteration in level of consciousness (LOC) C) Tonic-clonic seizures D) Shortness of breath

Ans: B Feedback: Alteration in LOC is the earliest sign of deterioration in a patient after a hemorrhagic stroke, such as mild drowsiness, slight slurring of speech, and sluggish papillary reaction. Sudden headache may occur, but generalized pain is less common. Seizures and shortness of breath are not identified as early signs of hemorrhagic stroke.

A nurse is caring for a critically ill patient with autonomic dysreflexia. What clinical manifestations would the nurse expect in this patient? A) Respiratory distress and projectile vomiting B) Bradycardia and hypertension C) Tachycardia and agitation D) Third-spacing and hyperthermia

Ans: B Feedback: Autonomic dysreflexia is characterized by a pounding headache, profuse sweating, nasal congestion, piloerection ("goose bumps"), bradycardia, and hypertension. It occurs in cord lesions above T6 after spinal shock has resolved; it does not result in vomiting, tachycardia

The nurse is caring for a patient recovering from an ischemic stroke. What intervention best addresses a potential complication after an ischemic stroke? A) Providing frequent small meals rather than three larger meals B) Teaching the patient to perform deep breathing and coughing exercises C) Keeping a urinary catheter in situ for the full duration of recovery D) Limiting intake of insoluble fiber

Ans: B Feedback: Because pneumonia is a potential complication of stroke, deep breathing and coughing exercises should be encouraged unless contraindicated. No particular need exists to provide frequent meals and normally fiber intake should not be restricted. Urinary catheters should be discontinued as soon as possible

A patient has been admitted to the ICU after being recently diagnosed with an aneurysm and the patient's admission orders include specific aneurysm precautions. What nursing action will the nurse incorporate into the patient's plan of care? A) Elevate the head of the bed to 45 degrees. B) Maintain the patient on complete bed rest. C) Administer enemas when the patient is constipated. D) Avoid use of thigh-high elastic compression stockings.

Ans: B Feedback: Cerebral aneurysm precautions are implemented for the patient with a diagnosis of aneurysm to provide a nonstimulating environment, prevent increases in ICP, and prevent further bleeding. The patient is placed on immediate and absolute bed rest in a quiet, nonstressful environment because activity, pain, and anxiety elevate BP, which increases the risk for bleeding. Visitors, except for family, are restricted. The head of the bed is elevated 15 to 30 degrees to promote venous drainage and decrease ICP. Some neurologists, however, prefer that the patient remains flat to increase cerebral perfusion. No enemas are permitted, but stool softeners and mild laxatives are prescribed. Thigh-high elastic compression stockings or sequential compression boots may be ordered to decrease the patient's risk for deep vein thrombosis (DVT).

Following a spinal cord injury a patient is placed in halo traction. While performing pin site care, the nurse notes that one of the traction pins has become detached. The nurse would be correct in implementing what priority nursing action? A) Complete the pin site care to decrease risk of infection. B) Notify the neurosurgeon of the occurrence. C) Stabilize the head in a lateral position. D) Reattach the pin to prevent further head trauma.

Ans: B Feedback: If one of the pins became detached, the head is stabilized in neutral position by one person while another notifies the neurosurgeon. Reattaching the pin as a nursing intervention would not be done due to risk of increased injury. Pin site care would not be a priority in this instance. Prevention of neurologic injury is the priority.

A rehabilitation nurse caring for a patient who has had a stroke is approached by the patient's family and asked why the patient has to do so much for herself when she is obviously struggling. What would be the nurse's best answer? A) "We are trying to help her be as useful as she possibly can." B) "The focus on care in a rehabilitation facility is to help the patient to resume as much self-care as possible." C) "We aren't here to care for her the way the hospital staff did; we are here to help her get better so she can go home." D) "Rehabilitation means helping patients do exactly what they did before their stroke."

Ans: B Feedback: In both acute care and rehabilitation facilities, the focus is on teaching the patient to resume as much self-care as possible. The goal of rehabilitation is not to be "useful," nor is it to return patients to their prestroke level of functioning, which may be unrealistic.

A patient with spinal cord injury has a nursing diagnosis of altered mobility and the nurse recognizes the increased the risk of deep vein thrombosis (DVT). Which of the following would be included as an appropriate nursing intervention to prevent a DVT from occurring? A) Placing the patient on a fluid restriction as ordered B) Applying thigh-high elastic stockings C) Administering an antifibrinolyic agent D) Assisting the patient with passive range of motion (PROM) exercises

Ans: B Feedback: It is important to promote venous return to the heart and prevent venous stasis in a patient with altered mobility. Applying elastic stockings will aid in the prevention of a DVT. The patient should not be placed on fluid restriction because a dehydrated state will increase the risk of clotting throughout the body. Antifibrinolytic agents cause the blood to clot, which is absolutely contraindicated in this situation. PROM exercises are not an effective protection against the development of DVT.

The staff educator is precepting a nurse new to the critical care unit when a patient with a T2 spinal cord injury is admitted. The patient is soon exhibiting manifestations of neurogenic shock. In addition to monitoring the patient closely, what would be the nurse's most appropriate action? A) Prepare to transfuse packed red blood cells. B) Prepare for interventions to increase the patient's BP. C) Place the patient in the Trendelenberg position. D) Prepare an ice bath to lower core body temperature.

Ans: B Feedback: Manifestations of neurogenic shock include decreased BP and heart rate. Cardiac markers would be expected to rise in cardiogenic shock. Transfusion, repositioning, and ice baths are not indicated interventions.

A patient is admitted to the neurologic ICU with a spinal cord injury. When assessing the patient the nurse notes there is a sudden depression of reflex activity in the spinal cord below the level of injury. What should the nurse suspect? A) Epidural hemorrhage B) Hypertensive emergency C) Spinal shock D) Hypovolemia

Ans: C Feedback: In spinal shock, the reflexes are absent, BP and heart rate fall, and respiratory failure can occur. Hypovolemia, hemorrhage, and hypertension do not cause this sudden change in neurologic function.

A neurologic flow chart is often used to document the care of a patient with a traumatic brain injury. At what point in the patient's care should the nurse begin to use a neurologic flow chart? A) When the patient's condition begins to deteriorate B) As soon as the initial assessment is made C) At the beginning of each shift D) When there is a clinically significant change in the patient's condition

Ans: B Feedback: Neurologic parameters are assessed initially and as frequently as the patient's condition requires. As soon as the initial assessment is made, the use of a neurologic flowchart is started and maintained. A new chart is not begun at the start of every shift.

A patient who has sustained a nondepressed skull fracture is admitted to the acute medical unit. Nursing care should include which of the following? A) Preparation for emergency craniotomy B) Watchful waiting and close monitoring C) Administration of inotropic drugs D) Fluid resuscitation

Ans: B Feedback: Nondepressed skull fractures generally do not require surgical treatment; however, close observation of the patient is essential. A craniotomy would not likely be needed if the fracture is nondepressed. Even if treatment is warranted, it is unlikely to include inotropes or fluid resuscitation.

A family member brings the patient to the clinic for a follow-up visit after a stroke. The family member asks the nurse what he can do to decrease his chance of having another stroke. What would be the nurse's best answer? A) "Have your heart checked regularly." B) "Stop smoking as soon as possible." C) "Get medication to bring down your sodium levels." D) "Eat a nutritious diet."

Ans: B Feedback: Smoking is a modifiable and highly significant risk factor for stroke. The significance of smoking, and the potential benefits of quitting, exceed the roles of sodium, diet, and regular medical assessments.

A nurse is reviewing the trend of a patient's scores on the Glasgow Coma Scale (GCS). This allows the nurse to gauge what aspect of the patient's status? A) Reflex activity B) Level of consciousness C) Cognitive ability D) Sensory involvement

Ans: B Feedback: The Glasgow Coma Scale (GCS) examines three responses related to LOC: eye opening, best verbal response, and best motor response.

The nurse is performing stroke risk screenings at a hospital open house. The nurse has identified four patients who might be at risk for a stroke. Which patient is likely at the highest risk for a hemorrhagic stroke? A) White female, age 60, with history of excessive alcohol intake B) White male, age 60, with history of uncontrolled hypertension C) Black male, age 60, with history of diabetes D) Black male, age 50, with history of smoking

Ans: B Feedback: Uncontrolled hypertension is the primary cause of a hemorrhagic stroke. Control of hypertension, especially in individuals over 55 years of age, clearly reduces the risk for hemorrhagic stroke. Additional risk factors are increased age, male gender, and excessive alcohol intake. Another high-risk group includes African Americans, where the incidence of first stroke is almost twice that as in Caucasians.

The nurse is caring for a patient who is rapidly progressing toward brain death. The nurse should be aware of what cardinal signs of brain death? Select all that apply. A) Absence of pain response B) Apnea C) Coma D) Absence of brain stem reflexes E) Absence of deep tendon reflexes

Ans: B, C, D Feedback: The three cardinal signs of brain death upon clinical examination are coma, the absence of brain stem reflexes, and apnea. Absences of pain response and deep tendon reflexes are not necessarily indicative of brain death.

As a member of the stroke team, the nurse knows that thrombolytic therapy carries the potential for benefit and for harm. The nurse should be cognizant of what contraindications for thrombolytic therapy? Select all that apply. A) INR above 1.0 B) Recent intracranial pathology C) Sudden symptom onset D) Current anticoagulation therapy E) Symptom onset greater than 3 hours prior to admission

Ans: B, D, E Feedback: Some of the absolute contraindications for thrombolytic therapy include symptom onset greater than 3 hours before admission, a patient who is anticoagulated (with an INR above 1.7), or a patient who has recently had any type of intracranial pathology (e.g., previous stroke, head injury, trauma)

A 25-year-old female patient with brain metastases is considering her life expectancy after her most recent meeting with her oncologist. Based on the fact that the patient is not receiving treatment for her brain metastases, what is the nurse's most appropriate action? A) Promoting the patient's functional status and ADLs B) Ensuring that the patient receives adequate palliative care C) Ensuring that the family does not tell the patient that her condition is terminal D) Promoting adherence to the prescribed medication regimen

Ans: B- ensuring that the patient receives adequate palliative care Feedback: Patients with intracerebral metastases who are not treated have a steady downhill course with a limited survival time, whereas those who are treated may survive for slightly longer periods, but for most cure is not possible. Palliative care is thus necessary. This is a priority over promotion of function and the family should not normally withhold information from the patient. Adherence to medications such as analgesics is important, but palliative care is a high priority.

13. A patient has been admitted to the neurologic unit for the treatment of a newly diagnosed brain tumor. The patient has just exhibited seizure activity for the first time. What is the nurse's priority response to this event? A) Identify the triggers that precipitated the seizure. B) Implement precautions to ensure the patient's safety. C) Teach the patient's family about the relationship between brain tumors and seizure activity. D) Ensure that the patient is housed in a private room.

Ans: B- implement precautions to ensure the patients safety Feedback: Patients with seizures are carefully monitored and protected from injury. Patient safety is a priority over health education, even though this is appropriate and necessary. Specific triggers may or may not be evident; identifying these is not the highest priority. A private room is preferable, but not absolutely necessary.

14. A patient diagnosed with a pituitary adenoma has arrived on the neurologic unit. When planning the patient's care, the nurse should be aware that the effects of the tumor will primarily depend on what variable? A) Whether the tumor utilizes aerobic or anaerobic respiration B) The specific hormones secreted by the tumor C) The patient's pre-existing health status D) Whether the tumor is primary or the result of metastasis

Ans: B- the specific hormones secreted by the tumor Feedback: Functioning pituitary tumors can produce one or more hormones normally produced by the anterior pituitary and the effects of the tumor depend largely on the identity of these hormones. This variable is more significant than the patient's health status or whether the tumor is primary versus secondary. Anaerobic and aerobic respiration is not relevant.

A patient is admitted to the neurologic ICU with a C4 spinal cord injury. When writing the plan of care for this patient, which of the following nursing diagnoses would the nurse prioritize in the immediate care of this patient? A) Risk for impaired skin integrity related to immobility and sensory loss B) Impaired physical mobility related to loss of motor function C) Ineffective breathing patterns related to weakness of the intercostal muscles D) Urinary retention related to inability to void spontaneously

Ans: C Feedback: A nursing diagnosis related to breathing pattern would be the priority for this patient. A C4 spinal cord injury will require ventilatory support, due to the diaphragm and intercostals being affected. The other nursing diagnoses would be used in the care plan, but not designated as a higher priority than ineffective breathing patterns.

A patient has had an ischemic stroke and has been admitted to the medical unit. What action should the nurse perform to best prevent joint deformities? A) Place the patient in the prone position for 30 minutes/day. B) Assist the patient in acutely flexing the thigh to promote movement. C) Place a pillow in the axilla when there is limited external rotation. D) Place patient's hand in pronation.

Ans: C Feedback: A pillow in the axilla prevents adduction of the affected shoulder and keeps the arm away from the chest. The prone position with a pillow under the pelvis, not flat, promotes hyperextension of the hip joints, essential for normal gait. To promote venous return and prevent edema, the upper thigh should not be flexed acutely. The hand is placed in slight supination, not pronation, which is its most functional position.

A community health nurse is giving an educational presentation about stroke and heart disease at the local senior citizens center. What nonmodifiable risk factor for stroke should the nurse cite? A) Female gender B) Asian American race C) Advanced age D) Smoking

Ans: C Feedback: Advanced age, male gender, and race are well-known nonmodifiable risk factors for stroke. High-risk groups include people older than 55 years of age; the incidence of stroke more than doubles in each successive decade. Men have a higher rate of stroke than that of women. Another high-risk group is African Americans; the incidence of first stroke in African Americans is almost twice that as in Caucasian Americans; Asian American race is not a risk factor. Smoking is a modifiable risk.

The ED nurse is caring for a patient who has been brought in by ambulance after sustaining a fall at home. What physical assessment finding is suggestive of a basilar skull fracture? A) Epistaxis B) Periorbital edema C) Bruising over the mastoid D) Unilateral facial numbness

Ans: C Feedback: An area of ecchymosis (bruising) may be seen over the mastoid (Battle's sign) in a basilar skull fracture. Numbness, edema, and epistaxis are not directly associated with a basilar skull fracture.

A patient is brought to the ED by her family after falling off the roof. A family member tells the nurse that when the patient fell she was "knocked out," but came to and "seemed okay." Now she is complaining of a severe headache and not feeling well. The care team suspects an epidural hematoma, prompting the nurse to prepare for which priority intervention? A) Insertion of an intracranial monitoring device B) Treatment with antihypertensives C) Emergency craniotomy D) Administration of anticoagulant therapy

Ans: C Feedback: An epidural hematoma is considered an extreme emergency. Marked neurologic deficit or respiratory arrest can occur within minutes. Treatment consists of making an opening through the skull to decrease ICP emergently, remove the clot, and control the bleeding. Antihypertensive medications would not be a priority. Anticoagulant therapy should not be ordered for a patient who has a cranial bleed. This could further increase bleeding activity. Insertion of an intracranial monitoring device may be done during the surgery, but is not priority for this patient.

The nurse is discharging home a patient who suffered a stroke. He has a flaccid right arm and leg and is experiencing problems with urinary incontinence. The nurse makes a referral to a home health nurse because of an awareness of what common patient response to a change in body image? A) Denial B) Fear C) Depression D) Disassociation

Ans: C Feedback: Depression is a common and serious problem in the patient who has had a stroke. It can result from a profound disruption in his or her life and changes in total function, leaving the patient with a loss of independence. The nurse needs to encourage the patient to verbalize feelings to assess the effect of the stroke on self-esteem. Denial, fear, and disassociation are not the most common patient response to a change in body image, although each can occur in some patients.

A patient diagnosed with a hemorrhagic stroke has been admitted to the neurologic ICU. The nurse knows that teaching for the patient and family needs to begin as soon as the patient is settled on the unit and will continue until the patient is discharged. What will family education need to include? A) How to differentiate between hemorrhagic and ischemic stroke B) Risk factors for ischemic stroke C) How to correctly modify the home environment D) Techniques for adjusting the patient's medication dosages at home

Ans: C Feedback: For a patient with a hemorrhagic stroke, teaching addresses the use of assistive devices or modification of the home environment to help the patient live with the disability. This is more important to the patient's needs than knowing about risk factors for ischemic stroke. It is not necessary for the family to differentiate between different types of strokes. Medication regimens should never be altered without consultation.

The nurse is caring for a patient diagnosed with an ischemic stroke and knows that effective positioning of the patient is important. Which of the following should be integrated into the patient's plan of care? A) The patient's hip joint should be maintained in a flexed position. B) The patient should be in a supine position unless ambulating. C) The patient should be placed in a prone position for 15 to 30 minutes several times a day. D) The patient should be placed in a Trendelenberg position two to three times daily to promote cerebral perfusion.

Ans: C Feedback: If possible, the patient is placed in a prone position for 15 to 30 minutes several times a day. A small pillow or a support is placed under the pelvis, extending from the level of the umbilicus to the upper third of the thigh. This helps to promote hyperextension of the hip joints, which is essential for normal gait, and helps prevent knee and hip flexion contractures. The hip joints should not be maintained in flexion and the Trendelenberg position is not indicated.

A nurse is performing a nursing assessment of a patient suspected of having a musculoskeletal disorder. What is the primary focus of the nursing assessment with a patient who has a musculoskeletal disorder? A) Range of motion B) Activities of daily living C) Gait D) Strength

B

A patient is admitted to the neurologic ICU with a spinal cord injury. In writing the patient's care plan, the nurse specifies that contractures can best be prevented by what action? A) Repositioning the patient every 2 hours B) Initiating range-of-motion exercises (ROM) as soon as the patient initiates C) Initiating (ROM) exercises as soon as possible after the injury D) Performing ROM exercises once a day

Ans: C Feedback: Passive ROM exercises should be implemented as soon as possible after injury. It would be inappropriate to wait for the patient to first initiate exercises. Toes, metatarsals, ankles, knees, and hips should be put through a full ROM at least four, and ideally five, times daily. Repositioning alone will not prevent contractures.

The nurse is reviewing the medication administration record of a female patient who possesses numerous risk factors for stroke. Which of the woman's medications carries the greatest potential for reducing her risk of stroke? A) Naproxen 250 PO b.i.d. B) Calcium carbonate 1,000 mg PO b.i.d. C) Aspirin 81 mg PO o.d. D) Lorazepam 1 mg SL b.i.d. PRN

Ans: C Feedback: Research findings suggest that low-dose aspirin may lower the risk of stroke in women who are at risk. Naproxen, lorazepam, and calcium supplements do not have this effect.

Splints have been ordered for a patient who is at risk of developing footdrop following a spinal cord injury. The nurse caring for this patient knows that the splints are removed and reapplied when? A) At the patient's request B) Each morning and evening C) Every 2 hours D) One hour prior to mobility exercises

Ans: C Feedback: The feet are prone to footdrop; therefore, various types of splints are used to prevent footdrop. When used, the splints are removed and reapplied every 2 hours.

A patient diagnosed with transient ischemic attacks (TIAs) is scheduled for a carotid endarterectomy. The nurse explains that this procedure will be done for what purpose? A) To decrease cerebral edema B) To prevent seizure activity that is common following a TIA C) To remove atherosclerotic plaques blocking cerebral flow D) To determine the cause of the TIA

Ans: C Feedback: The main surgical procedure for select patients with TIAs is carotid endarterectomy, the removal of an atherosclerotic plaque or thrombus from the carotid artery to prevent stroke in patients with occlusive disease of the extracranial arteries. An endarterectomy does not decrease cerebral edema, prevent seizure activity, or determine the cause of a TIA.

The nurse is preparing health education for a patient who is being discharged after hospitalization for a hemorrhagic stroke. What content should the nurse include in this education? A) Mild, intermittent seizures can be expected. B) Take ibuprofen for complaints of a serious headache. C) Take antihypertensive medication as ordered. D) Drowsiness is normal for the first week after discharge.

Ans: C Feedback: The patient and family are provided with information that will enable them to cooperate with the care and restrictions required during the acute phase of hemorrhagic stroke and to prepare the patient to return home. Patient and family teaching includes information about the causes of hemorrhagic stroke and its possible consequences. Symptoms of hydrocephalus include gradual onset of drowsiness and behavioral changes. Hypertension is the most serious risk factor, suggesting that appropriate antihypertensive treatment is essential for a patient being discharged. Seizure activity is not normal; complaints of a serious headache should be reported to the physician before any medication is taken. Drowsiness is not normal or expected.

The nurse caring for a patient with a spinal cord injury notes that the patient is exhibiting early signs and symptoms of disuse syndrome. Which of the following is the most appropriate nursing action? A) Limit the amount of assistance provided with ADLs. B) Collaborate with the physical therapist and immobilize the patient's extremities temporarily. C) Increase the frequency of ROM exercises. D) Educate the patient about the importance of frequent position changes.

Ans: C Feedback: To prevent disuse syndrome, ROM exercises must be provided at least four times a day, and care is taken to stretch the Achilles tendon with exercises. The patient is repositioned frequently and is maintained in proper body alignment whether in bed or in a wheelchair. The patient must be repositioned by caregivers, not just taught about repositioning. It is inappropriate to limit assistance for the sole purpose of preventing disuse syndrome.

A patient with a spinal cord injury has experienced several hypotensive episodes. How can the nurse best address the patient's risk for orthostatic hypotension? A) Administer an IV bolus of normal saline prior to repositioning. B) Maintain bed rest until normal BP regulation returns. C) Monitor the patient's BP before and during position changes. D) Allow the patient to initiate repositioning.

Ans: C Feedback: To prevent hypotensive episodes, close monitoring of vital signs before and during position changes is essential. Prolonged bed rest carries numerous risks and it is not possible to provide a bolus before each position change. Following the patient's lead may or may not help regulate BP.

21. The nurse educator is discussing neoplasms with a group of recent graduates. The educator explains that the effects of neoplasms are caused by the compression and infiltration of normal tissue. The physiologic changes that result can cause what pathophysiologic events? Select all that apply. A) Intracranial hemorrhage B) Infection of cerebrospinal fluid C) Increased ICP D) Focal neurologic signs E) Altered pituitary function

Ans: C- increased ICP D- focal neurologic signs e- altered pituitary function Feedback: The effects of neoplasms are caused by the compression and infiltration of tissue. A variety of physiologic changes result, causing any or all of the following pathophysiologic events: increased ICP and cerebral edema, seizure activity and focal neurologic signs, hydrocephalus, and altered pituitary function.

The nurse is writing a care plan for a patient with brain metastases. The nurse decides that an appropriate nursing diagnosis is "anxiety related to lack of control over the health circumstances." In establishing this plan of care for the patient, the nurse should include what intervention? A) The patient will receive antianxiety medications every 4 hours. B) The patient's family will be instructed on planning the patient's care. C) The patient will be encouraged to verbalize concerns related to the disease and its treatment. D) The patient will begin intensive therapy with the goal of distraction.

Ans: C- the patient will be encouraged to verbalize concerns related to the disease and its treatment Feedback: Patients need the opportunity to exercise some control over their situation. A sense of mastery can be gained as they learn to understand the disease and its treatment and how to deal with their feelings. Distraction and administering medications will not allow the patient to gain control over anxiety. Delegating planning to the family will not help the patient gain a sense of control and autonomy.

24. A patient who has been experiencing numerous episodes of unexplained headaches and vomiting has subsequently been referred for testing to rule out a brain tumor. What characteristic of the patient's vomiting is most consistent with a brain tumor? A) The patient's vomiting is accompanied by epistaxis. B) The patient's vomiting does not relieve his nausea. C) The patient's vomiting is unrelated to food intake. D) The patient's emesis is blood-tinged.

Ans: C- the patients vomiting is unrelated to food intake Feedback: Vomiting is often unrelated to food intake if caused by a brain tumor. The presence or absence of blood is not related to the possible etiology and vomiting may or may not relieve the patient's nausea.

22. The nurse is caring for a patient newly diagnosed with a primary brain tumor. The patient asks the nurse where his tumor came from. What would be the nurse's best response? A) "Your tumor originated from somewhere outside the CNS." B) "Your tumor likely started out in one of your glands." C) "Your tumor originated from cells within your brain itself." D) "Your tumor is from nerve tissue somewhere in your body."

Ans: C- your tumor originated from cells within your brain itself Feedback: Primary brain tumors originate from cells and structures within the brain. Secondary brain tumors are metastatic tumors that originate somewhere else in the body. The scenario does not indicate that the patient's tumor is a pituitary tumor or a neuroma.

A patient diagnosed with a cerebral aneurysm reports a severe headache to the nurse. What action is a priority for the nurse? A) Sit with the patient for a few minutes. B) Administer an analgesic. C) Inform the nurse-manager. D) Call the physician immediately.

Ans: D Feedback: A headache may be an indication that the aneurysm is leaking. The nurse should notify the physician immediately. The physician will decide whether administration of an analgesic is indicated. Informing the nurse-manager is not necessary. Sitting with the patient is appropriate, once the physician has been notified of the change in the patient's condition.

A nurse in the ICU is providing care for a patient who has been admitted with a hemorrhagic stroke. The nurse is performing frequent neurologic assessments and observes that the patient is becoming progressively more drowsy over the course of the day. What is the nurse's best response to this assessment finding? A) Report this finding to the physician as an indication of decreased metabolism. B) Provide more stimulation to the patient and monitor the patient closely. C) Recognize this as the expected clinical course of a hemorrhagic stroke. D) Report this to the physician as a possible sign of clinical deterioration.

Ans: D Feedback: Alteration in LOC often is the earliest sign of deterioration in a patient with a hemorrhagic stroke. Drowsiness and slight slurring of speech may be early signs that the LOC is deteriorating. This finding is unlikely to be the result of metabolic changes and it is not expected. Stimulating a patient with an acute stroke is usually contraindicated.

The nurse has implemented interventions aimed at facilitating family coping in the care of a patient with a traumatic brain injury. How can the nurse best facilitate family coping? A) Help the family understand that the patient could have died. B) Emphasize the importance of accepting the patient's new limitations. C) Have the members of the family plan the patient's inpatient care. D) Assist the family in setting appropriate short-term goals.

Ans: D Feedback: Helpful interventions to facilitate coping include providing family members with accurate and honest information and encouraging them to continue to set well-defined, short-term goals. Stating that a patient's condition could be worse downplays their concerns. Emphasizing the importance of acceptance may not necessarily help the family accept the patient's condition. Family members cannot normally plan a patient's hospital care, although they may contribute to the care in some ways.

What should be included in the patient's care plan when establishing an exercise program for a patient affected by a stroke? A) Schedule passive range of motion every other day. B) Keep activity limited, as the patient may be over stimulated. C) Have the patient perform active range-of-motion (ROM) exercises once a day. D) Exercise the affected extremities passively four or five times a day.

Ans: D Feedback: The affected extremities are exercised passively and put through a full ROM four or five times a day to maintain joint mobility, regain motor control, prevent development of a contracture in the paralyzed extremity, prevent further deterioration of the neuromuscular system, and enhance circulation. Active ROM exercises should ideally be performed more than once per day.

An ED nurse has just received a call from EMS that they are transporting a 17-year-old man who has just sustained a spinal cord injury (SCI). The nurse recognizes that the most common cause of this type of injury is what? A) Sports-related injuries B) Acts of violence C) Injuries due to a fall D) Motor vehicle accidents

Ans: D Feedback: The most common causes of SCIs are motor vehicle crashes (46%), falls (22%), violence (16%), and sports (12%).

A patient with a cerebral aneurysm exhibits signs and symptoms of an increase in intracranial pressure (ICP). What nursing intervention would be most appropriate for this patient? A) Range-of-motion exercises to prevent contractures B) Encouraging independence with ADLs to promote recovery C) Early initiation of physical therapy D) Absolute bed rest in a quiet, nonstimulating environment

Ans: D Feedback: The patient is placed on immediate and absolute bed rest in a quiet, nonstressful environment because activity, pain, and anxiety elevate BP, which increases the risk for bleeding. Visitors are restricted. The nurse administers all personal care. The patient is fed and bathed to prevent any exertion that might raise BP.

After a subarachnoid hemorrhage, the patient's laboratory results indicate a serum sodium level of less than 126 mEq/L. What is the nurse's most appropriate action? A) Administer a bolus of normal saline as ordered. B) Prepare the patient for thrombolytic therapy as ordered. C) Facilitate testing for hypothalamic dysfunction. D) Prepare to administer 3% NaCl by IV as ordered.

Ans: D Feedback: The patient may be experiencing syndrome of inappropriate antidiuretic hormone (SIADH) or cerebral salt-wasting syndrome. The treatment most often is the use of IV hypertonic 3% saline. A normal saline bolus would exacerbate the problem and there is no indication for tests of hypothalamic function or thrombolytic therapy.

A female patient is diagnosed with a right-sided stroke. The patient is now experiencing hemianopsia. How might the nurse help the patient manage her potential sensory and perceptional difficulties? A) Keep the lighting in the patient's room low. B) Place the patient's clock on the affected side. C) Approach the patient on the side where vision is impaired. D) Place the patient's extremities where she can see them.

Ans: D Feedback: The patient with homonymous hemianopsia (loss of half of the visual field) turns away from the affected side of the body and tends to neglect that side and the space on that side; this is called amorphosynthesis. In such instances, the patient cannot see food on half of the tray, and only half of the room is visible. It is important for the nurse to remind the patient constantly of the other side of the body, to maintain alignment of the extremities, and if possible, to place the extremities where the patient can see them. Patients with a decreased field of vision should be approached on the side where visual perception is intact. All visual stimuli (clock, calendar, and television) should be placed on this side. The patient can be taught to turn the head in the direction of the defective visual field to compensate for this loss. Increasing the natural or artificial lighting in the room and providing eyeglasses are important in increasing vision. There is no reason to keep the lights dim.

A patient recovering from a stroke has severe shoulder pain from subluxation of the shoulder and is being cared for on the unit. To prevent further injury and pain, the nurse caring for this patient is aware of what principle of care? A) The patient should be fitted with a cast because use of a sling should be avoided due to adduction of the affected shoulder. B) Elevation of the arm and hand can lead to further complications associated with edema. C) Passively exercising the affected extremity is avoided in order to minimize pain. D) The patient should be taught to interlace fingers, place palms together, and slowly bring scapulae forward to avoid excessive force to shoulder.

Ans: D Feedback: To prevent shoulder pain, the nurse should never lift a patient by the flaccid shoulder or pull on the affected arm or shoulder. The patient is taught how to move and exercise the affected arm/shoulder through proper movement and positioning. The patient is instructed to interlace the fingers, place the palms together, and push the clasped hands slowly forward to bring the scapulae forward; he or she then raises both hands above the head. This is repeated throughout the day. The use of a properly worn sling when the patient is out of bed prevents the paralyzed upper extremity from dangling without support. Range-of-motion exercises are still vitally important in preventing a frozen shoulder and ultimately atrophy of subcutaneous tissues, which can cause more pain. Elevation of the arm and hand is also important in preventing dependent edema of the hand.

7. A 37-year-old man is brought to the clinic by his wife because he is experiencing loss of motor function and sensation. The physician suspects the patient has a spinal cord tumor and hospitalizes him for diagnostic testing. In light of the need to diagnose spinal cord compression from a tumor, the nurse will most likely prepare the patient for what test? A) Anterior-posterior x-ray B) Ultrasound C) Lumbar puncture D) MRI

Ans: D- MRI Feedback: The MRI scan is the most commonly used diagnostic procedure. It is the most sensitive diagnostic tool that is particularly helpful in detecting epidural spinal cord compression and vertebral bone metastases.

9. While assessing the patient at the beginning of the shift, the nurse inspects a surgical dressing covering the operative site after the patients' cervical diskectomy. The nurse notes that the drainage is 75% saturated with serosanguineous discharge. What is the nurse's most appropriate action? A) Page the physician and report this sign of infection. B) Reinforce the dressing and reassess in 1 to 2 hours. C) Reposition the patient to prevent further hemorrhage. D) Inform the surgeon of the possibility of a dural leak.

Ans: D- inform the surgeon of the possibility of a dural leak Feedback: After a cervical diskectomy, the nurse will monitor the operative site and dressing covering this site. Serosanguineous drainage may indicate a dural leak. This constitutes a risk for meningitis, but is not a direct sign of infection. This should be reported to the surgeon, not just reinforced and observed.

10. A patient, diagnosed with cancer of the lung, has just been told he has metastases to the brain. What change in health status would the nurse attribute to the patient's metastatic brain disease? A) Chronic pain B) Respiratory distress C) Fixed pupils D) Personality changes

Ans: D- personality changes Feedback: Neurologic signs and symptoms include headache, gait disturbances, visual impairment, personality changes, altered mentation (memory loss and confusion), focal weakness, paralysis, aphasia, and seizures. Pain, respiratory distress, and fixed pupils are not among the more common neurologic signs and symptoms of metastatic brain disease.

15. A male patient with a metastatic brain tumor is having a generalized seizure and begins vomiting. What should the nurse do first? A) Perform oral suctioning. B) Page the physician. C) Insert a tongue depressor into the patient's mouth. D) Turn the patient on his side.

Ans: D- turn the patient on his side Feedback: The nurse's first response should be to place the patient on his side to prevent him from aspirating emesis. Inserting something into the seizing patient's mouth is no longer part of a seizure protocol. Obtaining supplies to suction the patient would be a delegated task. Paging or calling the physician would only be necessary if this is the patient's first seizure.

A patient is admitted to the medical unit with a diagnosis of intestinal obstruction. When planning this patients care, which of the following nursing diagnoses should the nurse prioritize? • Ineffective Tissue Perfusion Related to Bowel Ischemia • Imbalanced Nutrition: Less Than Body Requirements Related to Impaired Absorption • Anxiety Related to Bowel Obstruction and Subsequent Hospitalization • Impaired Skin Integrity Related to Bowel Obstruction

Ans: A Feedback: When the bowel is completely obstructed, the possibility of strangulation and tissue necrosis (i.e., tissue death) warrants surgical intervention. As such, this immediate physiologic need is a nursing priority. Nutritional support and management of anxiety are necessary, but bowel ischemia is a more immediate threat. Skin integrity is not threatened.

37. A hospital patient has experienced a seizure. In the immediate recovery period, what action best protects the patients safety? A) Place the patient in a side-lying position. B) Pad the patients bed rails. C) Administer antianxiety medications as ordered. D) Reassure the patient and family members.

Ans: A Feedback: To prevent complications, the patient is placed in the side-lying position to facilitate drainage of oral secretions. Suctioning is performed, if needed, to maintain a patent airway and prevent aspiration. None of the other listed actions promotes safety during the immediate recovery period.

A nurse is providing care for a patient who has a diagnosis of irritable bowel syndrome (IBS). When planning this patients care, the nurse should collaborate with the patient and prioritize what goal? A. Patient will accurately identify foods that trigger symptoms. B. Patient will demonstrate appropriate care of his ileostomy. C. Patient will demonstrate appropriate use of standard infection control precautions. D. Patient will adhere to recommended guidelines for mobility and activity.

Ans: A Feedback: A major focus of nursing care for the patient with IBS is to identify factors that exacerbate symptoms. Surgery is not used to treat this health problem and infection control is not a concern that is specific to this diagnosis. Establishing causation likely is more important to the patient than managing physical activity.

A nurse is caring for a patient admitted with symptoms of an anorectal infection; cultures indicate that the patient has a viral infection. The nurse should anticipate the administration of what drug? • Acyclovir (Zovirax) • Doxycycline (Vibramycin) • Penicillin (penicillin • Metronidazole (Flagyl)

Ans: A Feedback: Acyclovir (Zovirax) is often given to patients with viral anorectal infections. Doxycycline (Vibramycin) and penicillin (penicillin G) are drugs of choice for bacterial infections. Metronidazole (Flagyl) is used for other infections with a bacterial etiology; it is ineffective against viruses.

A 35-year-old male patient presents at the emergency department with symptoms of a small bowel obstruction. In collaboration with the primary care provider, what intervention should the nurse prioritize? • Insertion of a nasogastric tube • Insertion of a central venous catheter • Administration of a mineral oil enema • Administration of a glycerin suppository and an oral laxative

Ans: A Feedback: Decompression of the bowel through a nasogastric tube is necessary for all patients with small bowel obstruction. Peripheral IV access is normally sufficient. Enemas, suppositories, and laxatives are not indicated if an obstruction is present.

The nurse is providing care for a patient with chronic obstructive pulmonary disease. When describing the process of respiration the nurse explains how oxygen and carbon dioxide are exchanged between the pulmonary capillaries and the alveoli. The nurse is describing what process? A) Diffusion B) Osmosis C) Active transport D) Filtration

Ans: A Feedback: Diffusion is the natural tendency of a substance to move from an area of higher concentration to one of lower concentration. It occurs through the random movement of ions and molecules. Examples of diffusion are the exchange of oxygen and carbon dioxide between the pulmonary capillaries and alveoli and the tendency of sodium to move from the ECF compartment, where the sodium concentration is high, to the ICF, where its concentration is low. Osmosis occurs when two different solutions are separated by a membrane that is impermeable to the dissolved substances; fluid shifts through the membrane from the region of low solute concentration to the region of high solute concentration until the solutions are of equal concentration. Active transport implies that energy must be expended for the movement to occur against a concentration gradient. Movement of water and solutes occurring from an area of high hydrostatic pressure to an area of low hydrostatic pressure is filtration.

The nurse is caring for a patient in metabolic alkalosis. The patient has an NG tube to low intermittent suction for a diagnosis of bowel obstruction. What drug would the nurse expect to find on the medication orders? A) Cimetidine B) Maalox C) Potassium chloride elixir D) Furosemide

Ans: A Feedback: H2 receptor antagonists, such as cimetidine (Tagamet), reduce the production of gastric HCl, thereby decreasing the metabolic alkalosis associated with gastric suction. Maalox is an oral simethicone used to break up gas in the GI system and would be of no benefit in treating a patient in metabolic alkalosis. KCl would only be given if the patient were hypokalemic, which is not stated in the scenario. Furosemide (Lasix) would only be given if the patient were fluid overloaded, which is not stated in the scenario.

A patient has questioned the nurse's administration of IV normal saline, asking whether sterile water would be a more appropriate choice than "saltwater." Under what circumstances would the nurse administer electrolyte-free water intravenously? A) Never, because it rapidly enters red blood cells, causing them to rupture. B) When the patient is severely dehydrated resulting in neurologic signs and symptoms C) When the patient is in excess of calcium and/or magnesium ions D) When a patient's fluid volume deficit is due to acute or chronic renal failure

Ans: A Feedback: IV solutions contain dextrose or electrolytes mixed in various proportions with water. Pure, electrolyte-free water can never be administered by IV because it rapidly enters red blood cells and causes them to rupture.

You are the nurse evaluating a newly admitted patient's laboratory results, which include several values that are outside of reference ranges. Which of the following would cause the release of antidiuretic hormone (ADH)? A) Increased serum sodium B) Decreased serum potassium C) Decreased hemoglobin D) Increased platelets

Ans: A Feedback: Increased serum sodium causes increased thirst and the release of ADH by the posterior pituitary gland. When serum osmolality decreases and thirst and ADH secretions are suppressed, the kidney excretes more water to restore normal osmolality. Levels of potassium, hemoglobin, and platelets do not directly affect ADH release.

You are the nurse caring for a patient who is to receive IV daunorubicin, a chemotherapeutic agent. You start the infusion and check the insertion site as per protocol. During your most recent check, you note that the IV has infiltrated so you stop the infusion. What is your main concern with this infiltration? A) Extravasation of the medication B) Discomfort to the patient C) Blanching at the site D) Hypersensitivity reaction to the medication

Ans: A Feedback: Irritating medications, such as chemotherapeutic agents, can cause pain, burning, and redness at the site. Blistering, inflammation, and necrosis of tissues can occur. The extent of tissue damage is determined by the medication concentration, the quantity that extravasated, infusion site location, the tissue response, and the extravasation duration. Extravasation is the priority over the other listed consequences.

A teenage patient with a pilonidal cyst has been brought for care by her mother. The nurse who is contributing to the patients care knows that treatment will be chosen based on what risk? • Risk for infection • Risk for bowel incontinence • Risk for constipation • Risk for impaired tissue perfusion

Ans: A Feedback: Pilonidal cysts frequently develop into an abscess, necessitating surgical repair. These cysts do not contribute to bowel incontinence, constipation, or impaired tissue perfusion.

The nurse is providing care for a patient whose inflammatory bowel disease has necessitated hospital treatment. Which of the following would most likely be included in the patients medication regimen? • Anticholinergic medications 30 minutes before a meal • Antiemetics on a PRN basis • Vitamin B12 injections to prevent pernicious anemia • Beta adrenergic blockers to reduce bowel motility

Ans: A Feedback: The nurse administers anticholinergic medications 30 minutes before a meal as prescribed to decrease intestinal motility and administers analgesics as prescribed for pain. Antiemetics, vitamin B12 injections and beta blockers do not address the signs, symptoms, or etiology of inflammatory bowel disease.

A nurse is preparing to provide care for a patient whose exacerbation of ulcerative colitis has required hospital admission. During an exacerbation of this health problem, the nurse would anticipate that the patients stools will have what characteristics? • Watery with blood and mucus • Hard and black or tarry • Dry and streaked with blood • Loose with visible fatty streaks

Ans: A Feedback: The predominant symptoms of ulcerative colitis are diarrhea and abdominal pain. Stools may be bloody and contain mucus. Stools are not hard, dry, tarry, black or fatty in patients who have ulcerative colitis.

A burn patient is transitioning from the acute phase of the injury to the rehabilitation phase. The patient tells the nurse, "I can't wait to have surgery to reconstruct my face so I look normal again." What would be the nurse's best response? A) "That's something that you and your doctor will likely talk about after your scars mature." B) "That is something for you to talk to your doctor about because it's not a nursing responsibility." C) "I know this is really important to you, but you have to realize that no one can make you look like you used to." D) "Unfortunately, it's likely that you will have most of these scars for the rest of your life."

Ans: A Feedback: Burn reconstruction is a treatment option after all scars have matured and is discussed within the first few years after injury. Even though this is not a nursing responsibility, the nurse should still respond appropriately to the patient's query. It is true that the patient will not realistically look like he or she used to, but this does not instill hope.

A patient has been admitted to a burn intensive care unit with extensive full-thickness burns over 25% of the body. After ensuring cardiopulmonary stability, what would be the nurse's immediate, priority concern when planning this patient's care? A) Fluid status B) Risk of infection C) Nutritional status D) Psychosocial coping

Ans: A Feedback: During the early phase of burn care, the nurse is most concerned with fluid resuscitation, to correct large-volume fluid loss through the damaged skin. Infection control and early nutritional support are important, but fluid resuscitation is an immediate priority. Coping is a higher priority later in the recovery period.

A patient in the emergent/resuscitative phase of a burn injury has had blood work and arterial blood gases drawn. Upon analysis of the patient's laboratory studies, the nurse will expect the results to indicate what? A) Hyperkalemia, hyponatremia, elevated hematocrit, and metabolic acidosis B) Hypokalemia, hypernatremia, decreased hematocrit, and metabolic acidosis C) Hyperkalemia, hypernatremia, decreased hematocrit, and metabolic alkalosis D) Hypokalemia, hyponatremia, elevated hematocrit, and metabolic alkalosis

Ans: A Feedback: Fluid and electrolyte changes in the emergent/resuscitative phase of a burn injury include hyperkalemia related to the release of potassium into the extracellular fluid, hyponatremia from large amounts of sodium lost in trapped edema fluid, hemoconcentration that leads to an increased hematocrit, and loss of bicarbonate ions that results in metabolic acidosis.

A patient has sustained a severe burn injury and is thought to have an impaired intestinal mucosal barrier. Since this patient is considered at an increased risk for infection, what intervention will best assist in avoiding increased intestinal permeability and prevent early endotoxin translocation? A) Early enteral feeding B) Administration of prophylactic antibiotics C) Bowel cleansing procedures D) Administration of stool softeners

Ans: A Feedback: If the intestinal mucosa receives some type of protection against permeability change, infection could be avoided. Early enteral feeding is one step to help avoid this increased intestinal permeability and prevent early endotoxin translocation. Antibiotics are seldom prescribed prophylactically because of the risk of promoting resistant strains of bacteria. A bowel cleansing procedure would not be ordered for this patient. The administration of stool softeners would not assist in avoiding increased intestinal permeability and prevent early endotoxin translocation.

An emergency department nurse learns from the paramedics that they are transporting a patient who has suffered injury from a scald from a hot kettle. What variables will the nurse consider when determining the depth of burn? A) The causative agent B) The patient's preinjury health status C) The patient's prognosis for recovery D) The circumstances of the accident

Ans: A Feedback: The following factors are considered in determining the depth of a burn: how the injury occurred, causative agent (such as flame or scalding liquid), temperature of the burning agent, duration of contact with the agent, and thickness of the skin. The patient's preinjury status, circumstances of the accident, and prognosis for recovery are important, but are not considered when determining the depth of the burn.

A patient who is in the acute phase of recovery from a burn injury has yet to experience adequate pain control. What pain management strategy is most likely to meet this patient's needs? A) A patient-controlled analgesia (PCA) system B) Oral opioids supplemented by NSAIDs C) Distraction and relaxation techniques supplemented by NSAIDs D) A combination of benzodiazepines and topical anesthetics

Ans: A Feedback: The goal of treatment is to provide a long-acting analgesic that will provide even coverage for this long-term discomfort. It is helpful to use escalating doses when initiating the medication to reach the level of pain control that is acceptable to the patient. The use of patient-controlled analgesia (PCA) gives control to the patient and achieves this goal. Patients cannot normally achieve adequate pain control without the use of opioids, and parenteral administration is usually required.

29. When caring for a patient with increased ICP the nurse knows the importance of monitoring for possible secondary complications, including syndrome of inappropriate antidiuretic hormone (SIADH). What nursing interventions would the nurse most likely initiate if the patient developed SIADH? A) Fluid restriction B) Transfusion of platelets C) Transfusion of fresh frozen plasma (FFP) D) Electrolyte restriction

Ans: A Feedback: The nurse also assesses for complications of increased ICP, including diabetes insipidus, and SIADH. SIADH requires fluid restriction and monitoring of serum electrolyte levels. Transfusions are unnecessary.

You are the nurse caring for a 77-year-old male patient who has been involved in a motor vehicle accident. You and your colleague note that the patient's labs indicate minimally elevated serum creatinine levels, which your colleague dismisses. What can this increase in creatinine indicate in older adults? A) Substantially reduced renal function B) Acute kidney injury C) Decreased cardiac output D) Alterations in ratio of body fluids to muscle mass

Ans: A Feedback: Normal physiologic changes of aging, including reduced cardiac, renal, and respiratory function, and reserve and alterations in the ratio of body fluids to muscle mass, may alter the responses of elderly people to fluid and electrolyte changes and acid-base disturbances. Renal function declines with age, as do muscle mass and daily exogenous creatinine production. Therefore, high-normal and minimally elevated serum creatinine values may indicate substantially reduced renal function in older adults. Acute kidney injury is likely to cause a more significant increase in serum creatinine.

You are caring for a patient admitted with a diagnosis of acute kidney injury. When you review your patient's most recent laboratory reports, you note that the patient's magnesium levels are high. You should prioritize assessment for which of the following health problems? A) Diminished deep tendon reflexes B) Tachycardia C) Cool, clammy skin D) Acute flank pain

Ans: A Feedback: To gauge a patient's magnesium status, the nurse should check deep tendon reflexes. If the reflex is absent, this may indicate high serum magnesium. Tachycardia, flank pain, and cool, clammy skin are not typically associated with hypermagnesemia.

23. The nurse is caring for a patient whose recent health history includes an altered LOC. What should be the nurses first action when assessing this patient? A) Assessing the patients verbal response B) Assessing the patients ability to follow complex commands C) Assessing the patients judgment D) Assessing the patients response to pain

Ans: A Feedback: Assessment of the patient with an altered LOC often starts with assessing the verbal response through determining the patients orientation to time, person, and place. In most cases, this assessment will precede each of the other listed assessments, even though each may be indicated

17. A patient has developed diabetes insipidus after having increased ICP following head trauma. What nursing assessment best addresses this complication? A) Vigilant monitoring of fluid balance B) Continuous BP monitoring C) Serial arterial blood gases (ABGs) D) Monitoring of the patients airway for patency

Ans: A Feedback: Diabetes insipidus requires fluid and electrolyte replacement, along with the administration of vasopressin, to replace and slow the urine output. Because of these alterations in fluid balance, careful monitoring is necessary. None of the other listed assessments directly addresses the major manifestations of diabetes insipidus.

14. The nurse has created a plan of care for a patient who is at risk for increased ICP. The patients care plan should specify monitoring for what early sign of increased ICP? A) Disorientation and restlessness B) Decreased pulse and respirations C) Projectile vomiting D) Loss of corneal reflex

Ans: A Feedback: Early indicators of ICP include disorientation and restlessness. Later signs include decreased pulse and respirations, projectile vomiting, and loss of brain stem reflexes, such as the corneal reflex.

26. The nurse is caring for a patient who sustained a moderate head injury following a bicycle accident. The nurses most recent assessment reveals that the patients respiratory effort has increased. What is the nurses most appropriate response? A) Inform the care team and assess for further signs of possible increased ICP. B) Administer bronchodilators as ordered and monitor the patients LOC. C) Increase the patients bed height and reassess in 30 minutes. D) Administer a bolus of normal saline as ordered.

Ans: A Feedback: Increased respiratory effort can be suggestive of increasing ICP, and the care team should be promptly informed. A bolus of IV fluid will not address the problem. Repositioning the patient and administering bronchodilators are insufficient responses, even though these actions may later be ordered.

33. A patient is postoperative day 1 following intracranial surgery. The nurses assessment reveals that the patients LOC is slightly decreased compared with the day of surgery. What is the nurses best response to this assessment finding? A) Recognize that this may represent the peak of post-surgical cerebral edema. B) Alert the surgeon to the possibility of an intracranial hemorrhage. C) Understand that the surgery may have been unsuccessful. D) Recognize the need to refer the patient to the palliative care team.

Ans: A Feedback: Some degree of cerebral edema occurs after brain surgery; it tends to peak 24 to 36 hours after surgery, producing decreased responsiveness on the second postoperative day. As such, there is not necessarily any need to deem the surgery unsuccessful or to refer the patient to palliative care. A decrease in LOC is not evidence of an intracranial hemorrhage.

21. The nurse is caring for a patient with permanent neurologic impairments resulting from a traumatic head injury. When working with this patient and family, what mutual goal should be prioritized? A) Achieve as high a level of function as possible. B) Enhance the quantity of the patients life. C) Teach the family proper care of the patient. D) Provide community assistance.

Ans: A Feedback: The overarching goals of care are to achieve as high a level of function as possible and to enhance the quality of life for the patient with neurologic impairment and his or her family. This goal encompasses family and community participation.

31. A patient is recovering from intracranial surgery performed approximately 24 hours ago and is complaining of a headache that the patient rates at 8 on a 10-point pain scale. What nursing action is most appropriate? A) Administer morphine sulfate as ordered. B) Reposition the patient in a prone position. C) Apply a hot pack to the patients scalp. D) Implement distraction techniques.

Ans: A Feedback: The patient usually has a headache after a craniotomy as a result of stretching and irritation of nerves in the scalp during surgery. Morphine sulfate may also be used in the management of postoperative pain in patients who have undergone a craniotomy. Prone positioning is contraindicated due to the consequent increase in ICP. Distraction would likely be inadequate to reduce pain and a hot pack may cause vasodilation and increased pain.

26. A patient has been admitted to the hospital after diagnostic imaging revealed the presence of a gastric outlet obstruction (GOO). What is the nurse's priority intervention? Administration of antiemetics Insertion of an NG tube for decompression Infusion of hypotonic IV solution Administration of proton pump inhibitors as ordered

Ans: B Feedback: In treating the patient with gastric outlet obstruction, the first consideration is to insert an NG tube to decompress the stomach. This is a priority over fluid or medication administration.

A 15-year-old child is brought to the emergency department with symptoms of hyperglycemia and is subsequently diagnosed with diabetes. Based on the fact that the child's pancreatic beta cells are being destroyed, the patient would be diagnosed with what type of diabetes? A) Type 1 diabetes B) Type 2 diabetes C) Non-insulin-dependent diabetes D) Prediabetes

Ans: A Feedback: Beta cell destruction is the hallmark of type 1 diabetes. Non-insulin-dependent diabetes is synonymous with type 2 diabetes, which involves insulin resistance and impaired insulin secretion, but not beta cell destruction. Prediabetes is characterized by normal glucose metabolism, but a previous history of hyperglycemia, often during illness or pregnancy.

A patient presents to the clinic complaining of symptoms that suggest diabetes. What criteria would support checking blood levels for the diagnosis of diabetes? A) Fasting plasma glucose greater than or equal to 126 mg/dL B) Random plasma glucose greater than 150 mg/dL C) Fasting plasma glucose greater than 116 mg/dL on 2 separate occasions D) Random plasma glucose greater than 126 mg/dL

Ans: A Feedback: Criteria for the diagnosis of diabetes include symptoms of diabetes plus random plasma glucose greater than or equal to 200 mg/dL, or a fasting plasma glucose greater than or equal to 126 mg/dL.

A patient has been living with type 2 diabetes for several years, and the nurse realizes that the patient is likely to have minimal contact with the health care system. In order to ensure that the patient maintains adequate blood sugar control over the long term, the nurse should recommend which of the following? A) Participation in a support group for persons with diabetes B) Regular consultation of websites that address diabetes management C) Weekly telephone "check-ins" with an endocrinologist D) Participation in clinical trials relating to antihyperglycemics

Ans: A Feedback: Participation in support groups is encouraged for patients who have had diabetes for many years as well as for those who are newly diagnosed. This is more interactive and instructive than simply consulting websites. Weekly telephone contact with an endocrinologist is not realistic in most cases. Participation in research trials may or may not be beneficial and appropriate, depending on patients' circumstances.

A diabetes nurse educator is teaching a group of patients with type 1 diabetes about "sick day rules." What guideline applies to periods of illness in a diabetic patient? A) Do not eliminate insulin when nauseated and vomiting. B) Report elevated glucose levels greater than 150 mg/dL. C) Eat three substantial meals a day, if possible. D) Reduce food intake and insulin doses in times of illness.

Ans: A Feedback: The most important issue to teach patients with diabetes who become ill is not to eliminate insulin doses when nausea and vomiting occur. Rather, they should take their usual insulin or oral hypoglycemic agent dose, then attempt to consume frequent, small portions of carbohydrates. In general, blood sugar levels will rise but should be reported if they are greater than 300 mg/dL.

A diabetic educator is discussing "sick day rules" with a newly diagnosed type 1 diabetic. The educator is aware that the patient will require further teaching when the patient states what? A) "I will not take my insulin on the days when I am sick, but I will certainly check my blood sugar every 2 hours." B) "If I cannot eat a meal, I will eat a soft food such as soup, gelatin, or pudding six to eight times a day." C) "I will call the doctor if I am not able to keep liquids in my body due to vomiting or diarrhea." D) "I will call the doctor if my blood sugar is over 300 mg/dL or if I have ketones in my urine."

Ans: A Feedback: The nurse must explanation the "sick day rules" again to the patient who plans to stop taking insulin when sick. The nurse should emphasize that the patient should take insulin agents as usual and test one's blood sugar and urine ketones every 3 to 4 hours. In fact, insulin-requiring patients may need supplemental doses of regular insulin every 3 to 4 hours. The patient should report elevated glucose levels (greater than 300 mg/dL or as otherwise instructed) or urine ketones to the physician. If the patient is not able to eat normally, the patient should be instructed to substitute soft foods such a gelatin, soup, and pudding. If vomiting, diarrhea, or fever persists, the patient should have an intake of liquids every 30 to 60 minutes to prevent dehydration.

14. A patient was treated in the emergency department and critical care unit after ingesting bleach. What possible complication of the resulting gastritis should the nurse recognize? Esophageal or pyloric obstruction related to scarring Uncontrolled proliferation of H. pylori Gastric hyperacidity related to excessive gastrin secretion Chronic referred pain in the lower abdomen

Ans: A Feedback: A severe form of acute gastritis is caused by the ingestion of strong acid or alkali, which may cause the mucosa to become gangrenous or to perforate. Scarring can occur, resulting in pyloric stenosis (narrowing or tightening) or obstruction. Chronic referred pain to the lower abdomen is a symptom of peptic ulcer disease, but would not be an expected finding for a patient who has ingested a corrosive substance. Bacterial proliferation and hyperacidity would not occur.

16. A patient is one month postoperative following restrictive bariatric surgery. The patient tells the clinic nurse that he has been having "trouble swallowing" for the past few days. What recommendation should the nurse make? Eating more slowly and chewing food more thoroughly Taking an OTC antacid or drinking a glass of milk prior to each meal Chewing gum to cause relaxation of the lower esophageal sphincter Drinking at least 12 ounces of liquid with each meal

Ans: A Feedback: Dysphagia may be prevented by educating patients to eat slowly, to chew food thoroughly, and to avoid eating tough foods such as steak or dry chicken or doughy bread. After bariatric procedures, patients should normally not drink beverages with meals. Medications or chewing gum will not alleviate this problem.

12. A patient presents to the walk-in clinic complaining of vomiting and burning in her mid-epigastria. The nurse knows that in the process of confirming peptic ulcer disease, the physician is likely to order a diagnostic test to detect the presence of what? Infection with Helicobacter pylori Excessive stomach acid secretion An incompetent pyloric sphincter A metabolic acid-base imbalance

Ans: A Feedback: H. pylori infection may be determined by endoscopy and histologic examination of a tissue specimen obtained by biopsy, or a rapid urease test of the biopsy specimen. Excessive stomach acid secretion leads to gastritis; however, peptic ulcers are caused by colonization of the stomach by H. pylori. Sphincter dysfunction and acidbase imbalances do not cause peptic ulcer disease.

2. A patient comes to the clinic complaining of pain in the epigastric region. What assessment question during the health interview would most help the nurse determine if the patient has a peptic ulcer? "Does your pain resolve when you have something to eat?" "Do over-the-counter pain medications help your pain?" "Does your pain get worse if you get up and do some exercise?" "Do you find that your pain is worse when you need to have a bowel movement?"

Ans: A Feedback: Pain relief after eating is associated with duodenal ulcers. The pain of peptic ulcers is generally unrelated to activity or bowel function and may or may not respond to analgesics.

40. A patient has come to the clinic complaining of pain just above her umbilicus. When assessing the patient, the nurse notes Sister Mary Joseph's nodules. The nurse should refer the patient to the primary care provider to be assessed for what health problem? A GI malignancy Dumping syndrome Peptic ulcer disease Esophageal/gastric obstruction

Ans: A Feedback: Palpable nodules around the umbilicus, called Sister Mary Joseph's nodules, are a sign of a GI malignancy, usually a gastric cancer. This would not be a sign of dumping syndrome, peptic ulcer disease, or esophageal/gastric obstruction.

38. A patient with gastritis required hospital treatment for an exacerbation of symptoms and receives a subsequent diagnosis of pernicious anemia due to malabsorption. When planning the patient's continuing care in the home setting, what assessment question is most relevant? "Does anyone in your family have experience at giving injections?" "Are you going to be anywhere with strong sunlight in the next few months?" "Are you aware of your blood type?" "Do any of your family members have training in first aid?"

Ans: A Feedback: Patients with malabsorption of vitamin B12 need information about lifelong vitamin B12 injections; the nurse may instruct a family member or caregiver how to administer the injections or make arrangements for the patient to receive the injections from a health care provider. Questions addressing sun exposure, blood type and first aid are not directly relevant.

28. Diagnostic imaging and physical assessment have revealed that a patient with peptic ulcer disease has suffered a perforated ulcer. The nurse recognizes that emergency interventions must be performed as soon as possible in order to prevent the development of what complication? Peritonitis Gastritis Gastroesophageal reflux Acute pancreatitis

Ans: A Feedback: Perforation is the erosion of the ulcer through the gastric serosa into the peritoneal cavity without warning. Chemical peritonitis develops within a few hours of perforation and is followed by bacterial peritonitis. Gastritis, reflux, and pancreatitis are not acute complications of a perforated ulcer.

8. A nurse in the postanesthesia care unit admits a patient following resection of a gastric tumor. Following immediate recovery, the patient should be placed in which position to facilitate patient comfort and gastric emptying? Fowler's Supine Left lateral Left Sim's

Ans: A Feedback: Positioning the patient in a Fowler's position postoperatively promotes comfort and facilitates emptying of the stomach following gastric surgery. Any position that involves lying down delays stomach emptying and is not recommended for this type of patient. Supine positioning and the left lateral (left Sim's) position do not achieve this goal.

3. A patient with a diagnosis of peptic ulcer disease has just been prescribed omeprazole (Prilosec). How should the nurse best describe this medication's therapeutic action? "This medication will reduce the amount of acid secreted in your stomach." "This medication will make the lining of your stomach more resistant to damage." "This medication will specifically address the pain that accompanies peptic ulcer disease." "This medication will help your stomach lining to repair itself."

Ans: A Feedback: Proton pump inhibitors like Prilosec inhibit the synthesis of stomach acid. PPIs do not increase the durability of the stomach lining, relieve pain, or stimulate tissue repair.

29. A nurse is performing the admission assessment of a patient whose high body mass index (BMI) corresponds to class III obesity. In order to ensure empathic and patient-centered care, the nurse should do which of the following? Examine one's own attitudes towards obesity in general and the patient in particular. Dialogue with the patient about the lifestyle and psychosocial factors that resulted in obesity. Describe one's own struggles with weight gain and weight loss to the patient. Elicit the patient's short-term and long-term goals for weight loss.

Ans: A Feedback: Studies suggest that health care providers, including nurses, harbor negative attitudes towards obese patients. Nurses have a responsibility to examine these attitudes and change them accordingly. This is foundational to all other areas of assessing this patient.

11. A patient who experienced an upper GI bleed due to gastritis has had the bleeding controlled and the patient's condition is now stable. For the next several hours, the nurse caring for this patient should assess for what signs and symptoms of recurrence? Tachycardia, hypotension, and tachypnea Tarry, foul-smelling stools Diaphoresis and sudden onset of abdominal pain Sudden thirst, unrelieved by oral fluid administration

Ans: A Feedback: Tachycardia, hypotension, and tachypnea are signs of recurrent bleeding. Patients who have had one GI bleed are at risk for recurrence. Tarry stools are expected short-term findings after a hemorrhage. Hemorrhage is not normally associated with sudden thirst or diaphoresis.

35. A patient has received a diagnosis of gastric cancer and is awaiting a surgical date. During the preoperative period, the patient should adopt what dietary guidelines? Eat small, frequent meals with high calorie and vitamin content. Eat frequent meals with an equal balance of fat, carbohydrates, and protein. Eat frequent, low-fat meals with high protein content. Try to maintain the pre-diagnosis pattern of eating.

Ans: A Feedback: The nurse encourages the patient to eat small, frequent portions of nonirritating foods to decrease gastric irritation. Food supplements should be high in calories, as well as vitamins A and C and iron, to enhance tissue repair.

A patient who has undergone a lower limb amputation is preparing to be discharged home. What outcome is necessary prior to discharge? A) Patient can demonstrate safe use of assistive devices. B) Patient has a healed, nontender, nonadherent scar. C) Patient can perform activities of daily living independently. D) Patientis free of pain.

Ans: A Feedback: A patient should be able to use assistive devices appropriately and safely prior to discharge. Scar formation will not be complete at the time of hospital discharge. It is anticipated that the patient will require some assistance with ADLs postdischarge. Pain should be well managed, but may or may not be wholly absent.

A patient was fitted with an arm cast after fracturing her humerus. Twelve hours after the application of the cast, the patient tells the nurse that her arm hurts. Analgesics do not relieve the pain. What would be the most appropriate nursing action? A) Prepare the patient for opening or bivalving of the cast. B) Obtain an order for a different analgesic. C) Encourage the patient to wiggle and move the fingers. D) Petal the edges of the patient's cast.

Ans: A Feedback: Acute compartment syndrome involves a sudden and severe decrease in blood flow to the tissues distal to an area of injury that results in ischemic necrosis if prompt, decisive intervention does not occur. Removing or bivalving the cast is necessary to relieve pressure. Ordering different analgesics does not address the underlying problem. Encouraging the patient to move the fingers or perform range-of-motion exercises will not treat or prevent compartment syndrome. Petaling the edges of a cast with tape prevents abrasions and skin breakdown, not compartment syndrome.

A rehabilitation nurse is working with a patient who has had a below-the-knee amputation. The nurse knows the importance of the patient's active participation in self-care. In order to determine the patient's ability to be an active participant in self-care, the nurse should prioritize assessment of what variable? A) The patient's attitude B) The patient's learning style C) The patient's nutritional status D) The patient's presurgical level of function

Ans: A Feedback: Amputation of an extremity affects the patient's ability to provide adequate self-care. The patient is encouraged to be an active participant in self-care. The patient and the nurse need to maintain positive attitudes and to minimize fatigue and frustration during the learning process. Balanced nutrition and the patient's learning style are important variables in the rehabilitation process but the patient's attitude is among the most salient variables. The patient's presurgical level of function may or may not affect participation in rehabilitation.

The surgical nurse is admitting a patient from postanesthetic recovery following the patient's below-the-knee amputation. The nurse recognizes the patient's high risk for postoperative hemorrhage and should keep which of the following at the bedside? A) A tourniquet B) A syringe preloaded with vitamin K C) A unit of packed red blood cells, placed on ice D) A dose of protamine sulfate

Ans: A Feedback: Immediate postoperative bleeding may develop slowly or may take the form of massive hemorrhage resulting from a loosened suture. A large tourniquet should be in plain sight at the patient's bedside so that, if severe bleeding occurs, it can be applied to the residual limb to control the hemorrhage. PRBCs cannot be kept at the bedside. Vitamin K and protamine sulfate are antidotes to warfarin and heparin, but are not administered to treat active postsurgical bleeding.

A 25-year-old man is involved in a motorcycle accident and injures his arm. The physician diagnoses the man with an intra-articular fracture and splints the injury. The nurse implements the teaching plan developed for this patient. What sequela of intra-articular fractures should the nurse describe regarding this patient? A) Post-traumatic arthritis B) Fat embolism syndrome (FES) C) Osteomyelitis D) Compartment syndrome

Ans: A Feedback: Intra-articular fractures often lead to post-traumatic arthritis. Research does not indicate a correlation between intra-articular fractures and FES, osteomyelitis, or compartment syndrome.

The patient scheduled for a Syme amputation is concerned about the ability to eventually stand on the amputated extremity. How should the nurse best respond to the patient's concern? A) "You will eventually be able to withstand full weight-bearing after the amputation." B) "You will have minimal weight-bearing on this extremity but you'll be taught how to use an assistive device." C) "You likely will not be able to use this extremity but you will receive teaching on use of a wheelchair." D) "You will be fitted for a prosthesis which may or may not allow you to walk."

Ans: A Feedback: Syme amputation (modified ankle disarticulation amputation) is performed most frequently for extensive foot trauma and produces a painless, durable extremity end that can withstand full weight-bearing. Therefore, each of the other teaching statements is incorrect.

A nurse is caring for a patient who had a right below-the-knee amputation (BKA). The nurse recognizes the importance of implementing measures that focus on preventing flexion contracture of the hip and maintaining proper positioning. Which of the following measures will best achieve these goals? A) Encouraging the patient to turn from side to side and to assume a prone position B) Initiating ROM exercises of the hip and knee 10 to 12 weeks after the amputation C) Minimizing movement of the flexor muscles of the hip D) Encouraging the patient to sit in a chair for at least 8 hours a day

Ans: A Feedback: The nurse encourages the patient to turn from side to side and to assume a prone position, if possible, to stretch the flexor muscles and to prevent flexion contracture of the hip. Postoperative ROM exercises are started early, because contracture deformities develop rapidly. ROM exercises include hip and knee exercises for patients with BKAs. The nurse also discourages sitting for prolonged periods of time.

A patient has returned to the postsurgical unit from the PACU after an above-the-knee amputation of the right leg. Results of the nurse's initial postsurgical assessment were unremarkable but the patient has called out. The nurse enters the room and observes copious quantities of blood at the surgical site. What should be the nurse's initial action? A) Apply a tourniquet. B) Elevate the residual limb. C) Apply sterile gauze. D) Call the surgeon.

Ans: A Feedback: The nurse should apply a tourniquet in the event of postsurgical hemorrhage. Elevating the limb and applying sterile gauze are likely insufficient to stop the hemorrhage. The nurse should attempt to control the immediate bleeding before contacting the surgeon.

A nurse is writing a care plan for a patient admitted to the emergency department (ED) with an open fracture. The nurse will assign priority to what nursing diagnosis for a patient with an open fracture of the radius? A) Risk for Infection B) Risk for Ineffective Role Performance C) Risk for Perioperative Positioning Injury D) Risk for Powerlessness

Ans: A Feedback: The patient has a significant risk for osteomyelitis and tetanus due to the fact that the fracture is open. Powerlessness and ineffective role performance are psychosocial diagnoses that may or may not apply, and which would be superseded by immediate physiologic threats such as infection. Surgical positioning injury is not plausible, since surgery is not likely indicated.

An emergency department nurse is assessing a 17-year-old soccer player who presented with a knee injury. The patient's description of the injury indicates that his knee was struck medially while his foot was on the ground. The nurse knows that the patient likely has experienced what injury? A) Lateral collateral ligament injury B) Medial collateral ligament injury C) Anterior cruciate ligament injury D) Posterior cruciate ligament injury

Ans: A Feedback: When the knee is struck medially, damage may occur to the lateral collateral ligament. If the knee is struck laterally, damage may occur to the medial collateral ligament. The ACL and PCL are not typically injured in this way.

30. The nurse is admitting a patient to the unit who is scheduled for removal of an intracranial mass. What diagnostic procedures might be included in this patients admission orders? Select all that apply. A) Transcranial Doppler flow study B) Cerebral angiography C) MRI D) Cranial radiography E) Electromyelography (EMG)

Ans: A, B, C Feedback: Preoperative diagnostic procedures may include a CT scan to demonstrate the lesion and show the degree of surrounding brain edema, the ventricular size, and the displacement. An MRI scan provides information similar to that of a CT scan with improved tissue contrast, resolution, and anatomic definition. Cerebral angiography may be used to study a tumors blood supply or to obtain information about vascular lesions. Transcranial Doppler flow studies are used to evaluate the blood flow within intracranial blood vessels. Regular x-rays of the skull would not be diagnostic for an intracranial mass. An EMG would not be ordered prior to intracranial surgery to remove a mass.

39. A nurse is presenting a class at a bariatric clinic about the different types of surgical procedures offered by the clinic. When describing the implications of different types of surgeries, the nurse should address which of the following topics? Select all that apply. Specific lifestyle changes associated with each procedure Implications of each procedure for eating habits Effects of different surgeries on bowel function Effects of various bariatric surgeries on fertility Effects of different surgeries on safety of future immunizations

Ans: A, B, C Feedback: Different bariatric surgical procedures entail different lifestyle modifications; patients must be well informed about the specific lifestyle changes, eating habits, and bowel habits that may result from a particular procedure. Bariatric surgeries do not influence the future use of immunizations or fertility, though pregnancy should be avoided for 18 months after bariatric surgery.

40. An adult patient has sought care for the treatment of headaches that have become increasingly severe and frequent over the past several months. Which of the following questions addresses potential etiological factors? Select all that apply? A) Are you exposed to any toxins or chemicals at work? B) How would you describe your ability to cope with stress? C) What medications are you currently taking? D) When was the last time you were hospitalized? E) Does anyone else in your family struggle with headaches?

Ans: A, B, C, E Feedback: Headaches are multifactorial, and may involve medications, exposure to toxins, family history, and stress. Hospitalization is an unlikely contributor to headaches.

A nurse is planning the care of an older adult patient who will soon be discharged home after treatment for a fractured hip. In an effort to prevent future fractures, the nurse should encourage which of the following? Select all that apply. A) Regular bone density testing B) A high-calcium diet C) Use of falls prevention precautions D) Use of corticosteroids as ordered E) Weight-bearing exercise

Ans: A, B, C, E Feedback: Health promotion measures after an older adult's hip fracture include weight-bearing exercise, promotion of a healthy diet, falls prevention, and bone density testing. Corticosteroids have the potential to reduce bone density and increase the risk for fractures.

An older adult patient experienced a fall and required treatment for a fractured hip on the orthopedic unit. Which of the following are contributory factors to the incidence of falls and fractured hips among the older adult population? Select all that apply. A) Loss of visual acuity B) Adverse medication effects C) Slowed reflexes D) Hearing loss E) Muscle weakness

Ans: A, B, C, E Feedback: Older adults are generally vulnerable to falls and have a high incidence of hip fracture. Weak quadriceps muscles, medication effects, vision loss, and slowed reflexes are among the factors that contribute to the incidence of falls. Decreased hearing is not noted to contribute to the incidence of falls.

A nurse is caring for a patient who has been admitted to the hospital with diverticulitis. Which of the following would be appropriate nursing diagnoses for this patient? Select all that apply. A. Acute Pain Related to Increased Peristalsis and GI Inflammation B. Activity Intolerance Related to Generalized Weakness C. Bowel Incontinence Related to Increased Intestinal Peristalsis D. Deficient Fluid Volume Related to Anorexia, Nausea, and Diarrhea E. Impaired Urinary Elimination Related to GI Pressure on the Bladder

Ans: A, B, D Feedback: Patients with diverticulitis are likely to experience pain and decreased activity levels, and are at risk of fluid volume deficit. The patient is unlikely to experience fecal incontinence and urinary function is not directly influenced.

24. The nurse caring for a patient in a persistent vegetative state is regularly assessing for potential complications. Complications of neurologic dysfunction for which the nurse should assess include which of the following? Select all that apply. A) Contractures B) Hemorrhage C) Pressure ulcers D) Venous thromboembolism E) Pneumonia

Ans: A, C, D, E Feedback: Based on the assessment data, potential complications may include respiratory distress or failure, pneumonia, aspiration, pressure ulcer, deep vein thrombosis (DVT), and contractures. The pathophysiology of decreased LOC does not normally create a heightened risk for hemorrhage.

You are doing discharge teaching with a patient who has hypophosphatemia during his time in hospital. The patient has a diet ordered that is high in phosphate. What foods would you teach this patient to include in his diet? Select all that apply. A) Milk B) Beef C) Poultry D) Green vegetables E) Liver

Ans: A, C, E Feedback: If the patient experiences mild hypophosphatemia, foods such as milk and milk products, organ meats, nuts, fish, poultry, and whole grains should be encouraged.

A nurse is teaching a patient with a partial-thickness wound how to wear his elastic pressure garment. How would the nurse instruct the patient to wear this garment? A) 4 to 6 hours a day for 6 months B) During waking hours for 2 to 3 months after the injury C) Continuously D) At night while sleeping for a year after the injury

Ans: C Feedback: Elastic pressure garments are worn continuously (i.e., 23 hours a day).

A patient is in the acute phase of a burn injury. One of the nursing diagnoses in the plan of care is Ineffective Coping Related to Trauma of Burn Injury. What interventions appropriately address this diagnosis? Select all that apply. A) Promote truthful communication. B) Avoid asking the patient to make decisions. C) Teach the patient coping strategies. D) Administer benzodiazepines as ordered. E) Provide positive reinforcement.

Ans: A, C, E Feedback: The nurse can assist the patient to develop effective coping strategies by setting specific expectations for behavior, promoting truthful communication to build trust, helping the patient practice appropriate strategies, and giving positive reinforcement when appropriate. The patient may benefit from being able to make decisions regarding his or her care. Benzodiazepines may be needed for short-term management of anxiety, but they are not used to enhance coping.

A gerontologic nurse is teaching students about the high incidence and prevalence of dehydration in older adults. What factors contribute to this phenomenon? Select all that apply. A) Decreased kidney mass B) Increased conservation of sodium C) Increased total body water D) Decreased renal blood flow E) Decreased excretion of potassium

Ans: A, D, E Feedback: Dehydration in the elderly is common as a result of decreased kidney mass, decreased glomerular filtration rate, decreased renal blood flow, decreased ability to concentrate urine, inability to conserve sodium, decreased excretion of potassium, and a decrease of total body water.

The nurse is caring for a patient who is undergoing diagnostic testing for suspected malabsorption. When taking this patients health history and performing the physical assessment, the nurse should recognize what finding as most consistent with this diagnosis? • Recurrent constipation coupled with weight loss • Foul-smelling diarrhea that contains fat • Fever accompanied by a rigid, tender abdomen • Bloody bowel movements accompanied by fecal incontinence

Ans: B Feedback: The hallmarks of malabsorption syndrome from any cause are diarrhea or frequent, loose, bulky, foulsmelling stools that have increased fat content and are often grayish (steatorrhea). Constipation and bloody bowel movements are not suggestive of malabsorption syndromes. Fever and a tender, rigid abdomen are associated with peritonitis.

A patient with a diagnosis of colon cancer is 2 days postoperative following bowel resection and anastomosis. The nurse has planned the patients care in the knowledge of potential complications. What assessment should the nurse prioritize? • Close monitoring of temperature • Frequent abdominal auscultation • Assessment of hemoglobin, hematocrit, and red blood cell levels • Palpation of peripheral pulses and leg girth

Ans: B Feedback: After bowel surgery, it is important to frequently assess the abdomen, including bowel sounds and abdominal girth, to detect bowel obstruction. The resumption of bowel motility is a priority over each of the other listed assessments, even though each should be performed by the nurse.

A nurse caring for a patient with colorectal cancer is preparing the patient for upcoming surgery. The nurse administers cephalexin (Keflex) to the patient and explains what rationale? • To treat any undiagnosed infections • To reduce intestinal bacteria levels • To reduce bowel motility • To reduce abdominal distention postoperatively

Ans: B Feedback: Antibiotics such a kanamycin (Kantrex), neomycin (Mycifradin), and cephalexin (Keflex) are administered orally the day before surgery to reduce intestinal bacterial. Preoperative antibiotics are not given to treat undiagnosed infections, reduce motility, or prevent abdominal distention.

A nurse is providing care for a patient whose recent colostomy has contributed to a nursing diagnosis of Disturbed Body Image Related to Colostomy. What intervention best addresses this diagnosis? A. Encourage the patient to conduct online research into colostomies. B. Engage the patient in the care of the ostomy to the extent that the patient is willing. C. Emphasize the fact that the colostomy was needed to alleviate a much more serious health problem. D. Emphasize the fact that the colostomy is temporary measure and is not permanent.

Ans: B Feedback: For some patients, becoming involved in the care of the ostomy helps to normalize it and enhance familiarity. Emphasizing the benefits of the intervention is unlikely to improve the patients body image, since the benefits are likely already known. Online research is not likely to enhance the patients body image and some ostomies are permanent.

A patient has had an ileostomy created for the treatment of irritable bowel disease and the patient is now preparing for discharge. What should the patient be taught about changing this device in the home setting? • Apply antibiotic ointment as ordered after cleaning the stoma. • Apply a skin barrier to the peristomal skin prior to applying the pouch. • Dispose of the clamp with each bag change. • Cleanse the area surrounding the stoma with alcohol or chlorhexidine

Ans: B Feedback: Guidelines for changing an ileostomy appliance are as follows. Skin should be washed with soap and water, and dried. A skin barrier should be applied to the peristomal skin prior to applying the pouch. Clamps are supplied one per box and should be reused with each bag change. Topical antibiotics are not utilized, but an antifungal spray or powder may be used.

During a patients scheduled home visit, an older adult patient has stated to the community health nurse that she has been experiencing hemorrhoids of increasing severity in recent months. The nurse should recommend which of the following? • Regular application of an OTC antibiotic ointment • Increased fluid and fiber intake • Daily use of OTC glycerin suppositories • Use of an NSAID to reduce inflammation

Ans: B Feedback: Hemorrhoid symptoms and discomfort can be relieved by good personal hygiene and by avoiding excessive straining during defecation. A high-residue diet that contains fruit and bran along with an increased fluid intake may be all the treatment that is necessary to promote the passage of soft, bulky stools to prevent straining. Antibiotics, regular use of suppositories, and NSAIDs are not recommended, as they do not address the etiology of the health problem.

A nursing instructor is discussing hemorrhoids with the nursing class. Which patients would the nursing instructor identify as most likely to develop hemorrhoids? • A 45-year-old teacher who stands for 6 hours per day • A pregnant woman at 28 weeks gestation • A 37-year-old construction worker who does heavy lifting • A 60-year-old professional who is under stress

Ans: B Feedback: Hemorrhoids commonly affect 50% of patients after the age of 50. Pregnancy may initiate hemorrhoids or aggravate existing ones. This is due to increased constipation during pregnancy. The significance of pregnancy is greater than that of standing, lifting, or stress in the development of hemorrhoids.

A nurse caring for a patient with a newly created ileostomy assesses the patient and notes that the patient has had not ostomy output for the past 12 hours. The patient also complains of worsening nausea. What is the nurses priority action? • Facilitate a referral to the wound-ostomy-continence (WOC) nurse. • Report signs and symptoms of obstruction to the physician. • Encourage the patient to mobilize in order to enhance motility. • Contact the physician and obtain a swab of the stoma for culture.

Ans: B Feedback: It is important to report nausea and abdominal distention, which may indicate intestinal obstruction. This requires prompt medical intervention. Referral to the WOC nurse is not an appropriate short-term response, since medical treatment is necessary. Physical mobility will not normally resolve an obstruction. There is no need to collect a culture from the stoma, because infection is unrelated to this problem.

A nurse is caring for a patient with constipation whose primary care provider has recommended senna (Senokot) for the management of this condition. The nurse should provide which of the following education points? • Limit your fluid intake temporarily so you dont get diarrhea. • Avoid taking the drug on a long-term basis. • Make sure to take a multivitamin with each dose. • Take this on an empty stomach to ensure maximum effect.

Ans: B Feedback: Laxatives should not be taken on an ongoing basis in order to reduce the risk of dependence. Fluid should be increased, not limited, and there is no need to take each dose with a multivitamin. Senna does not need to be taken on an empty stomach.

The nurse caring for a patient post colon resection is assessing the patient on the second postoperative day. The nasogastric tube (NG) remains patent and continues at low intermittent wall suction. The IV is patent and infusing at 125 mL/hr. The patient reports pain at the incision site rated at a 3 on a 0-to-10 rating scale. During your initial shift assessment, the patient complains of cramps in her legs and a tingling sensation in her feet. Your assessment indicates decreased deep tendon reflexes (DTRs) and you suspect the patient has hypokalemia. What other sign or symptom would you expect this patient to exhibit? A) Diarrhea B) Dilute urine C) Increased muscle tone D) Joint pain

Ans: B Feedback: Manifestations of hypokalemia include fatigue, anorexia, nausea, vomiting, muscle weakness, leg cramps, decreased bowel motility, paresthesias (numbness and tingling), and dysrhythmias. If prolonged, hypokalemia can lead to an inability of the kidneys to concentrate urine, causing dilute urine (resulting in polyuria, nocturia) and excessive thirst. Potassium depletion suppresses the release of insulin and results in glucose intolerance. Decreased muscle strength and DTRs can be found on physical assessment. You would expect decreased, not increased, muscle strength with hypokalemia. The patient would not have diarrhea following bowel surgery, and increased bowel motility is inconsistent with hypokalemia.

A patient admitted with inflammatory bowel disease asks the nurse for help with menu selections. What menu selection is most likely the best choice for this patient? • Spinach • Tofu • Multigrain bagel • Blueberries

Ans: B Feedback: Nutritional management of inflammatory bowel disease requires ingestion of a diet that is bland, lowresidue, high-protein, and high-vitamin. Tofu meets each of the criteria. Spinach, multigrain bagels, and blueberries are not low-residue.

A nurse at an outpatient surgery center is caring for a patient who had a hemorrhoidectomy. What discharge education topics should the nurse address with this patient? • The appropriate use of antibiotics to prevent postoperative infection • The correct procedure for taking a sitz bath • The need to eat a low-residue, low-fat diet for the next 2 weeks • The correct technique for keeping the perianal region clean without the use of water

Ans: B Feedback: Sitz baths are usually indicated after perianal surgery. A low-residue, low-fat diet is not necessary and water is used to keep the region clean. Postoperative antibiotics are not normally prescribed.

You are the surgical nurse caring for a 65-year-old female patient who is postoperative day 1 following a thyroidectomy. During your shift assessment, the patient complains of tingling in her lips and fingers. She tells you that she has an intermittent spasm in her wrist and hand and she exhibits increased muscle tone. What electrolyte imbalance should you first suspect? A) Hypophosphatemia B) Hypocalcemia C) Hypermagnesemia D) Hyperkalemia

Ans: B Feedback: Tetany is the most characteristic manifestation of hypocalcemia and hypomagnesemia. Sensations of tingling may occur in the tips of the fingers, around the mouth, and, less commonly, in the feet. Hypophosphatemia creates central nervous dysfunction, resulting in seizures and coma. Hypermagnesemia creates hypoactive reflexes and somnolence. Signs of hyperkalemia include paresthesias and anxiety.

A medical nurse educator is reviewing a patient's recent episode of metabolic acidosis with members of the nursing staff. What should the educator describe about the role of the kidneys in metabolic acidosis? A) The kidneys retain hydrogen ions and excrete bicarbonate ions to help restore balance. B) The kidneys excrete hydrogen ions and conserve bicarbonate ions to help restore balance. C) The kidneys react rapidly to compensate for imbalances in the body. D) The kidneys regulate the bicarbonate level in the intracellular fluid.

Ans: B Feedback: The kidneys regulate the bicarbonate level in the ECF; they can regenerate bicarbonate ions as well as reabsorb them from the renal tubular cells. In respiratory acidosis and most cases of metabolic acidosis, the kidneys excrete hydrogen ions and conserve bicarbonate ions to help restore balance. In respiratory and metabolic alkalosis, the kidneys retain hydrogen ions and excrete bicarbonate ions to help restore balance. The kidneys obviously cannot compensate for the metabolic acidosis created by renal failure. Renal compensation for imbalances is relatively slow (a matter of hours or days).

A patient with a longstanding diagnosis of generalized anxiety disorder presents to the emergency room. The triage nurse notes upon assessment that the patient is hyperventilating. The triage nurse is aware that hyperventilation is the most common cause of which acid-base imbalance? A) Respiratory acidosis B) Respiratory alkalosis C) Increased PaCO2 D) CNS disturbances

Ans: B Feedback: The most common cause of acute respiratory alkalosis is hyperventilation. Extreme anxiety can lead to hyperventilation. Acute respiratory acidosis occurs in emergency situations, such as pulmonary edema, and is exhibited by hypoventilation and decreased PaCO2. CNS disturbances are found in extreme hyponatremia and fluid overload.

You are caring for a patient with a diagnosis of pancreatitis. The patient was admitted from a homeless shelter and is a vague historian. The patient appears malnourished and on day 3 of the patient's admission total parenteral nutrition (TPN) has been started. Why would you know to start the infusion of TPN slowly? A) Patients receiving TPN are at risk for hypercalcemia if calories are started too rapidly. B) Malnourished patients receiving parenteral nutrition are at risk for hypophosphatemia if calories are started too aggressively. C) Malnourished patients who receive fluids too rapidly are at risk for hypernatremia. D) Patients receiving TPN need a slow initiation of treatment in order to allow digestive enzymes to accumulate

Ans: B Feedback: The nurse identifies patients who are at risk for hypophosphatemia and monitors them. Because malnourished patients receiving parenteral nutrition are at risk when calories are introduced too aggressively, preventive measures involve gradually introducing the solution to avoid rapid shifts of phosphorus into the cells. Patients receiving TPN are not at risk for hypercalcemia or hypernatremia if calories or fluids are started to rapidly. Digestive enzymes are not a relevant consideration.

An older adult who resides in an assisted living facility has sought care from the nurse because of recurrent episodes of constipation. Which of the following actions should the nurse first perform? A. Encourage the patient to take stool softener daily. B. Assess the patients food and fluid intake. C. Assess the patients surgical history. D. Encourage the patient to take fiber supplements.

Ans: B Feedback: The nurse should follow the nursing process and perform an assessment prior to interventions. The patients food and fluid intake is more likely to affect bowel function than surgery.

A 16-year-old presents at the emergency department complaining of right lower quadrant pain and is subsequently diagnosed with appendicitis. When planning this patients nursing care, the nurse should prioritize what nursing diagnosis? A. Imbalanced Nutrition: Less Than Body Requirements Related to Decreased Oral Intake B. Risk for Infection Related to Possible Rupture of Appendix C. Constipation Related to Decreased Bowel Motility and Decreased Fluid Intake D. Chronic Pain Related to Appendicitis

Ans: B Feedback: The patient with a diagnosis of appendicitis has an acute risk of infection related to the possibility of rupture. This immediate physiologic risk is a priority over nutrition and constipation, though each of these concerns should be addressed by the nurse. The pain associated with appendicitis is acute, not chronic.

A patient admitted with acute diverticulitis has experienced a sudden increase in temperature and complains of a sudden onset of exquisite abdominal tenderness. The nurses rapid assessment reveals that the patients abdomen is uncharacteristically rigid on palpation. What is the nurses best response? A. Administer a Fleet enema as ordered and remain with the patient. B. Contact the primary care provider promptly and report these signs of perforation. C. Position the patient supine and insert an NG tube. D. Page the primary care provider and report that the patient may be obstructed.

Ans: B Feedback: The patients change in status is suggestive of perforation, which is a surgical emergency. Obstruction does not have this presentation involving fever and abdominal rigidity. An enema would be strongly contraindicated. An order is needed for NG insertion and repositioning is not a priority.

The physician has ordered a peripheral IV to be inserted before the patient goes for computed tomography. What should the nurse do when selecting a site on the hand or arm for insertion of an IV catheter? A) Choose a hairless site if available. B) Consider potential effects on the patient's mobility when selecting a site. C) Have the patient briefly hold his arm over his head before insertion. D) Leave the tourniquet on for at least 3 minutes.

Ans: B Feedback: Ideally, both arms and hands are carefully inspected before choosing a specific venipuncture site that does not interfere with mobility. Instruct the patient to hold his arm in a dependent position to increase blood flow. Never leave a tourniquet in place longer than 2 minutes. The site does not necessarily need to be devoid of hair.

The community health nurse is performing a home visit to an 84-year-old woman recovering from hip surgery. The nurse notes that the woman seems uncharacteristically confused and has dry mucous membranes. When asked about her fluid intake, the patient states, "I stop drinking water early in the day because it is just too difficult to get up during the night to go to the bathroom." What would be the nurse's best response? A) "I will need to have your medications adjusted so you will need to be readmitted to the hospital for a complete workup." B) "Limiting your fluids can create imbalances in your body that can result in confusion. Maybe we need to adjust the timing of your fluids." C) "It is normal to be a little confused following surgery, and it is safe not to urinate at night." D) "If you build up too much urine in your bladder, it can cause you to get confused, especially when your body is under stress. "

Ans: B Feedback: In elderly patients, the clinical manifestations of fluid and electrolyte disturbances may be subtle or atypical. For example, fluid deficit may cause confusion or cognitive impairment in the elderly person. There is no mention of medications in the stem of the question or any specific evidence given for the need for readmission to the hospital. Confusion is never normal, common, or expected in the elderly. Urinary retention does normally cause confusion.

A nurse in the neurologic ICU has orders to infuse a hypertonic solution into a patient with increased intracranial pressure. This solution will increase the number of dissolved particles in the patient's blood, creating pressure for fluids in the tissues to shift into the capillaries and increase the blood volume. This process is best described as which of the following? A) Hydrostatic pressure B) Osmosis and osmolality C) Diffusion D) Active transport

Ans: B Feedback: Osmosis is the movement of fluid from a region of low solute concentration to a region of high solute concentration across a semipermeable membrane. Hydrostatic pressure refers to changes in water or volume related to water pressure. Diffusion is the movement of solutes from an area of greater concentration to lesser concentration; the solutes in an intact vascular system are unable to move so diffusion normally should not be taking place. Active transport is the movement of molecules against the concentration gradient and requires adenosine triphosphate (ATP) as an energy source; this process typically takes place at the cellular level and is not involved in vascular volume changes.

A hospitalized patient with impaired vision must get a picture in his or her mind of the hospital room and its contents in order to mobilize independently and safely. What must the nurse monitor in the patient's room? A) That a commode is always available at the bedside B) That all furniture remains in the same position C) That visitors do not leave items on the bedside table D) That the patient's slippers stay under the bed

B

12. The nurse is participating in the care of a patient with increased ICP. What diagnostic test is contraindicated in this patients treatment? A) Computed tomography (CT) scan B) Lumbar puncture C) Magnetic resonance imaging (MRI) D) Venous Doppler studies

Ans: B Feedback: A lumbar puncture in a patient with increased ICP may cause the brain to herniate from the withdrawal of fluid and change in pressure during the lumbar puncture. Herniation of the brain is a dire and frequently fatal event. CT, MRI, and venous Doppler are considered noninvasive procedures and they would not affect the ICP itself.

38. A nurse is caring for a patient who experiences debilitating cluster headaches. The patient should be taught to take appropriate medications at what point in the course of the onset of a new headache? A) As soon as the patients pain becomes unbearable B) As soon as the patient senses the onset of symptoms C) Twenty to 30 minutes after the onset of symptoms D) When the patient senses his or her symptoms peaking

Ans: B Feedback: A migraine or a cluster headache in the early phase requires abortive medication therapy instituted as soon as possible. Delaying medication administration would lead to unnecessary pain.

16. A clinic nurse is caring for a patient diagnosed with migraine headaches. During the patient teaching session, the patient questions the nurse regarding alcohol consumption. What would the nurse be correct in telling the patient about the effects of alcohol? A) Alcohol causes hormone fluctuations. B) Alcohol causes vasodilation of the blood vessels. C) Alcohol has an excitatory effect on the CNS. D) Alcohol diminishes endorphins in the brain.

Ans: B Feedback: Alcohol causes vasodilation of the blood vessels and may exacerbate migraine headaches. Alcohol has a depressant effect on the CNS. Alcohol does not cause hormone fluctuations, nor does it decrease endorphins (morphine-like substances produced by the body) in the brain.

10. While completing a health history on a patient who has recently experienced a seizure, the nurse would assess for what characteristic associated with the postictal state? A) Epileptic cry B) Confusion C) Urinary incontinence D) Body rigidity

Ans: B Feedback: In the postictal state (after the seizure), the patient is often confused and hard to arouse and may sleep for hours. The epileptic cry occurs from the simultaneous contractions of the diaphragm and chest muscles that occur during the seizure. Urinary incontinence and intense rigidity of the entire body are followed by alternating muscle relaxation and contraction (generalized tonicclonic contraction) during the seizure.

9. A patient exhibiting an altered level of consciousness (LOC) due to blunt-force trauma to the head is admitted to the ED. The physician determines the patients injury is causing increased intracranial pressure (ICP). The nurse should gauge the patients LOC on the results of what diagnostic tool? A) Monro-Kellie hypothesis B) Glasgow Coma Scale C) Cranial nerve function D) Mental status examination

Ans: B Feedback: LOC, a sensitive indicator of neurologic function, is assessed based on the criteria in the Glasgow Coma Scale: eye opening, verbal response, and motor response. The Monro-Kellie hypothesis states that because of the limited space for expansion within the skull, an increase in any one of the components (blood, brain tissue, cerebrospinal fluid) causes a change in the volume of the others. Cranial nerve function and the mental status examination would be part of the neurologic examination for this patient, but would not be the priority in evaluating LOC.

2. The nurse is providing care for a patient who is unconscious. What nursing intervention takes highest priority? A) Maintaining accurate records of intake and output B) Maintaining a patent airway C) Inserting a nasogastric (NG) tube as ordered D) Providing appropriate pain control

Ans: B Feedback: Maintaining a patent airway always takes top priority, even though each of the other listed actions is necessary and appropriate.

13. The nurse is caring for a patient who is in status epilepticus. What medication does the nurse know may be given to halt the seizure immediately? A) Intravenous phenobarbital (Luminal) B) Intravenous diazepam (Valium) C) Oral lorazepam (Ativan) D) Oral phenytoin (Dilantin)

Ans: B Feedback: Medical management of status epilepticus includes IV diazepam (Valium) and IV lorazepam (Ativan) given slowly in an attempt to halt seizures immediately. Other medications (phenytoin, phenobarbital) are given later to maintain a seizure-free state. Oral medications are not given during status epilepticus.

The nurse is discussing macrovascular complications of diabetes with a patient. The nurse would address what topic during this dialogue? A) The need for frequent eye examinations for patients with diabetes B) The fact that patients with diabetes have an elevated risk of myocardial infarction C) The relationship between kidney function and blood glucose levels D) The need to monitor urine for the presence of albumin

Ans: B Feedback: Myocardial infarction and stroke are considered macrovascular complications of diabetes, while the effects on vision and renal function are considered to be microvascular.

34. A school nurse is called to the playground where a 6-year-old girl has been found unresponsive and staring into space, according to the playground supervisor. How would the nurse document the girls activity in her chart at school? A) Generalized seizure B) Absence seizure C) Focal seizure D) Unclassified seizure

Ans: B Feedback: Staring episodes characterize an absence seizure, whereas focal seizures, generalized seizures, and unclassified seizures involve uncontrolled motor activity.

15. The neurologic ICU nurse is admitting a patient following a craniotomy using the supratentorial approach. How should the nurse best position the patient? A) Position the patient supine. B) Maintain head of bed (HOB) elevated at 30 to 45 degrees. C) Position patient in prone position. D) Maintain bed in Trendelenberg position.

Ans: B Feedback: The patient undergoing a craniotomy with a supratentorial (above the tentorium) approach should be placed with the HOB elevated 30 to 45 degrees, with the neck in neutral alignment. Each of the other listed positions would cause a dangerous elevation in ICP.

An occupational health nurse is screening a group of workers for diabetes. What statement should the nurse interpret as suggestive of diabetes? A) "I've always been a fan of sweet foods, but lately I'm turned off by them." B) "Lately, I drink and drink and can't seem to quench my thirst." C) "No matter how much sleep I get, it seems to take me hours to wake up." D) "When I went to the washroom the last few days, my urine smelled odd."

Ans: B Feedback: Classic clinical manifestations of diabetes include the "three Ps": polyuria, polydipsia, and polyphagia. Lack of interest in sweet foods, fatigue, and foul-smelling urine are not suggestive of diabetes.

A patient with type 2 diabetes achieves adequate glycemic control through diet and exercise. Upon being admitted to the hospital for a cholecystectomy, however, the patient has required insulin injections on two occasions. The nurse would identify what likely cause for this short-term change in treatment? A) Alterations in bile metabolism and release have likely caused hyperglycemia. B) Stress has likely caused an increase in the patient's blood sugar levels. C) The patient has likely overestimated her ability to control her diabetes using nonpharmacologic measures. D) The patient's volatile fluid balance surrounding surgery has likely caused unstable blood sugars.

Ans: B Feedback: During periods of physiologic stress, such as surgery, blood glucose levels tend to increase, because levels of stress hormones (epinephrine, norepinephrine, glucagon, cortisol, and growth hormone) increase. The patient's need for insulin is unrelated to the action of bile, the patient's overestimation of previous blood sugar control, or fluid imbalance.

A patient with a history of type 1 diabetes has just been admitted to the critical care unit (CCU) for diabetic ketoacidosis. The CCU nurse should prioritize what assessment during the patient's initial phase of treatment? A) Monitoring the patient for dysrhythmias B) Maintaining and monitoring the patient's fluid balance C) Assessing the patient's level of consciousness D) Assessing the patient for signs and symptoms of venous thromboembolism

Ans: B Feedback: In addition to treating hyperglycemia, management of DKA is aimed at correcting dehydration, electrolyte loss, and acidosis before correcting the hyperglycemia with insulin. The nurse should monitor the patient for dysrhythmias, decreased LOC and VTE, but restoration and maintenance of fluid balance is the highest priority.

A nurse is caring for a patient with type 1 diabetes who is being discharged home tomorrow. What is the best way to assess the patient's ability to prepare and self-administer insulin? A) Ask the patient to describe the process in detail. B) Observe the patient drawing up and administering the insulin. C) Provide a health education session reviewing the main points of insulin delivery. D) Review the patient's first hemoglobin A1C result after discharge.

Ans: B Feedback: Nurses should assess the patient's ability to perform diabetes related self-care as soon as possible during the hospitalization or office visit to determine whether the patient requires further diabetes teaching. While consulting a home care nurse is beneficial, an initial assessment should be performed during the hospitalization or office visit. Nurses should directly observe the patient performing the skills such as insulin preparation and infection, blood glucose monitoring, and foot care. Simply questioning the patient about these skills without actually observing performance of the skill is not sufficient. Further education does not guarantee learning.

A medical nurse is caring for a patient with type 1 diabetes. The patient's medication administration record includes the administration of regular insulin three times daily. Knowing that the patient's lunch tray will arrive at 11:45, when should the nurse administer the patient's insulin? A) 10:45 B) 11:15 C) 11:45 D) 11:50

Ans: B Feedback: Regular insulin is usually administered 20-30 min before a meal. Earlier administration creates a risk for hypoglycemia; later administration creates a risk for hyperglycemia.

The nurse is caring for a patient who is postoperative following a craniotomy. When writing the plan of care, the nurse identifies a diagnosis of deficient fluid volume related to fluid restriction and osmotic diuretic use. What would be an appropriate intervention for this diagnosis? A) Change the patients position as indicated. B) Monitor serum electrolytes. C) Maintain NPO status. D) Monitor arterial blood gas (ABG) values.

Ans: B Feedback: The postoperative fluid regimen depends on the type of neurosurgical procedure and is determined on an individual basis. The volume and composition of fluids are adjusted based on daily serum electrolyte values, along with fluid intake and output. Fluids may have to be restricted in patients with cerebral edema. Changing the patients position, maintaining an NPO status, and monitoring ABG values do not relate to the nursing diagnosis of deficient fluid volume.

1. A nurse is caring for a patient who has just been diagnosed with a peptic ulcer. When teaching the patient about his new diagnosis, how should the nurse best describe a peptic ulcer? Inflammation of the lining of the stomach Erosion of the lining of the stomach or intestine Bleeding from the mucosa in the stomach Viral invasion of the stomach wall

Ans: B Feedback: A peptic ulcer is erosion of the lining of the stomach or intestine. Peptic ulcers are often accompanied by bleeding and inflammation, but these are not the definitive characteristics.

27. A patient with a history of peptic ulcer disease has presented to the emergency department (ED) in distress. What assessment finding would lead the ED nurse to suspect that the patient has a perforated ulcer? The patient has abdominal bloating that developed rapidly. The patient has a rigid, "boardlike" abdomen that is tender. The patient is experiencing intense lower right quadrant pain. The patient is experiencing dizziness and confusion with no apparent hemodynamic changes.

Ans: B Feedback: An extremely tender and rigid (boardlike) abdomen is suggestive of a perforated ulcer. None of the other listed signs and symptoms is suggestive of a perforated ulcer.

6. A nurse caring for a patient who has had bariatric surgery is developing a teaching plan in anticipation of the patient's discharge. Which of the following is essential to include? Drink a minimum of 12 ounces of fluid with each meal. Eat several small meals daily spaced at equal intervals. Choose foods that are high in simple carbohydrates. Sit upright when eating and for 30 minutes afterward.

Ans: B Feedback: Due to decreased stomach capacity, the patient must consume small meals at intervals to meet nutritional requirements while avoiding a feeling of fullness and complications such as dumping syndrome. The patient should not consume fluids with meals and low-Fowler's positioning is recommended during and after meals. Carbohydrates should be limited.

34. A patient has recently received a diagnosis of gastric cancer; the nurse is aware of the importance of assessing the patient's level of anxiety. Which of the following actions is most likely to accomplish this? The nurse gauges the patient's response to hypothetical outcomes. The patient is encouraged to express fears openly. The nurse provides detailed and accurate information about the disease. The nurse closely observes the patient's body language.

Ans: B Feedback: Encouraging the patient to discuss his or her fears and anxieties is usually the best way to assess a patient's anxiety. Presenting hypothetical situations is a surreptitious and possibly inaccurate way of assessing anxiety. Observing body language is part of assessment, but it is not the complete assessment. Presenting information may alleviate anxiety for some patients, but it is not an assessment.

33. A patient who is obese is exploring bariatric surgery options and presented to a bariatric clinic for preliminary investigation. The nurse interviews the patient, analyzing and documenting the data. Which of the following nursing diagnoses may be a contraindication for bariatric surgery? Disturbed Body Image Related to Obesity Deficient Knowledge Related to Risks and Expectations of Surgery Anxiety Related to Surgery Chronic Low Self-Esteem Related to Obesity

Ans: B Feedback: It is expected that patients seeking bariatric surgery may have challenges with body image and self-esteem related to their obesity. Anxiety is also expected when facing surgery. However, if the patient's knowledge remains deficient regarding the risks and realistic expectations for surgery, this may show that the patient is not an appropriate surgical candidate.

10. A nurse is assessing a patient who has peptic ulcer disease. The patient requests more information about the typical causes of Helicobacter pylori infection. What would it be appropriate for the nurse to instruct the patient? Most affected patients acquired the infection during international travel. Infection typically occurs due to ingestion of contaminated food and water. Many people possess genetic factors causing a predisposition to H. pylori infection. The H. pylori microorganism is endemic in warm, moist climates.

Ans: B Feedback: Most peptic ulcers result from infection with the gram-negative bacteria H. pylori, which may be acquired through ingestion of food and water. The organism is endemic to all areas of the United States. Genetic factors have not been identified.

19. A patient comes to the bariatric clinic to obtain information about bariatric surgery. The nurse assesses the obese patient knowing that in addition to meeting the criterion of morbid obesity, a candidate for bariatric surgery must also demonstrate what? Knowledge of the causes of obesity and its associated risks Adequate understanding of required lifestyle changes Positive body image and high self-esteem Insight into why past weight loss efforts failed

Ans: B Feedback: Patients seeking bariatric surgery should be free of serious mental disorders and motivated to comply with lifestyle changes related to eating patterns, dietary choices, and elimination. While assessment of knowledge about causes of obesity and its associated risks as well as insight into the reasons why previous diets have been ineffective are included in the client's plan of care, these do not predict positive client outcomes following bariatric surgery. Most obese patients have an impaired body image and alteration in self-esteem. An obese patient with a positive body image would be unlikely to seek this surgery unless he or she was experiencing significant comorbidities.

30. A patient has been prescribed orlistat (Xenical) for the treatment of obesity. When providing relevant health education for this patient, the nurse should ensure the patient is aware of what potential adverse effect of treatment? Bowel incontinence Flatus with oily discharge Abdominal pain Heat intolerance

Ans: B Feedback: Side effects of orlistat include increased frequency of bowel movements, gas with oily discharge, decreased food absorption, decreased bile flow, and decreased absorption of some vitamins. This drug does not cause bowel incontinence, abdominal pain, or heat intolerance.

A nurse admits a patient who has a fracture of the nose that has resulted in a skin tear and involvement of the mucous membranes of the nasal passages. The orthopedic nurse is aware that this description likely indicates which type of fracture? A) Compression B) Compound C) Impacted D) Transverse

Ans: B Feedback: A compound fracture involves damage to the skin or mucous membranes and is also called an open fracture. A compression fracture involves compression of bone and is seen in vertebral fractures. An impacted fracture occurs when a bone fragment is driven into another bone fragment. A transverse fracture occurs straight across the bone shaft.

An older adult patient has fallen in her home and is brought to the emergency department by ambulance with a suspected fractured hip. X-rays confirm a fracture of the left femoral neck. When planning assessments during the patient's presurgical care, the nurse should be aware of the patient's heightened risk of what complication? A) Osteomyelitis B) Avascular necrosis C) Phantom pain D) Septicemia

Ans: B Feedback: Fractures of the neck of the femur may damage the vascular system that supplies blood to the head and the neck of the femur, and the bone may become ischemic. For this reason, AVN is common in patients with femoral neck fractures. Infections are not immediate complications and phantom pain applies to patients with amputations, not hip fractures.

An elite high school football player has been diagnosed with a shoulder dislocation. The patient has been treated and is eager to resume his role on his team, stating that he is not experiencing pain. What should the nurse emphasize during health education? A) The need to take analgesia regardless of the short-term absence of pain B) The importance of adhering to the prescribed treatment and rehabilitation regimen C) The fact that he has a permanently increased risk of future shoulder dislocations D) The importance of monitoring for intracapsular bleeding once he resumes playing

Ans: B Feedback: Patients who have experienced sports-related injuries are often highly motivated to return to their previous level of activity. Adherence to restriction of activities and gradual resumption of activities needs to be reinforced. Appropriate analgesia use must be encouraged, but analgesia does not necessarily have to be taken in the absence of pain. If healing is complete, the patient does not likely have a greatly increased risk of reinjury. Dislocations rarely cause bleeding after the healing process.

Which of the following is the most appropriate nursing intervention to facilitate healing in a patient who has suffered a hip fracture? A) Administer analgesics as required. B) Place a pillow between the patient's legs when turning. C) Maintain prone positioning at all times. D) Encourage internal and external rotation of the affected leg.

Ans: B Feedback: Placing a pillow between the patient's legs when turning prevents adduction and supports the patient's legs. Administering analgesics addresses pain but does not directly protect bone remodeling and promote healing. Rotation of the affected leg can cause dislocation and must be avoided. Prone positioning does not need to be maintained at all times.

The orthopedic nurse should assess for signs and symptoms of Volkmann's contracture if a patient has fractured which of the following bones? A) Femur B) Humerus C) Radial head D) Clavicle

Ans: B Feedback: The most serious complication of a supracondylar fracture of the humerus is Volkmann's ischemic contracture, which results from antecubital swelling or damage to the brachial artery. This complication is specific to humeral fractures.

A patient has sustained a long bone fracture and the nurse is preparing the patient's care plan. Which of the following should the nurse include in the care plan? A) Administer vitamin D and calcium supplements as ordered. B) Monitor temperature and pulses of the affected extremity. C) Perform passive range of motion exercises as tolerated. D) Administer corticosteroids as ordered.

Ans: B Feedback: The nurse should include monitoring for sufficient blood supply by assessing the color, temperature, and pulses of the affected extremity. Weight-bearing exercises are encouraged, but passive ROM exercises have the potential to cause pain and inhibit healing. Corticosteroids, vitamin D, and calcium are not normally administered.

A patient with a simple arm fracture is receiving discharge education from the nurse. What would the nurse instruct the patient to do? A) Elevate the affected extremity to shoulder level when at rest. B) Engage in exercises that strengthen the unaffected muscles. C) Apply topical anesthetics to accessible skin surfaces as needed. D) Avoid using analgesics so that further damage is not masked.

Ans: B Feedback: The nurse will encourage the patient to engage in exercises that strengthen the unaffected muscles. Comfort measures may include appropriate use of analgesics and elevation of the affected extremity to the heart level. Topical anesthetics are not typically used.

The nurse is providing care for a patient who has had a below-the-knee amputation. The nurse enters the patient's room and finds him resting in bed with his residual limb supported on pillow. What is the nurse's most appropriate action? A) Inform the surgeon of this finding. B) Explain the risks of flexion contracture to the patient. C) Transfer the patient to a sitting position. D) Encourage the patient to perform active ROM exercises with the residual limb.

Ans: B Feedback: The residual limb should not be placed on a pillow, because a flexion contracture of the hip may result. There is no acute need to contact the patient's surgeon. Encouraging exercise or transferring the patient does not address the risk of flexion contracture.

A nurse's assessment of a patient's knee reveals edema, tenderness, muscle spasms, and ecchymosis. The patient states that 2 days ago he ran 10 miles and now it "really hurts to stand up." The nurse should plan care based on the belief that the patient has experienced what? A) A first-degree strain B) A second-degree strain C) A first-degree sprain D) A second-degree sprain

Ans: B Feedback: A second-degree strain involves tearing of muscle fibers and is manifested by notable loss of load-bearing strength with accompanying edema, tenderness, muscle spasm, and ecchymosis. A first-degree strain reflects tearing of a few muscle fibers and is accompanied by minor edema, tenderness, and mild muscle spasm, without noticeable loss of function. However, this patient states a loss of function. A sprain normally involves twisting, which is inconsistent with the patient's overuse injury.

23. A nurse is providing care for a patient who is postoperative day 2 following gastric surgery. The nurse's assessment should be planned in light of the possibility of what potential complications? Select all that apply. Malignant hyperthermia Atelectasis Pneumonia Metabolic imbalances Chronic gastritis

Ans: B, C, D Feedback: After surgery, the nurse assesses the patient for complications secondary to the surgical intervention, such as pneumonia, atelectasis, or metabolic imbalances resulting from the GI disruption. Malignant hyperthermia is an intraoperative complication. Chronic gastritis is not a surgical complication.

A nurse in a busy emergency department provides care for many patients who present with contusions, strains, or sprains. Treatment modalities that are common to all of these musculoskeletal injuries include which of the following? Select all that apply. A) Massage B) Applying ice C) Compression dressings D) Resting the affected extremity E) Corticosteroids F) Elevating the injured limb

Ans: B, C, D, F Feedback: Treatment of contusions, strains, and sprains consists of resting and elevating the affected part, applying cold, and using a compression bandage. Massage and corticosteroids are not used to treat these injuries.

An adult patient has been diagnosed with diverticular disease after ongoing challenges with constipation. The patient will be treated on an outpatient basis. What components of treatment should the nurse anticipate? Select all that apply. A. Anticholinergic medications B. Increased fiber intake C. Enemas on alternating days D. Reduced fat intake E. Fluid reduction

Ans: B, D Feedback: Patients whose diverticular disease does not warrant hospital treatment often benefit from a high-fiber, low-fat diet. Neither enemas nor anticholinergics are indicated, and fluid intake is encouraged.

A nurse is assessing a patients stoma on postoperative day 3. The nurse notes that the stoma has a shiny appearance and a bright red color. How should the nurse best respond to this assessment finding? A. Irrigate the ostomy to clear a possible obstruction. B. Contact the primary care provider to report this finding. C. Document that the stoma appears healthy and well perfused. D. Document a nursing diagnosis of Impaired Skin Integrity.

Ans: C Feedback: A healthy, viable stoma should be shiny and pink to bright red. This finding does not indicate that the stoma is blocked or that skin integrity is compromised.

A patients screening colonoscopy revealed the presence of numerous polyps in the large bowel. What principle should guide the subsequent treatment of this patients health problem? A. Adherence to a high-fiber diet will help the polyps resolve. B. The patient should be assured that these are a normal, age-related physiologic change. C. The patients polyps constitute a risk factor for cancer. D. The presence of polyps is associated with an increased risk of bowel obstruction.

Ans: C Feedback: Although most polyps do not develop into invasive neoplasms, they must be identified and followed closely. They are very common, but are not classified as a normal, age-related physiologic change. Diet will not help them resolve and they do not typically lead to obstructions.

An older adult has a diagnosis of Alzheimers disease and has recently been experiencing fecal incontinence. However, the nurse has observed no recent change in the character of the patients stools. What is the nurses most appropriate intervention? • Keep a food diary to determine the foods that exacerbate the patients symptoms. • Provide the patient with a bland, low-residue diet. • Toilet the patient on a frequent, scheduled basis. • Liaise with the primary care provider to obtain an order for loperamide.

Ans: C Feedback: Because the patients fecal incontinence is most likely attributable to cognitive decline, frequent toileting is an appropriate intervention. Loperamide is unnecessary in the absence of diarrhea. Specific foods are not likely to be a cause of, or solution to, this patients health problem.

The nurse is preparing to insert a peripheral IV catheter into a patient who will require fluids and IV antibiotics. How should the nurse always start the process of insertion? A) Leave one hand ungloved to assess the site. B) Cleanse the skin with normal saline. C) Ask the patient about allergies to latex or iodine. D) Remove excessive hair from the selected site.

Ans: C Feedback: Before preparing the skin, the nurse should ask the patient if he or she is allergic to latex or iodine, which are products commonly used in preparing for IV therapy. A local reaction could result in irritation to the IV site, or, in the extreme, it could result in anaphylaxis, which can be life threatening. Both hands should always be gloved when preparing for IV insertion, and latex-free gloves must be used or the patient must report not having latex allergies. The skin is not usually cleansed with normal saline prior to insertion. Removing excessive hair at the selected site is always secondary to allergy inquiry.

A patients health history is suggestive of inflammatory bowel disease. Which of the following would suggest Crohns disease, rather that ulcerative colitis, as the cause of the patients signs and symptoms? A. A pattern of distinct exacerbations and remissions B. Severe diarrhea C. An absence of blood in stool D. Involvement of the rectal mucosa

Ans: C Feedback: Bloody stool is far more common in cases of UC than in Crohns. Rectal involvement is nearly 100% in cases of UC (versus 20% in Crohns) and patients with UC typically experience severe diarrhea. UC is also characterized by a pattern of remissions and exacerbations, while Crohns often has a more prolonged and variable course.

A nurse is planning discharge teaching for a 21-year-old patient with a new diagnosis of ulcerative colitis. When planning family assessment, the nurse should recognize that which of the following factors will likely have the greatest impact on the patients coping after discharge? • The familys ability to take care of the patients special diet needs • The familys ability to monitor the patients changing health status • The familys ability to provide emotional support • The familys ability to manage the patients medication regimen

Ans: C Feedback: Emotional support from the family is key to the patients coping after discharge. A 21-year-old would be expected to self-manage the prescribed medication regimen and the family would not be primarily responsible for monitoring the patients health status. It is highly beneficial if the family is willing and able to accommodate the patients dietary needs, but emotional support is paramount and cannot be solely provided by the patient alone.

A nurse is working with a patient who has chronic constipation. What should be included in patient teaching to promote normal bowel function? • Use glycerin suppositories on a regular basis. • Limit physical activity in order to promote bowel peristalsis. • Consume high-residue, high-fiber foods. • Resist the urge to defecate until the urge becomes intense.

Ans: C Feedback: Goals for the patient include restoring or maintaining a regular pattern of elimination by responding to the urge to defecate, ensuring adequate intake of fluids and high-fiber foods, learning about methods to avoid constipation, relieving anxiety about bowel elimination patterns, and avoiding complications. Ongoing use of pharmacologic aids should not be promoted, due to the risk of dependence. Increased mobility helps to maintain a regular pattern of elimination. The urge to defecate should be heeded.

One day after a patient is admitted to the medical unit, you note that the patient is oliguric. You notify the acute-care nurse practitioner who orders a fluid challenge of 200 mL of normal saline solution over 15 minutes. This intervention will achieve which of the following? A) Help distinguish hyponatremia from hypernatremia B) Help evaluate pituitary gland function C) Help distinguish reduced renal blood flow from decreased renal function D) Help provide an effective treatment for hypertension-induced oliguria

Ans: C Feedback: If a patient is not excreting enough urine, the health care provider needs to determine whether the depressed renal function is the result of reduced renal blood flow, which is a fluid volume deficit (FVD or prerenal azotemia), or acute tubular necrosis that results in necrosis or cellular death from prolonged FVD. A typical example of a fluid challenge involves administering 100 to 200 mL of normal saline solution over 15 minutes. The response by a patient with FVD but with normal renal function is increased urine output and an increase in blood pressure. Laboratory examinations are needed to distinguish hyponatremia from hypernatremia. A fluid challenge is not used to evaluate pituitary gland function. A fluid challenge may provide information regarding hypertension-induced oliguria, but it is not an effective treatment.

A patients colorectal cancer has necessitated a hemicolectomy with the creation of a colostomy. In the 4 days since the surgery, the patient has been unwilling to look at the ostomy or participate in any aspects of ostomy care. What is the nurses most appropriate response to this observation? • Ensure that the patient knows that he or she will be responsible for care after discharge. • Reassure the patient that many people are fearful after the creation of an ostomy. • Acknowledge the patients reluctance and initiate discussion of the factors underlying it. • Arrange for the patient to be seen by a social worker or spiritual advisor.

Ans: C Feedback: If the patient is reluctant to participate in ostomy care, the nurse should attempt to dialogue about this with the patient and explore the factors that underlie it. It is presumptive to assume that the patients behavior is motivated by fear. Assessment must precede referrals and emphasizing the patients responsibilities may or may not motivate the patient.

You are performing an admission assessment on an older adult patient newly admitted for end-stage liver disease. What principle should guide your assessment of the patient's skin turgor? A) Overhydration is common among healthy older adults. B) Dehydration causes the skin to appear spongy. C) Inelastic skin turgor is a normal part of aging. D) Skin turgor cannot be assessed in patients over 70.

Ans: C Feedback: Inelastic skin is a normal change of aging. However, this does not mean that skin turgor cannot be assessed in older patients. Dehydration, not overhydration, causes inelastic skin with tenting. Overhydration, not dehydration, causes the skin to appear edematous and spongy.

You are making initial shift assessments on your patients. While assessing one patient's peripheral IV site, you note edema around the insertion site. How should you document this complication related to IV therapy? A) Air emboli B) Phlebitis C) Infiltration D) Fluid overload

Ans: C Feedback: Infiltration is the administration of nonvesicant solution or medication into the surrounding tissue. This can occur when the IV cannula dislodges or perforates the wall of the vein. Infiltration is characterized by edema around the insertion site, leakage of IV fluid from the insertion site, discomfort and coolness in the area of infiltration, and a significant decrease in the flow rate. Air emboli, phlebitis, and fluid overload are not indications of infiltration.

You are caring for a 65-year-old male patient admitted to your medical unit 72 hours ago with pyloric stenosis. A nasogastric tube placed upon admission has been on low intermittent suction ever since. Upon review of the morning's blood work, you notice that the patient's potassium is below reference range. You should recognize that the patient may be at risk for what imbalance? A) Hypercalcemia B) Metabolic acidosis C) Metabolic alkalosis D) Respiratory acidosis

Ans: C Feedback: Probably the most common cause of metabolic alkalosis is vomiting or gastric suction with loss of hydrogen and chloride ions. The disorder also occurs in pyloric stenosis in which only gastric fluid is lost. Vomiting, gastric suction, and pyloric stenosis all remove potassium and can cause hypokalemia. This patient would not be at risk for hypercalcemia; hyperparathyroidism and cancer account for almost all cases of hypercalcemia. The nasogastric tube is removing stomach acid and will likely raise pH. Respiratory acidosis is unlikely since no change was reported in the patient's respiratory status.

The nurse in the medical ICU is caring for a patient who is in respiratory acidosis due to inadequate ventilation. What diagnosis could the patient have that could cause inadequate ventilation? A) Endocarditis B) Multiple myeloma C) Guillain-Barré syndrome D) Overdose of amphetamines

Ans: C Feedback: Respiratory acidosis is always due to inadequate excretion of CO2 with inadequate ventilation, resulting in elevated plasma CO2 concentrations and, consequently, increased levels of carbonic acid. Acute respiratory acidosis occurs in emergency situations, such as acute pulmonary edema, aspiration of a foreign object, atelectasis, pneumothorax, overdose of sedatives, sleep apnea, administration of oxygen to a patient with chronic hypercapnia (excessive CO2 in the blood), severe pneumonia, and acute respiratory distress syndrome. Respiratory acidosis can also occur in diseases that impair respiratory muscles, such as muscular dystrophy, myasthenia gravis, and Guillain-Barré syndrome. The other listed diagnoses are not associated with respiratory acidosis.

The current phase of a patient's treatment for a burn injury prioritizes wound care, nutritional support, and prevention of complications such as infection. Based on these care priorities, the patient is in what phase of burn care? A) Emergent B) Immediate resuscitative C) Acute D) Rehabilitation

Ans: C Feedback: The acute or intermediate phase of burn care follows the emergent/resuscitative phase and begins 48 to 72 hours after the burn injury. During this phase, attention is directed toward continued assessment and maintenance of respiratory and circulatory status, fluid and electrolyte balance, and gastrointestinal function. Infection prevention, burn wound care (i.e., wound cleaning, topical antibacterial therapy, wound dressing, dressing changes, wound débridement, and wound grafting), pain management, and nutritional support are priorities at this stage. Priorities during the emergent or immediate resuscitative phase include first aid, prevention of shock and respiratory distress, detection and treatment of concomitant injuries, and initial wound assessment and care. The priorities during the rehabilitation phase include prevention of scars and contractures, rehabilitation, functional and cosmetic reconstruction, and psychosocial counseling.

You are called to your patient's room by a family member who voices concern about the patient's status. On assessment, you find the patient tachypnic, lethargic, weak, and exhibiting a diminished cognitive ability. You also find 3+ pitting edema. What electrolyte imbalance is the most plausible cause of this patient's signs and symptoms? A) Hypocalcemia B) Hyponatremia C) Hyperchloremia D) Hypophosphatemia

Ans: C Feedback: The signs and symptoms of hyperchloremia are the same as those of metabolic acidosis: hypervolemia and hypernatremia. Tachypnea; weakness; lethargy; deep, rapid respirations; diminished cognitive ability; and hypertension occur. If untreated, hyperchloremia can lead to a decrease in cardiac output, dysrhythmias, and coma. A high chloride level is accompanied by a high sodium level and fluid retention. With hypocalcemia, you would expect tetany. There would not be edema with hyponatremia. Signs or symptoms of hypophosphatemia are mainly neurologic.

A nurse is working with a patient who is learning to care for a continent ileostomy (Kock pouch). Following the initial period of healing, the nurse is teaching the patient how to independently empty the ileostomy. The nurse should teach the patient to do which of the following actions? • Aim to eventually empty the pouch every 90 minutes. • Avoid emptying the pouch until it is visibly full. • Insert the catheter approximately 5 cm into the pouch. • Aspirate the contents of the pouch using a 60 mL piston syringe.

Ans: C Feedback: To empty a Kock pouch, the catheter is gently inserted approximately 5 cm to the point of the valve or nipple. The length of time between drainage periods is gradually increased until the reservoir needs to be drained only every 4 to 6 hours and irrigated once each day. It is not appropriate to wait until the pouch is full, and this would not be visible. The contents of the pouch are not aspirated.

The nurse is assessing the patient for the presence of a Chvostek's sign. What electrolyte imbalance would a positive Chvostek's sign indicate? A) Hypermagnesemia B) Hyponatremia C) Hypocalcemia D) Hyperkalemia

Ans: C Feedback: You can induce Chvostek's sign by tapping the patient's facial nerve adjacent to the ear. A brief contraction of the upper lip, nose, or side of the face indicates Chvostek's sign. Both hypomagnesemia and hypocalcemia may be tested using the Chvostek's sign.

A nurse who provides care on a burn unit is preparing to apply a patient's ordered topical antibiotic ointment. What action should the nurse perform when administering this medication? A) Apply the new ointment without disturbing the existing layer of ointment. B) Apply the ointment using a sterile tongue depressor. C) Apply a layer of ointment approximately 1/16 inch thick. D) Gently irrigate the wound bed after applying the antibiotic ointment.

Ans: C Feedback: After removing the old ointment from the wound bed, the nurse should apply a layer of ointment 1/16-inch thick using clean gloves. The wound would not be irrigated after application of new ointment.

A triage nurse in the emergency department (ED) receives a phone call from a frantic father who saw his 4-year-old child tip a pot of boiling water onto her chest. The father has called an ambulance. What would the nurse in the ED receiving the call instruct the father to do? A) Cover the burn with ice and secure with a towel. B) Apply butter to the area that is burned. C) Immerse the child in a cool bath. D) Avoid touching the burned area under any circumstances.

Ans: C Feedback: After the flames or heat source have been removed or extinguished, the burned area and adherent clothing are soaked with cool water briefly to cool the wound and halt the burning process. Cool water is the best first-aid measure. Ice and butter are contraindicated. Appropriate first aid necessitates touching the burn.

A nurse is developing a care plan for a patient with a partial-thickness burn, and determines that an appropriate goal is to maintain position of joints in alignment. What is the best rationale for this intervention? A) To prevent neuropathies B) To prevent wound breakdown C) To prevent contractures D) To prevent heterotopic ossification

Ans: C Feedback: To prevent the complication of contractures, the nurse will establish a goal to maintain position of joints in alignment. Gentle range of motion exercises and a consult to PT and OT for exercises and positioning recommendations are also appropriate interventions for the prevention of contractures. Joint alignment is not maintained specifically for preventing neuropathy, wound breakdown, or heterotopic ossification.

A nurse educator is reviewing peripheral IV insertion with a group of novice nurses. How should these nurses be encouraged to deal with excess hair at the intended site? A) Leave the hair intact. B) Shave the area. C) Clip the hair in the area. D) Remove the hair with a depilatory.

Ans: C Feedback: Hair can be a source of infection and should be removed by clipping; it should not be left at the site. Shaving the area can cause skin abrasions, and depilatories can irritate the skin.

A nurse is planning care for a nephrology patient with a new nursing graduate. The nurse states, "A patient in renal failure partially loses the ability to regulate changes in pH." What is the cause of this partial inability? A) The kidneys regulate and reabsorb carbonic acid to change and maintain pH. B) The kidneys buffer acids through electrolyte changes. C) The kidneys regenerate and reabsorb bicarbonate to maintain a stable pH. D) The kidneys combine carbonic acid and bicarbonate to maintain a stable pH.

Ans: C Feedback: The kidneys regulate the bicarbonate level in the ECF; they can regenerate bicarbonate ions as well as reabsorb them from the renal tubular cells. In respiratory acidosis and most cases of metabolic acidosis, the kidneys excrete hydrogen ions and conserve bicarbonate ions to help restore balance. The lungs regulate and reabsorb carbonic acid to change and maintain pH. The kidneys do not buffer acids through electrolyte changes; buffering occurs in reaction to changes in pH. Carbonic acid works as the chemical medium to exchange O2 and CO2 in the lungs to maintain a stable pH whereas the kidneys use bicarbonate as the chemical medium to maintain a stable pH by moving and eliminating H+.

20. Following a traumatic brain injury, a patient has been in a coma for several days. Which of the following statements is true of this patients current LOC? A) The patient occasionally makes incomprehensible sounds. B) The patients current LOC will likely become a permanent state. C) The patient may occasionally make nonpurposeful movements. D) The patient is incapable of spontaneous respirations.

Ans: C Feedback: Coma is a clinical state of unarousable unresponsiveness in which no purposeful responses to internal or external stimuli occur, although nonpurposeful responses to painful stimuli and brain stem reflexes may be present. Verbal sounds, however, are atypical. Ventilator support may or may not be necessary. Comas are not permanent states.

11. A patient with increased ICP has a ventriculostomy for monitoring ICP. The nurses most recent assessment reveals that the patient is now exhibiting nuchal rigidity and photophobia. The nurse would be correct in suspecting the presence of what complication? A) Encephalitis B) CSF leak C) Meningitis D) Catheter occlusion

Ans: C Feedback: Complications of a ventriculostomy include ventricular infectious meningitis and problems with the monitoring system. Nuchal rigidity and photophobia are clinical manifestations of meningitis, but are not suggestive of encephalitis, a CSF leak, or an occluded catheter.

28. A patient has experienced a seizure in which she became rigid and then experienced alternating muscle relaxation and contraction. What type of seizure does the nurse recognize? A) Unclassified seizure B) Absence seizure C) Generalized seizure D) Focal seizure

Ans: C Feedback: Generalized seizures often involve both hemispheres of the brain, causing both sides of the body to react. Intense rigidity of the entire body may occur, followed by alternating muscle relaxation and contraction (generalized tonicclonic contraction). This pattern of rigidity does not occur in patients who experience unclassified, absence, or focal seizures.

6. A patient who has been on long-term phenytoin (Dilantin) therapy is admitted to the unit. In light of the adverse of effects of this medication, the nurse should prioritize which of the following in the patients plan of care? A) Monitoring of pulse oximetry B) Administration of a low-protein diet C) Administration of thorough oral hygiene D) Fluid restriction as ordered

Ans: C Feedback: Gingival hyperplasia (swollen and tender gums) can be associated with long-term phenytoin (Dilantin) use. Thorough oral hygiene should be provided consistently and encouraged after discharge. Fluid and protein restriction are contraindicated and there is no particular need for constant oxygen saturation monitoring.

25. The nurse is caring for a patient with a brain tumor. What drug would the nurse expect to be ordered to reduce the edema surrounding the tumor? A) Solumedrol B) Dextromethorphan C) Dexamethasone D) Furosemide

Ans: C Feedback: If a brain tumor is the cause of the increased ICP, corticosteroids (e.g., dexamethasone) help reduce the edema surrounding the tumor. Solumedrol, a steroid, and furosemide, a loop diuretic, are not the drugs of choice in this instance. Dextromethorphan is used in cough medicines.

3. The nurse is caring for a patient in the ICU who has a brain stem herniation and who is exhibiting an altered level of consciousness. Monitoring reveals that the patients mean arterial pressure (MAP) is 60 mm Hg with an intracranial pressure (ICP) reading of 5 mm Hg. What is the nurses most appropriate action? A) Position the patient in the high Fowlers position as tolerated. B) Administer osmotic diuretics as ordered. C) Participate in interventions to increase cerebral perfusion pressure. D) Prepare the patient for craniotomy.

Ans: C Feedback: The cerebral perfusion pressure (CPP) is 55 mm Hg, which is considered low. The normal CPP is 70 to 100 mm Hg. Patients with a CPP of less than 50 mm Hg experience irreversible neurologic damage. As a result, interventions are necessary. A craniotomy is not directly indicated. Diuretics and increased height of bed would exacerbate the patients condition.

A nurse in a stroke rehabilitation facility recognizes that the brain regulates swallowing. Damage to what area of the brain will most affect the patients ability to swallow? A) Temporal lobe B) Medulla oblongata C) Cerebellum D) Pons

B

A public health nurse is organizing a campaign that will address the leading cause of musculoskeletal- related disability in the United States. The nurse should focus on what health problem? A) Osteoporosis B) Arthritis C) Hip fractures D) Lower back pain

B

1. A patient is being admitted to the neurologic ICU following an acute head injury that has resulted in cerebral edema. When planning this patients care, the nurse would expect to administer what priority medication? A) Hydrochlorothiazide (HydroDIURIL) B) Furosemide (Lasix) C) Mannitol (Osmitrol) D) Spirolactone (Aldactone)

Ans: C Feedback: The osmotic diuretic mannitol is given to dehydrate the brain tissue and reduce cerebral edema. This drug acts by reducing the volume of brain and extracellular fluid. Spirolactone, furosemide, and hydrochlorothiazide are diuretics that are not typically used in the treatment of increased ICP resulting from cerebral edema.

32. A patient is recovering from intracranial surgery that was performed using the transsphenoidal approach. The nurse should be aware that the patient may have required surgery on what neurologic structure? A) Cerebellum B) Hypothalamus C) Pituitary gland D) Pineal gland

Ans: C Feedback: The transsphenoidal approach (through the mouth and nasal sinuses) is often used to gain access to the pituitary gland. This surgical approach would not allow for access to the pineal gland, cerebellum, or hypothalamus.

35. A neurologic nurse is reviewing seizures with a group of staff nurses. How should this nurse best describe the cause of a seizure? A) Sudden electrolyte changes throughout the brain B) A dysrhythmia in the peripheral nervous system C) A dysrhythmia in the nerve cells in one section of the brain D) Sudden disruptions in the blood flow throughout the brain

Ans: C Feedback: The underlying cause of a seizure is an electrical disturbance (dysrhythmia) in the nerve cells in one section of the brain; these cells emit abnormal, recurring, uncontrolled electrical discharges. Seizures are not caused by changes in blood flow or electrolytes.

7. A nurse is admitting a patient with a severe migraine headache and a history of acute coronary syndrome. What migraine medication would the nurse question for this patient? A) Rizatriptan (Maxalt) B) Naratriptan (Amerge) C) Sumatriptan succinate (Imitrex) D) Zolmitriptan (Zomig)

Ans: C Feedback: Triptans can cause chest pain and are contraindicated in patients with ischemic heart disease. Maxalt, Amerge, and Zomig are triptans used in routine clinical use for the treatment of migraine headaches.

39. A nurse is collaborating with the interdisciplinary team to help manage a patients recurrent headaches. What aspect of the patients health history should the nurse identify as a potential contributor to the patients headaches? A) The patient leads a sedentary lifestyle. B) The patient takes vitamin D and calcium supplements. C) The patient takes vasodilators for the treatment of angina. D) The patient has a pattern of weight loss followed by weight gain.

Ans: C Feedback: Vasodilators are known to contribute to headaches. Weight fluctuations, sedentary lifestyle, and vitamin supplements are not known to have this effect.

A patient newly diagnosed with type 2 diabetes is attending a nutrition class. What general guideline would be important to teach the patients at this class? A) Low fat generally indicates low sugar. B) Protein should constitute 30% to 40% of caloric intake. C) Most calories should be derived from carbohydrates. D) Animal fats should be eliminated from the diet.

Ans: C Feedback: Currently, the ADA and the Academy of Nutrition and Dietetics (formerly the American Dietetic Association) recommend that for all levels of caloric intake, 50% to 60% of calories should be derived from carbohydrates, 20% to 30% from fat, and the remaining 10% to 20% from protein.Low fat does not automatically mean low sugar. Dietary animal fat does not need to be eliminated from the diet.

A nurse is providing health education to an adolescent newly diagnosed with type 1 diabetes mellitus and her family. The nurse teaches the patient and family that which of the following nonpharmacologic measures will decrease the body's need for insulin? A) Adequate sleep B) Low stimulation C) Exercise D) Low-fat diet

Ans: C Feedback: Exercise lowers blood glucose, increases levels of HDLs, and decreases total cholesterol and triglyceride levels. Low fat intake and low levels of stimulation do not reduce a patient's need for insulin. Adequate sleep is beneficial in reducing stress, but does not have an effect that is pronounced as that of exercise.

A student with diabetes tells the school nurse that he is feeling nervous and hungry. The nurse assesses the child and finds he has tachycardia and is diaphoretic with a blood glucose level of 50 mg/dL (2.8 mmol/L). What should the school nurse administer? A) A combination of protein and carbohydrates, such as a small cup of yogurt B) Two teaspoons of sugar dissolved in a cup of apple juice C) Half of a cup of juice, followed by cheese and crackers D) Half a sandwich with a protein-based filling

Ans: C Feedback: Initial treatment for hypoglycemia is 15 g concentrated carbohydrate, such as two or three glucose tablets, 1 tube glucose gel, or 0.5 cup juice. After initial treatment, the nurse should follow with a snack including starch and protein, such as cheese and crackers, milk and crackers, or half of a sandwich. It is unnecessary to add sugar to juice, even it if is labeled as unsweetened juice, because the fruit sugar in juice contains enough simple carbohydrate to raise the blood glucose level and additional sugar may result in a sharp rise in blood sugar that will last for several hours.

A diabetes educator is teaching a patient about type 2 diabetes. The educator recognizes that the patient understands the primary treatment for type 2 diabetes when the patient states what? A) "I read that a pancreas transplant will provide a cure for my diabetes." B) "I will take my oral antidiabetic agents when my morning blood sugar is high." C) "I will make sure to follow the weight loss plan designed by the dietitian." D) "I will make sure I call the diabetes educator when I have questions about my insulin."

Ans: C Feedback: Insulin resistance is associated with obesity; thus the primary treatment of type 2 diabetes is weight loss. Oral antidiabetic agents may be added if diet and exercise are not successful in controlling blood glucose levels. If maximum doses of a single category of oral agents fail to reduce glucose levels to satisfactory levels, additional oral agents may be used. Some patients may require insulin on an ongoing basis or on a temporary basis during times of acute psychological stress, but it is not the central component of type 2 treatment. Pancreas transplantation is associated with type 1 diabetes.

A nurse is assessing a patient who is experiencing peripheral neurovascular dysfunction. What assessment findings are most consistent with this diagnosis? A) Hot skin with a capillary refill of 1 to 2 seconds B) Absence of feeling, capillary refill of 4 to 5 seconds, and cool skin C) Pain, diaphoresis, and erythema D) Jaundiced skin, weakness, and capillary refill of 3 seconds

B

A patient with type 1 diabetes has told the nurse that his most recent urine test for ketones was positive. What is the nurse's most plausible conclusion based on this assessment finding? A) The patient should withhold his next scheduled dose of insulin. B) The patient should promptly eat some protein and carbohydrates. C) The patient's insulin levels are inadequate. D) The patient would benefit from a dose of metformin (Glucophage).

Ans: C Feedback: Ketones in the urine signal that there is a deficiency of insulin and that control of type 1 diabetes is deteriorating. Withholding insulin or eating food would exacerbate the patient's ketonuria. Metformin will not cause short-term resolution of hyperglycemia.

Which of the following patients with type 1 diabetes is most likely to experience adequate glucose control? A) A patient who skips breakfast when his glucose reading is greater than 220 mg/dL B) A patient who never deviates from her prescribed dose of insulin C) A patient who adheres closely to a meal plan and meal schedule D) A patient who eliminates carbohydrates from his daily intake

Ans: C Feedback: The therapeutic goal for diabetes management is to achieve normal blood glucose levels without hypoglycemia. Therefore, diabetes management involves constant assessment and modification of the treatment plan by health professionals and daily adjustments in therapy (possibly including insulin) by patients. For patients who require insulin to help control blood glucose levels, maintaining consistency in the amount of calories and carbohydrates ingested at meals is essential. In addition, consistency in the approximate time intervals between meals, and the snacks, help maintain overall glucose control. Skipping meals is never advisable for person with type 1 diabetes.

5. A nurse is preparing to discharge a patient after recovery from gastric surgery. What is an appropriate discharge outcome for this patient? The patient's bowel movements maintain a loose consistency. The patient is able to tolerate three large meals a day. The patient maintains or gains weight. The patient consumes a diet high in calcium.

Ans: C Feedback: Expected outcomes for the patient following gastric surgery include ensuring that the patient is maintaining or gaining weight (patient should be weighed daily), experiencing no excessive diarrhea, and tolerating six small meals a day. Patients may require vitamin B12 supplementation by the intramuscular route and do not require a diet excessively rich in calcium.

17. A patient is receiving education about his upcoming Billroth I procedure (gastroduodenostomy). This patient should be informed that he may experience which of the following adverse effects associated with this procedure? Persistent feelings of hunger and thirst Constipation or bowel incontinence Diarrhea and feelings of fullness Gastric reflux and belching

Ans: C Feedback: Following a Billroth I, the patient may have problems with feelings of fullness, dumping syndrome, and diarrhea. Hunger and thirst, constipation, and gastric reflux are not adverse effects associated with this procedure.

20. A nurse is providing patient education for a patient with peptic ulcer disease secondary to chronic nonsteroidal anti-inflammatory drug (NSAID) use. The patient has recently been prescribed misoprostol (Cytotec). What would the nurse be most accurate in informing the patient about the drug? It reduces the stomach's volume of hydrochloric acid It increases the speed of gastric emptying It protects the stomach's lining It increases lower esophageal sphincter pressure

Ans: C Feedback: Misoprostol is a synthetic prostaglandin that, like prostaglandin, protects the gastric mucosa. NSAIDs decrease prostaglandin production and predispose the patient to peptic ulceration. Misoprostol does not reduce gastric acidity, improve emptying of the stomach, or increase lower esophageal sphincter pressure.

7. A nurse is completing a health history on a patient whose diagnosis is chronic gastritis. Which of the data should the nurse consider most significantly related to the etiology of the patient's health problem? Consumes one or more protein drinks daily. Takes over-the-counter antacids frequently throughout the day. Smokes one pack of cigarettes daily. Reports a history of social drinking on a weekly basis.

Ans: C Feedback: Nicotine reduces secretion of pancreatic bicarbonate, which inhibits neutralization of gastric acid and can underlie gastritis. Protein drinks do not result in gastric inflammation. Antacid use is a response to experiencing symptoms of gastritis, not the etiology of gastritis. Alcohol ingestion can lead to gastritis; however, this generally occurs in patients with a history of consumption of alcohol on a daily basis.

21. A nurse is providing anticipatory guidance to a patient who is preparing for bariatric surgery. The nurse learns that the patient is anxious about numerous aspects of the surgery. What intervention is most appropriate to alleviate the patient's anxiety? Emphasize the fact that bariatric surgery has a low risk of complications. Encourage the patient to focus on the benefits of the surgery. Facilitate the patient's contact with a support group. Obtain an order for a PRN benzodiazepine.

Ans: C Feedback: Support groups can be highly beneficial in relieving preoperative and postoperative anxiety and in promoting healthy coping. This is preferable to antianxiety medications. Downplaying the risks of surgery or focusing solely on the benefits is a simplistic and patronizing approach.

15. A patient who underwent gastric banding 3 days ago is having her diet progressed on a daily basis. Following her latest meal, the patient complains of dizziness and palpitations. Inspection reveals that the patient is diaphoretic. What is the nurse's best action? Insert a nasogastric tube promptly. Reposition the patient supine. Monitor the patient closely for further signs of dumping syndrome. Assess the patient for signs and symptoms of aspiration.

Ans: C Feedback: The patient's symptoms are characteristic of dumping syndrome, which results in a sensation of fullness, weakness, faintness, dizziness, palpitations, diaphoresis, cramping pains, and diarrhea. Aspiration is a less likely cause for the patient's symptoms. Supine positioning will likely exacerbate the symptoms and insertion of an NG tube is contraindicated due to the nature of the patient's surgery.

A 20 year-old is brought in by ambulance to the emergency department after being involved in a motorcycle accident. The patient has an open fracture of his tibia. The wound is highly contaminated and there is extensive soft-tissue damage. How would this patient's fracture likely be graded? A) Grade I B) Grade II C) Grade III D) Grade IV

Ans: C Feedback: Open fractures are graded according to the following criteria. Grade I is a clean wound less than 1 cm long. Grade II is a larger wound without extensive soft-tissue damage. Grade III is highly contaminated, has extensive soft-tissue damage, and is the most severe. There is no grade IV fracture.

31. A patient who is obese has been unable to lose weight successfully using lifestyle modifications and has mentioned the possibility of using weight-loss medications. What should the nurse teach the patient about pharmacologic interventions for the treatment of obesity? "Weight loss drugs have many side effects, and most doctors think they'll all be off the market in a few years." "There used to be a lot of hope that medications would help people lose weight, but it's been shown to be mostly a placebo effect." "Medications can be helpful, but few people achieve and maintain their desired weight loss with medications alone." "Medications are rapidly become the preferred method of weight loss in people for whom diet and exercise have not worked."

Ans: C Feedback: Though antiobesity drugs help some patients lose weight, their use rarely results in loss of more than 10% of total body weight. Patients are consequently unlikely to attain their desired weight through medication alone. They are not predicted to disappear from the market and results are not attributed to a placebo effect.

24. A patient is undergoing diagnostic testing for a tumor of the small intestine. What are the most likely symptoms that prompted the patient to first seek care? Hematemesis and persistent sensation of fullness Abdominal bloating and recurrent constipation Intermittent pain and bloody stool Unexplained bowel incontinence and fatty stools

Ans: C Feedback: When the patient is symptomatic from a tumor of the small intestine, benign tumors often present with intermittent pain. The next most common presentation is occult bleeding. The other listed signs and symptoms are not normally associated with the presentation of small intestinal tumors.

A school nurse is assessing a student who was kicked in the shin during a soccer game. The area of the injury has become swollen and discolored. The triage nurse recognizes that the patient has likely sustained what? A) Sprain B) Strain C) Contusion D) Dislocation

Ans: C Feedback: A contusion is a soft-tissue injury that results in bleeding into soft tissues, creating a hematoma and ecchymosis. A sprain is an injury to ligaments caused by wrenching or twisting. A strain is a "muscle pull" from overuse, overstretching, or excessive stress. A dislocation is a condition in which the articular surfaces of the bones forming a joint are no longer in anatomic contact. Because the injury is not at the site of a joint, the patient has not experienced a sprain, strain, or dislocation.

A nurse is planning the care of a patient with osteomyelitis that resulted from a diabetic foot ulcer. The patient requires a transmetatarsal amputation. When planning the patient's postoperative care, which of the following nursing diagnoses should the nurse most likely include in the plan of care? A) Ineffective Thermoregulation B) Risk-Prone Health Behavior C) Disturbed Body Image D) Deficient Diversion Activity

Ans: C Feedback: Amputations present a serious threat to any patient's body image. None of the other listed diagnoses is specifically associated with amputation.

A patient has presented to the emergency department with an injury to the wrist. The patient is diagnosed with a third-degree strain. Why would the physician order an x-ray of the wrist? A) Nerve damage is associated with third-degree strains. B) Compartment syndrome is associated with third-degree strains. C) Avulsion fractures are associated with third-degree strains. D) Greenstick fractures are associated with third-degree strains.

Ans: C Feedback: An x-ray should be obtained to rule out bone injury, because an avulsion fracture (in which a bone fragment is pulled away from the bone by a tendon) may be associated with a third-degree strain. Nerve damage, compartment syndrome, and greenstick fractures are not associated with third-degree strains.

A nurse is caring for a patient who has suffered a hip fracture and who will require an extended hospital stay. The nurse should ensure that the patient does which of the following in order to prevent common complications associated with a hip fracture? A) Avoid requesting analgesia unless pain becomes unbearable. B) Use supplementary oxygen when transferring or mobilizing. C) Increase fluid intake and perform prescribed foot exercises. D) Remain on bed rest for 14 days or until instructed by the orthopedic surgeon.

Ans: C Feedback: Deep vein thrombosis (DVT) is among the most common complications related to a hip fracture. To prevent DVT, the nurse encourages intake of fluids and ankle and foot exercises. The patient should not be told to endure pain; a proactive approach to pain control should be adopted. While respiratory complications commonly include atelectasis and pneumonia, the use of deep-breathing exercises, changes in position at least every 2 hours, and the use of incentive spirometry help prevent the respiratory complications more than using supplementary oxygen. Bed rest may be indicated in the short term, but is not normally required for 14 days.

A nurse is performing a shift assessment on an elderly patient who is recovering after surgery for a hip fracture. The nurse notes that the patient is complaining of chest pain, has an increased heart rate, and increased respiratory rate. The nurse further notes that the patient is febrile and hypoxic, coughing, and producing large amounts of thick, white sputum. The nurse recognizes that this is a medical emergency and calls for assistance, recognizing that this patient is likely demonstrating symptoms of what complication? A) Avascular necrosis of bone B) Compartment syndrome C) Fat embolism syndrome D) Complex regional pain syndrome

Ans: C Feedback: Fat embolism syndrome occurs most frequently in young adults and elderly patients who experience fractures of the proximal femur (i.e., hip fracture). Presenting features of fat embolism syndrome include hypoxia, tachypnea, tachycardia, and pyrexia. The respiratory distress response includes tachypnea, dyspnea, wheezes, precordial chest pain, cough, large amounts of thick, white sputum, and tachycardia. Avascular necrosis (AVN) occurs when the bone loses its blood supply and dies. This does not cause coughing. Complex regional pain syndrome does not have cardiopulmonary involvement.

A patient has come to the orthopedic clinic for a follow-up appointment 6 weeks after fracturing his ankle. Diagnostic imaging reveals that bone union is not taking place. What factor may have contributed to this complication? A) Inadequate vitamin D intake B) Bleeding at the injury site C) Inadequate immobilization D) Venous thromboembolism (VTE)

Ans: C Feedback: Inadequate fracture immobilization can delay or prevent union. A short-term vitamin D deficiency would not likely prevent bone union. VTE is a serious complication but would not be a cause of nonunion. Similarly, bleeding would not likely delay union.

A nurse is caring for a patient whose skin cancer will soon be removed by excision. Which of the following actions should the nurse perform? A) Teach the patient about early signs of secondary blistering diseases. B) Teach the patient about self-care after treatment. C) Assess the patients risk for recurrent malignancy. D) Assess the patient for adverse effects of radiotherapy.

B

Six weeks after an above-the-knee amputation (AKA), a patient returns to the outpatient office for a routine postoperative checkup. During the nurse's assessment, the patient reports symptoms of phantom pain. What should the nurse tell the patient to do to reduce the discomfort of the phantom pain? A) Apply intermittent hot compresses to the area of the amputation. B) Avoid activity until the pain subsides. C) Take opioid analgesics as ordered. D) Elevate the level of the amputation site.

Ans: C Feedback: Opioid analgesics may be effective in relieving phantom pain. Heat, immobility, and elevation are not noted to relieve this form of pain.

A young patient is being treated for a femoral fracture suffered in a snowboarding accident. The nurse's most recent assessment reveals that the patient is uncharacteristically confused. What diagnostic test should be performed on this patient? A) Electrolyte assessment B) Electrocardiogram C) Arterial blood gases D) Abdominal ultrasound

Ans: C Feedback: Subtle personality changes, restlessness, irritability, or confusion in a patient who has sustained a fracture are indications for immediate arterial blood gas studies due to the possibility of fat embolism syndrome. This assessment finding does not indicate an immediate need for electrolyte levels, an ECG, or abdominal ultrasound.

A patient is being treated for a fractured hip and the nurse is aware of the need to implement interventions to prevent muscle wasting and other complications of immobility. What intervention best addresses the patient's need for exercise? A) Performing gentle leg lifts with both legs B) Performing massage to stimulate circulation C) Encouraging frequent use of the overbed trapeze D) Encouraging the patient to log roll side to side once per hour

Ans: C Feedback: The patient is encouraged to exercise as much as possible by means of the overbed trapeze. This device helps strengthen the arms and shoulders in preparation for protected ambulation. Independent logrolling may result in injury due to the location of the fracture. Leg lifts would be contraindicated for the same reason. Massage by the nurse is not a substitute for exercise.

A patient is admitted to the orthopedic unit with a fractured femur after a motorcycle accident. The patient has been placed in traction until his femur can be rodded in surgery. For what early complications should the nurse monitor this patient? Select all that apply. A) Systemic infection B) Complex regional pain syndrome C) Deep vein thrombosis D) Compartment syndrome E) Fat embolism

Ans: C, D, E Feedback: Early complications include shock, fat embolism, compartment syndrome, and venous thromboemboli (deep vein thrombosis [DVT], pulmonary embolism [PE]). Infection and CRPS are later complications of fractures.

A patient has been diagnosed with a small bowel obstruction and has been admitted to the medical unit. The nurses care should prioritize which of the following outcomes? • Preventing infection • Maintaining skin and tissue integrity • Preventing nausea and vomiting • Maintaining fluid and electrolyte balance

Ans: D Feedback: All of the listed focuses of care are important for the patient with a small bowel obstruction. However, the patients risk of fluid and electrolyte imbalances is an immediate threat to safety, and is a priority in nursing assessment and interventions.

You are an emergency-room nurse caring for a trauma patient. Your patient has the following arterial blood gas results: pH 7.26, PaCO2 28, HCO3 11 mEq/L. How would you interpret these results? A) Respiratory acidosis with no compensation B) Metabolic alkalosis with a compensatory alkalosis C) Metabolic acidosis with no compensation D) Metabolic acidosis with a compensatory respiratory alkalosis

Ans: D Feedback: A low pH indicates acidosis (normal pH is 7.35 to 7.45). The PaCO3 is also low, which causes alkalosis. The bicarbonate is low, which causes acidosis. The pH bicarbonate more closely corresponds with a decrease in pH, making the metabolic component the primary problem.

A nurse is talking with a patient who is scheduled to have a hemicolectomy with the creation of a colostomy. The patient admits to being anxious, and has many questions concerning the surgery, the care of a stoma, and necessary lifestyle changes. Which of the following nursing actions is most appropriate? • Reassure the patient that the procedure is relatively low risk and that patients are usually successful in adjusting to an ostomy. • Provide the patient with educational materials that match the patients learning style. • Encourage the patient to write down these concerns and questions to bring forward to the surgeon. • Maintain an open dialogue with the patient and facilitate a referral to the wound-ostomycontinence (WOC) nurse.

Ans: D Feedback: A wound-ostomy-continence (WOC) nurse is a registered nurse who has received advanced education in an accredited program to care for patients with stomas. The enterostomal nurse therapist can assist with the selection of an appropriate stoma site, teach about stoma care, and provide emotional support. The surgeon is less likely to address the patients psychosocial and learning needs. Reassurance does not address the patients questions, and education may or may not alleviate anxiety.

When planning the care of a patient with a fluid imbalance, the nurse understands that in the human body, water and electrolytes move from the arterial capillary bed to the interstitial fluid. What causes this to occur? A) Active transport of hydrogen ions across the capillary walls B) Pressure of the blood in the renal capillaries C) Action of the dissolved particles contained in a unit of blood D) Hydrostatic pressure resulting from the pumping action of the heart

Ans: D Feedback: An example of filtration is the passage of water and electrolytes from the arterial capillary bed to the interstitial fluid; in this instance, the hydrostatic pressure results from the pumping action of the heart. Active transport does not move water and electrolytes from the arterial capillary bed to the interstitial fluid, filtration does. The number of dissolved particles in a unit of blood is concerned with osmolality. The pressure in the renal capillaries causes renal filtration.

A patient's most recent laboratory results show a slight decrease in potassium. The physician has opted to forego drug therapy but has suggested increasing the patient's dietary intake of potassium. Which of the following would be a good source of potassium? A) Apples B) Asparagus C) Carrots D) Bananas

Ans: D Feedback: Bananas are high in potassium. Apples, carrots, and asparagus are not high in potassium.

A nurse is caring for a patient with biliary colic and is aware that the patient may experience referred abdominal pain. Where would the nurse most likely expect this patient to experience referred pain? A) Midline near the umbilicus B) Below the right nipple C) Left groin area D) Right lower abdominal quadrant

B

You are caring for a patient with a secondary diagnosis of hypermagnesemia. What assessment finding would be most consistent with this diagnosis? A) Hypertension B) Kussmaul respirations C) Increased DTRs D) Shallow respirations

Ans: D Feedback: If hypermagnesemia is suspected, the nurse monitors the vital signs, noting hypotension and shallow respirations. The nurse also observes for decreased DTRs and changes in the level of consciousness. Kussmaul breathing is a deep and labored breathing pattern associated with severe metabolic acidosis, particularly diabetic ketoacidosis (DKA), but also renal failure. This type of patient is associated with decreased DTRs, not increased DTRs.

A nurse is caring for an older adult who has been experiencing severe Clostridium difficile-related diarrhea. When reviewing the patients most recent laboratory tests, the nurse should prioritize which of the following? • White blood cell level • Creatinine level • Hemoglobin level • Potassium level

Ans: D Feedback: In elderly patients, it is important to monitor the patients serum electrolyte levels closely. Diarrhea is less likely to cause an alteration in white blood cell, creatinine, and hemoglobin levels.

Diagnostic testing has been ordered to differentiate between normal anion gap acidosis and high anion gap acidosis in an acutely ill patient. What health problem typically precedes normal anion gap acidosis? A) Metastases B) Excessive potassium intake C) Water intoxication D) Excessive administration of chloride

Ans: D Feedback: Normal anion gap acidosis results from the direct loss of bicarbonate, as in diarrhea, lower intestinal fistulas, ureterostomies, and use of diuretics; early renal insufficiency; excessive administration of chloride; and the administration of parenteral nutrition without bicarbonate or bicarbonate-producing solutes (e.g., lactate). Based on these facts, the other listed options are incorrect.

The ICU nurse is caring for a patient who experienced trauma in a workplace accident. The patient is complaining of having trouble breathing with abdominal pain. An ABG reveals the following results: pH 7.28, PaCO2 50 mm Hg, HCO3- 23 mEq/L. The nurse should recognize the likelihood of what acid-base disorder? A) Respiratory acidosis B) Metabolic alkalosis C) Respiratory alkalosis D) Mixed acid-base disorder

Ans: D Feedback: Patients can simultaneously experience two or more independent acid-base disorders. A normal pH in the presence of changes in the PaCO2 and plasma HCO3- concentration immediately suggests a mixed disorder, making the other options incorrect.

A patient has been experiencing disconcerting GI symptoms that have been worsening in severity. Following medical assessment, the patient has been diagnosed with lactose intolerance. The nurse should recognize an increased need for what form of health promotion? • Annual screening colonoscopies • Adherence to recommended immunization schedules • Regular blood pressure monitoring • Frequent screening for osteoporosis

Ans: D Feedback: Persons with lactose intolerance often experience hypocalcemia and a consequent risk of osteoporosis related to malabsorption of calcium. Lactose intolerance does not create an increased need for screening for colorectal cancer, immunizations, or blood pressure monitoring.

A patient has been experiencing occasional episodes of constipation and has been unable to achieve consistent relief by increasing physical activity and improving his diet. What pharmacologic intervention should the nurse recommend to the patient for ongoing use? • Mineral oil enemas • Bisacodyl (Dulcolax) • Senna (Senokot) • Psyllium hydrophilic mucilloid (Metamucil)

Ans: D Feedback: Psyllium hydrophilic mucilloid (Metamucil) is a bulk-forming laxative that is safe for ongoing use. None of the other listed laxatives should be used on an ongoing basis because of the risk of dependence.

A 73-year-old man comes into the emergency department (ED) by ambulance after slipping on a small carpet in his home. The patient fell on his hip with a resultant fracture. He is alert and oriented; his pupils are equal and reactive to light and accommodation. His heart rate is elevated, he is anxious and thirsty, a Foley catheter is placed, and 40 mL of urine is present. What is the nurse's most likely explanation for the low urine output? A) The man urinated prior to his arrival to the ED and will probably not need to have the Foley catheter kept in place. B) The man likely has a traumatic brain injury, lacks antidiuretic hormone (ADH), and needs vasopressin. C) The man is experiencing symptoms of heart failure and is releasing atrial natriuretic peptide that results in decreased urine output. D) The man is having a sympathetic reaction, which has stimulated the renin-angiotensin-aldosterone system that results in diminished urine output.

Ans: D Feedback: Renin is released by the juxtaglomerular cells of the kidneys in response to decreased renal perfusion. Angiotensin-converting enzyme converts angiotensin I to angiotensin II. Angiotensin II, with its vasoconstrictor properties, increases arterial perfusion pressure and stimulates thirst. As the sympathetic nervous system is stimulated, aldosterone is released in response to an increased release of renin, which decreases urine production. Based on the nursing assessment and mechanism of injury, this is the most likely causing the lower urine output. The man urinating prior to his arrival to the ED is unlikely; the fall and hip injury would make his ability to urinate difficult. No assessment information indicates he has a head injury or heart failure.

The baroreceptors, located in the left atrium and in the carotid and aortic arches, respond to changes in the circulating blood volume and regulate sympathetic and parasympathetic neural activity as well as endocrine activities. Sympathetic stimulation constricts renal arterioles, causing what effect? A) Decrease in the release of aldosterone B) Increase of filtration in the Loop of Henle C) Decrease in the reabsorption of sodium D) Decrease in glomerular filtration

Ans: D Feedback: Sympathetic stimulation constricts renal arterioles; this decreases glomerular filtration, increases the release of aldosterone, and increases sodium and water reabsorption. None of the other listed options occurs with increased sympathetic stimulation.

36. The nurse is caring for a patient who has undergone supratentorial removal of a pituitary mass. What medication would the nurse expect to administer prophylactically to prevent seizures in this patient? A) Prednisone B) Dexamethasone C) Cafergot D) Phenytoin

Ans: D Feedback: Antiseizure medication (phenytoin, diazepam) is often prescribed prophylactically for patients who have undergone supratentorial craniotomy because of the high risk of seizures after this procedure. Prednisone and dexamethasone are steroids and do not prevent seizures. Cafergot is used in the treatment of migraines.

You are caring for a patient who is being treated on the oncology unit with a diagnosis of lung cancer with bone metastases. During your assessment, you note the patient complains of a new onset of weakness with abdominal pain. Further assessment suggests that the patient likely has a fluid volume deficit. You should recognize that this patient may be experiencing what electrolyte imbalance? A) Hypernatremia B) Hypomagnesemia C) Hypophosphatemia D) Hypercalcemia

Ans: D Feedback: The most common causes of hypercalcemia are malignancies and hyperparathyroidism. Anorexia, nausea, vomiting, and constipation are common symptoms of hypercalcemia. Dehydration occurs with nausea, vomiting, anorexia, and calcium reabsorption at the proximal renal tubule. Abdominal and bone pain may also be present. Primary manifestations of hypernatremia are neurologic and would not include abdominal pain and dehydration. Tetany is the most characteristic manifestation of hypomagnesemia, and this scenario does not mention tetany. The patient's presentation is inconsistent with hypophosphatemia.

A nurse is teaching a group of adults about screening and prevention of colorectal cancer. The nurse should describe which of the following as the most common sign of possible colon cancer? • Development of new hemorrhoids • Abdominal bloating and flank pain • Unexplained weight gain • Change in bowel habits

Ans: D Feedback: The most common presenting symptom associated with colorectal cancer is a change in bowel habits. The passage of blood is the second most common symptom. Symptoms may also include unexplained anemia, anorexia, weight loss, and fatigue. Hemorrhoids and bloating are atypical.

Which of the following is the most plausible nursing diagnosis for a patient whose treatment for colon cancer has necessitated a colonostomy? • Risk for Unstable Blood Glucose Due to Changes in Digestion and Absorption • Unilateral Neglect Related to Decreased Physical Mobility • Risk for Excess Fluid Volume Related to Dietary Changes and Changes In Absorption • Ineffective Sexuality Patterns Related to Changes in Self-Concept

Ans: D Feedback: The presence of an ostomy frequently has an effect on sexuality; this should be addressed thoughtfully in nursing care. None of the other listed diagnoses reflects the physiologic changes that result from colorectal surgery.

A patients large bowel obstruction has failed to resolve spontaneously and the patients worsening condition has warranted admission to the medical unit. Which of the following aspects of nursing care is most appropriate for this patient? • Administering bowel stimulants as ordered • Administering bulk-forming laxatives as ordered • Performing deep palpation as ordered to promote peristalsis • Preparing the patient for surgical bowel resection

Ans: D Feedback: The usual treatment for a large bowel obstruction is surgical resection to remove the obstructing lesion. Administration of laxatives or bowel stimulants are contraindicated if the bowel is obstructed. Palpation would be painful and has no therapeutic benefit.

You are working on a burns unit and one of your acutely ill patients is exhibiting signs and symptoms of third spacing. Based on this change in status, you should expect the patient to exhibit signs and symptoms of what imbalance? A) Metabolic alkalosis B) Hypermagnesemia C) Hypercalcemia D) Hypovolemia

Ans: D Feedback: Third-spacing fluid shift, which occurs when fluid moves out of the intravascular space but not into the intracellular space, can cause hypovolemia. Increased calcium and magnesium levels are not indicators of third-spacing fluid shift. Burns typically cause acidosis, not alkalosis.

A newly graduated nurse is admitting a patient with a long history of emphysema. The new nurse's preceptor is going over the patient's past lab reports with the new nurse. The nurse takes note that the patient's PaCO2 has been between 56 and 64 mm Hg for several months. The preceptor asks the new nurse why they will be cautious administering oxygen. What is the new nurse's best response? A) The patient's calcium will rise dramatically due to pituitary stimulation. B) Oxygen will increase the patient's intracranial pressure and create confusion. C) Oxygen may cause the patient to hyperventilate and become acidotic. D) Using oxygen may result in the patient developing carbon dioxide narcosis and hypoxemia.

Ans: D Feedback: When PaCO2 chronically exceeds 50 mm Hg, it creates insensitivity to CO2 in the respiratory medulla, and the use of oxygen may result in the patient developing carbon dioxide narcosis and hypoxemia. No information indicates the patient's calcium will rise dramatically due to pituitary stimulation. No feedback system that oxygen stimulates would create an increase in the patient's intracranial pressure and create confusion. Increasing the oxygen would not stimulate the patient to hyperventilate and become acidotic; rather, it would cause hypoventilation and acidosis.

A patient is brought to the emergency department from the site of a chemical fire, where he suffered a burn that involves the epidermis, dermis, and the muscle and bone of the right arm. On inspection, the skin appears charred. Based on these assessment findings, what is the depth of the burn on the patient's arm? A) Superficial partial-thickness B) Deep partial-thickness C) Full partial-thickness D) Full-thickness

Ans: D Feedback: A full-thickness burn involves total destruction of the epidermis and dermis and, in some cases, underlying tissue as well. Wound color ranges widely from white to red, brown, or black. The burned area is painless because the nerve fibers are destroyed. The wound can appear leathery; hair follicles and sweat glands are destroyed. Edema may also be present. Superficial partial-thickness burns involve the epidermis and possibly a portion of the dermis; the patient will experience pain that is soothed by cooling. Deep partial-thickness burns involve the epidermis, upper dermis, and portion of the deeper dermis; the patient will complain of pain and sensitivity to cold air. Full partial thickness is not a depth of burn.

The nurse caring for a patient who is recovering from full-thickness burns is aware of the patient's risk for contracture and hypertrophic scarring. How can the nurse best mitigate this risk? A) Apply skin emollients as ordered after granulation has occurred. B) Keep injured areas immobilized whenever possible to promote healing. C) Administer oral or IV corticosteroids as ordered. D) Encourage physical activity and range of motion exercises.

Ans: D Feedback: Exercise and the promotion of mobility can reduce the risk of contracture and hypertrophic scarring. Skin emollients are not normally used in the treatment of burns, and these do not prevent scarring. Steroids are not used to reduce scarring, as they also slow the healing process.

An occupational health nurse is called to the floor of a factory where a worker has sustained a flash burn to the right arm. The nurse arrives and the flames have been extinguished. The next step is to "cool the burn." How should the nurse cool the burn? A) Apply ice to the site of the burn for 5 to 10 minutes. B) Wrap the patient's affected extremity in ice until help arrives. C) Apply an oil-based substance or butter to the burned area until help arrives. D) Wrap cool towels around the affected extremity intermittently.

Ans: D Feedback: Once the burn has been sustained, the application of cool water is the best first-aid measure. Soaking the burn area intermittently in cool water or applying cool towels gives immediate and striking relief from pain, and limits local tissue edema and damage. However, never apply ice directly to the burn, never wrap the person in ice, and never use cold soaks or dressings for longer than several minutes; such procedures may worsen the tissue damage and lead to hypothermia in people with large burns. Butter is contraindicated.

A nurse is caring for a patient who has sustained a deep partial-thickness burn injury. In prioritizing the nursing diagnoses for the plan of care, the nurse will give the highest priority to what nursing diagnosis? A) Activity Intolerance B) Anxiety C) Ineffective Coping D) Acute Pain

Ans: D Feedback: Pain is inevitable during recovery from any burn injury. Pain in the burn patient has been described as one of the most severe causes of acute pain. Management of the often-severe pain is one of the most difficult challenges facing the burn team. While the other nursing diagnoses listed are valid, the presence of pain may contribute to these diagnoses. Management of the patient's pain is the priority, as it may have a direct correlation to the other listed nursing diagnoses.

An emergency department nurse has just admitted a patient with a burn. What characteristic of the burn will primarily determine whether the patient experiences a systemic response to this injury? A) The length of time since the burn B) The location of burned skin surfaces C) The source of the burn D) The total body surface area (TBSA) affected by the burn

Ans: D Feedback: Systemic effects are a result of several variables. However, TBSA and wound severity are considered the major factors that affect the presence or absence of systemic effects.

A patient is brought to the ED by paramedics, who report that the patient has partial-thickness burns on the chest and legs. The patient has also suffered smoke inhalation. What is the priority in the care of a patient who has been burned and suffered smoke inhalation? A) Pain B) Fluid balance C) Anxiety and fear D) Airway management

Ans: D Feedback: Systemic threats from a burn are the greatest threat to life. The ABCs of all trauma care apply during the early postburn period. While all options should be addressed, pain, fluid balance, and anxiety and fear do not take precedence over airway management.

While performing a patient's ordered wound care for the treatment of a burn, the patient has made a series of sarcastic remarks to the nurse and criticized her technique. How should the nurse best interpret this patient's behavior? A) The patient may be experiencing an adverse drug reaction that is affecting his cognition and behavior. B) The patient may be experiencing neurologic or psychiatric complications of his injuries. C) The patient may be experiencing inconsistencies in the care that he is being provided. D) The patient may be experiencing anger about his circumstances that he is deflecting toward the nurse.

Ans: D Feedback: The patient may experience feelings of anger. The anger may be directed outward toward those who escaped unharmed or toward those who are now providing care. While drug reactions, complications, and frustrating inconsistencies in care cannot be automatically ruled out, it is not uncommon for anger to be directed at caregivers.

A patient arrives in the emergency department after being burned in a house fire. The patient's burns cover the face and the left forearm. What extent of burns does the patient most likely have? A) 13% B) 25% C) 9% D) 18%

Ans: D Feedback: When estimating the percentage of body area or burn surface area that has been burned, the Rule of Nines is used: the face is 9%, and the forearm is 9% for a total of 18% in this patient.

You are caring for a patient who has a diagnosis of syndrome of inappropriate antidiuretic hormone secretion (SIADH). Your patient's plan of care includes assessment of specific gravity every 4 hours. The results of this test will allow the nurse to assess what aspect of the patient's health? A) Nutritional status B) Potassium balance C) Calcium balance D) Fluid volume status

Ans: D Feedback: A specific gravity will detect if the patient has a fluid volume deficit or fluid volume excess. Nutrition, potassium, and calcium levels are not directly indicated.

5. A patient with a documented history of seizure disorder experiences a generalized seizure. What nursing action is most appropriate? A) Restrain the patient to prevent injury. B) Open the patients jaws to insert an oral airway. C) Place patient in high Fowlers position. D) Loosen the patients restrictive clothing.

Ans: D Feedback: An appropriate nursing intervention would include loosening any restrictive clothing on the patient. No attempt should be made to restrain the patient during the seizure because muscular contractions are strong and restraint can produce injury. Do not attempt to pry open jaws that are clenched in a spasm to insert anything. Broken teeth and injury to the lips and tongue may result from such an action. If possible, place the patient on one side with head flexed forward, which allows the tongue to fall forward and facilitates drainage of saliva and mucus.

8. The nurse is caring for a patient with increased intracranial pressure (ICP). The patient has a nursing diagnosis of ineffective cerebral tissue perfusion. What would be an expected outcome that the nurse would document for this diagnosis? A) Copes with sensory deprivation. B) Registers normal body temperature. C) Pays attention to grooming. D) Obeys commands with appropriate motor responses

Ans: D Feedback: An expected outcome of the diagnosis of ineffective cerebral tissue perfusion in a patient with increased intracranial pressure (ICP) would include obeying commands with appropriate motor responses. Vitals signs and neurologic status are assessed every 15 minutes to every hour. Coping with sensory deprivation would relate to the nursing diagnosis of disturbed sensory perception. The outcome of registers normal body temperature relates to the diagnosis of potential for ineffective thermoregulation. Body image disturbance would have a potential outcome of pays attention to grooming.

22. The nurse is providing care for a patient who is withdrawing from heavy alcohol use. The nurse and other members of the care team are present at the bedside when the patient has a seizure. In preparation for documenting this clinical event, the nurse should note which of the following? A) The ability of the patient to follow instructions during the seizure. B) The success or failure of the care team to physically restrain the patient. C) The patients ability to explain his seizure during the postictal period. D) The patients activities immediately prior to the seizure.

Ans: D Feedback: Before and during a seizure, the nurse observes the circumstances before the seizure, including visual, auditory, or olfactory stimuli; tactile stimuli; emotional or psychological disturbances; sleep; and hyperventilation. Communication with the patient is not possible during a seizure and physical restraint is not attempted. The patients ability to explain the seizure is not clinically relevant.

18. What should the nurse suspect when hourly assessment of urine output on a patient postcraniotomy exhibits a urine output from a catheter of 1,500 mL for two consecutive hours? A) Cushing syndrome B) Syndrome of inappropriate antidiuretic hormone (SIADH) C) Adrenal crisis D) Diabetes insipidus

Ans: D Feedback: Diabetes insipidus is an abrupt onset of extreme polyuria that commonly occurs in patients after brain surgery. Cushing syndrome is excessive glucocorticoid secretion resulting in sodium and water retention. SIADH is the result of increased secretion of ADH; the patient becomes volume-overloaded, urine output diminishes, and serum sodium concentration becomes dilute. Adrenal crisis is undersecretion of glucocorticoids resulting in profound hypoglycemia, hypovolemia, and hypotension.

27. A patient has a poor prognosis after being involved in a motor vehicle accident resulting in a head injury. As the patients ICP increases and condition worsens, the nurse knows to assess for indications of approaching death. These indications include which of the following? A) Hemiplegia B) Dry mucous membranes C) Signs of internal bleeding D) Loss of brain stem reflexes

Ans: D Feedback: Loss of brain stem reflexes, including pupillary, corneal, gag, and swallowing reflexes, is an ominous sign of approaching death. Dry mucous membranes, hemiplegia, and bleeding must be promptly addressed, but none of these is a common sign of impending death.

A school nurse is teaching a group of high school students about risk factors for diabetes. Which of the following actions has the greatest potential to reduce an individual's risk for developing diabetes? A) Have blood glucose levels checked annually. B) Stop using tobacco in any form. C) Undergo eye examinations regularly. D) Lose weight, if obese.

Ans: D Feedback: Obesity is a major modifiable risk factor for diabetes. Smoking is not a direct risk factor for the disease. Eye examinations are necessary for persons who have been diagnosed with diabetes, but they do not screen for the disease or prevent it. Similarly, blood glucose checks do not prevent the diabetes.

19. During the examination of an unconscious patient, the nurse observes that the patients pupils are fixed and dilated. What is the most plausible clinical significance of the nurses finding? A) It suggests onset of metabolic problems. B) It indicates paralysis on the right side of the body. C) It indicates paralysis of cranial nerve X. D) It indicates an injury at the midbrain level.

Ans: D Feedback: Pupils that are fixed and dilated indicate injury at the midbrain level. This finding is not suggestive of unilateral paralysis, metabolic deficits, or damage to CN X.

A nurse is assessing a patient who has diabetes for the presence of peripheral neuropathy. The nurse should question the patient about what sign or symptom that would suggest the possible development of peripheral neuropathy? A) Persistently cold feet B) Pain that does not respond to analgesia C) Acute pain, unrelieved by rest D) The presence of a tingling sensation

Ans: D Feedback: Although approximately half of patients with diabetic neuropathy do not have symptoms, initial symptoms may include paresthesias (prickling, tingling, or heightened sensation) and burning sensations (especially at night). Cold and intense pain are atypical early signs of this complication.

A medical nurse is aware of the need to screen specific patients for their risk of hyperglycemic hyperosmolar syndrome (HHS). In what patient population does hyperosmolar nonketotic syndrome most often occur? A) Patients who are obese and who have no known history of diabetes B) Patients with type 1 diabetes and poor dietary control C) Adolescents with type 2 diabetes and sporadic use of antihyperglycemics D) Middle-aged or older people with either type 2 diabetes or no known history of diabetes

Ans: D Feedback: HHS occurs most often in older people (50 to 70 years of age) who have no known history of diabetes or who have type 2 diabetes.

A nurse is teaching basic "survival skills" to a patient newly diagnosed with type 1 diabetes. What topic should the nurse address? A) Signs and symptoms of diabetic nephropathy B) Management of diabetic ketoacidosis C) Effects of surgery and pregnancy on blood sugar levels D) Recognition of hypoglycemia and hyperglycemia

Ans: D Feedback: It is imperative that newly diagnosed patients know the signs and symptoms and management of hypo- and hyperglycemia. The other listed topics are valid points for education, but are not components of the patient's immediate "survival skills" following a new diagnosis.

An older adult patient with type 2 diabetes is brought to the emergency department by his daughter. The patient is found to have a blood glucose level of 623 mg/dL. The patient's daughter reports that the patient recently had a gastrointestinal virus and has been confused for the last 3 hours. The diagnosis of hyperglycemic hyperosmolar syndrome (HHS) is made. What nursing action would be a priority? A) Administration of antihypertensive medications B) Administering sodium bicarbonate intravenously C) Reversing acidosis by administering insulin D) Fluid and electrolyte replacement

Ans: D Feedback: The overall approach to HHS includes fluid replacement, correction of electrolyte imbalances, and insulin administration. Antihypertensive medications are not indicated, as hypotension generally accompanies HHS due to dehydration. Sodium bicarbonate is not administered to patients with HHS, as their plasma bicarbonate level is usually normal. Insulin administration plays a less important role in the treatment of HHS because it is not needed for reversal of acidosis, as in diabetic ketoacidosis (DKA).

A newly admitted patient with type 1 diabetes asks the nurse what caused her diabetes. When the nurse is explaining to the patient the etiology of type 1 diabetes, what process should the nurse describe? A) "The tissues in your body are resistant to the action of insulin, making the glucose levels in your blood increase." B) "Damage to your pancreas causes an increase in the amount of glucose that it releases, and there is not enough insulin to control it." C) "The amount of glucose that your body makes overwhelms your pancreas and decreases your production of insulin." D) "Destruction of special cells in the pancreas causes a decrease in insulin production. Glucose levels rise because insulin normally breaks it down."

Ans: D Feedback: Type 1 diabetes is characterized by the destruction of pancreatic beta cells, resulting in decreased insulin production, unchecked glucose production by the liver, and fasting hyperglycemia. Also, glucose derived from food cannot be stored in the liver and remains circulating in the blood, which leads to postprandial hyperglycemia. Type 2 diabetes involves insulin resistance and impaired insulin secretion. The body does not "make" glucose.

13. A patient with a peptic ulcer disease has had metronidazole (Flagyl) added to his current medication regimen. What health education related to this medication should the nurse provide? Take the medication on an empty stomach. Take up to one extra dose per day if stomach pain persists. Take at bedtime to mitigate the effects of drowsiness. Avoid drinking alcohol while taking the drug.

Ans: D Feedback: Alcohol must be avoided when taking Flagyl and the medication should be taken with food. This drug does not cause drowsiness and the dose should not be adjusted by the patient.

32. A patient has been diagnosed with peptic ulcer disease and the nurse is reviewing his prescribed medication regimen with him. What is currently the most commonly used drug regimen for peptic ulcers? Bismuth salts, antivirals, and histamine-2 (H2) antagonists H2 antagonists, antibiotics, and bicarbonate salts Bicarbonate salts, antibiotics, and ZES Antibiotics, proton pump inhibitors, and bismuth salts

Ans: D Feedback: Currently, the most commonly used therapy for peptic ulcers is a combination of antibiotics, proton pump inhibitors, and bismuth salts that suppress or eradicate H. pylori. H2 receptor antagonists are used to treat NSAID-induced ulcers and other ulcers not associated with H. pylori infection, but they are not the drug of choice. Bicarbonate salts are not used. ZES is the Zollinger-Ellison syndrome and not a drug.

22. A patient has just been diagnosed with acute gastritis after presenting in distress to the emergency department with abdominal symptoms. What would be the nursing care most needed by the patient at this time? Teaching the patient about necessary nutritional modification Helping the patient weigh treatment options Teaching the patient about the etiology of gastritis Providing the patient with physical and emotional support

Ans: D Feedback: For acute gastritis, the nurse provides physical and emotional support and helps the patient manage the symptoms, which may include nausea, vomiting, heartburn, and fatigue. The scenario describes a newly diagnosed patient; teaching about the etiology of the disease, lifestyle modifications, or various treatment options would be best provided at a later time.

18. A patient has experienced symptoms of dumping syndrome following bariatric surgery. To what physiologic phenomenon does the nurse attribute this syndrome? Irritation of the phrenic nerve due to diaphragmatic pressure Chronic malabsorption of iron and vitamins A and C Reflux of bile into the distal esophagus A sudden release of peptides

Ans: D Feedback: For many years, it had been theorized that the hypertonic gastric food boluses that quickly transit into the intestines drew extracellular fluid from the circulating blood volume into the small intestines to dilute the high concentration of electrolytes and sugars, resulting in symptoms. Now, it is thought that this rapid transit of the food bolus from the stomach into the small intestines instead causes a rapid and exuberant release of metabolic peptides that are responsible for the symptoms of dumping syndrome. It is not a result of phrenic nerve irritation, malabsorption, or bile reflux.

37. A nurse is caring for a patient hospitalized with an exacerbation of chronic gastritis. What health promotion topic should the nurse emphasize? Strategies for maintaining an alkaline gastric environment Safe technique for self-suctioning Techniques for positioning correctly to promote gastric healing Strategies for avoiding irritating foods and beverages

Ans: D Feedback: Measures to help relieve pain include instructing the patient to avoid foods and beverages that may be irritating to the gastric mucosa and instructing the patient about the correct use of medications to relieve chronic gastritis. An alkaline gastric environment is neither possible nor desirable. There is no plausible need for self-suctioning. Positioning does not have a significant effect on the presence or absence of gastric healing.

9. A community health nurse is preparing for an initial home visit to a patient discharged following a total gastrectomy for treatment of gastric cancer. What would the nurse anticipate that the plan of care is most likely to include? Enteral feeding via gastrostomy tube (G tube) Gastrointestinal decompression by nasogastric tube Periodic assessment for esophageal distension Monthly administration of injections of vitamin B12

Ans: D Feedback: Since vitamin B12 is absorbed in the stomach, the patient requires vitamin B12 replacement to prevent pernicious anemia. A gastrectomy precludes the use of a G tube. Since the stomach is absent, a nasogastric tube would not be indicated. As well, this is not possible in the home setting. Since there is no stomach to act as a reservoir and fluids and nutrients are passing directly into the jejunum, distension is unlikely.

4. A nurse is admitting a patient diagnosed with late-stage gastric cancer. The patient's family is distraught and angry that she was not diagnosed earlier in the course of her disease. What factor contributes to the fact that gastric cancer is often detected at a later stage? Gastric cancer does not cause signs or symptoms until metastasis has occurred. Adherence to screening recommendations for gastric cancer is exceptionally low. Early symptoms of gastric cancer are usually attributed to constipation. The early symptoms of gastric cancer are usually not alarming or highly unusual.

Ans: D Feedback: Symptoms of early gastric cancer, such as pain relieved by antacids, resemble those of benign ulcers and are seldom definitive. Symptoms are rarely a cause for alarm or for detailed diagnostic testing. Symptoms precede metastasis, however, and do not include constipation.

36. A nurse is caring for a patient who has a diagnosis of GI bleed. During shift assessment, the nurse finds the patient to be tachycardic and hypotensive, and the patient has an episode of hematemesis while the nurse is in the room. In addition to monitoring the patient's vital signs and level of consciousness, what would be a priority nursing action for this patient? Place the patient in a prone position. Provide the patient with ice water to slow any GI bleeding. Prepare for the insertion of an NG tube. Notify the physician.

Ans: D Feedback: The nurse must always be alert for any indicators of hemorrhagic gastritis, which include hematemesis (vomiting of blood), tachycardia, and hypotension. If these occur, the physician is notified and the patient's vital signs are monitored as the patient's condition warrants. Putting the patient in a prone position could lead to aspiration. Giving ice water is contraindicated as it would stimulate more vomiting.

25. A patient is recovering in the hospital following gastrectomy. The nurse notes that the patient has become increasingly difficult to engage and has had several angry outbursts at various staff members in recent days. The nurse's attempts at therapeutic dialogue have been rebuffed. What is the nurse's most appropriate action? Ask the patient's primary care provider to liaise between the nurse and the patient. Delegate care of the patient to a colleague. Limit contact with the patient in order to provide privacy. Make appropriate referrals to services that provide psychosocial support.

Ans: D Feedback: The nurse should enlist the services of clergy, psychiatric clinical nurse specialists, psychologists, social workers, and psychiatrists, if needed. This is preferable to delegating care, since the patient has become angry with other care providers as well. It is impractical and inappropriate to expect the primary care provider to act as a liaison. It would be inappropriate and unsafe to simply limit contact with the patient.

A nurse is preparing to discharge an emergency department patient who has been fitted with a sling to support her arm after a clavicle fracture. What should the nurse instruct the patient to do? A) Elevate the arm above the shoulder 3 to 4 times daily. B) Avoid moving the elbow, wrist, and fingers until bone remodeling is complete. C) Engage in active range of motion using the affected arm. D) Use the arm for light activities within the range of motion.

Ans: D Feedback: A patient with a clavicle fracture may use a sling to support the arm and relieve the pain. The patient may be permitted to use the arm for light activities within the range of comfort. The patient should not elevate the arm above the shoulder level until the ends of the bones have united, but the nurse should encourage the patient to exercise the elbow, wrist, and fingers.

Radiographs of a boy's upper arm show that the humerus appears to be fractured on one side and slightly bent on the other. This diagnostic result suggests what type of fracture? A) Impacted B) Compound C) Compression D) Greenstick

Ans: D Feedback: Greenstick fractures are an incomplete fracture that results in the bone being broken on one side, while the other side is bent. This is not characteristic of an impacted, compound, or compression fracture.

An emergency department patient is diagnosed with a hip dislocation. The patient's family is relieved that the patient has not suffered a hip fracture, but the nurse explains that this is still considered to be a medical emergency. What is the rationale for the nurse's statement? A) The longer the joint is displaced, the more difficult it is to get it back in place. B) The patient's pain will increase until the joint is realigned. C) Dislocation can become permanent if the process of bone remodeling begins. D) Avascular necrosis may develop at the site of the dislocation if it is not promptly resolved.

Ans: D Feedback: If a dislocation or subluxation is not reduced immediately, avascular necrosis (AVN) may develop. Bone remodeling does not take place because a fracture has not occurred. Realignment does not become more difficult with time and pain would subside with time, not become worse.

A patient is brought to the emergency department by ambulance after stepping in a hole and falling. While assessing him the nurse notes that his right leg is shorter than his left leg; his right hip is noticeably deformed and he is in acute pain. Imaging does not reveal a fracture. Which of the following is the most plausible explanation for this patient's signs and symptoms? A) Subluxated right hip B) Right hip contusion C) Hip strain D) Traumatic hip dislocation

Ans: D Feedback: Signs and symptoms of a traumatic dislocation include acute pain, change in positioning of the joint, shortening of the extremity, deformity, and decreased mobility. A subluxation would cause moderate deformity, or possibly no deformity. A contusion or strain would not cause obvious deformities.

A patient who has had an amputation is being cared for by a multidisciplinary rehabilitation team. What is the primary goal of this multidisciplinary team? A) Maximize the efficiency of care B) Ensure that the patient's health care is holistic C) Facilitate the patient's adjustment to a new body image D) Promote the patient's highest possible level of function

Ans: D Feedback: The multidisciplinary rehabilitation team helps the patient achieve the highest possible level of function and participation in life activities. The team is not primarily motivated by efficiency, the need for holistic care, or the need to foster the patient's body image, despite the fact that each of these are valid goals.

A nurse is preparing to discharge a patient from the emergency department after receiving treatment for an ankle sprain. While providing discharge education, the nurse should encourage which of the following? A) Apply heat for the first 24 to 48 hours after the injury. B) Maintain the ankle in a dependent position. C) Exercise hourly by performing rotation exercises of the ankle. D) Keep an elastic compression bandage on the ankle.

Ans: D Feedback: Treatment of a sprain consists of resting and elevating the affected part, applying cold, and using a compression bandage. After the acute inflammatory stage (usually 24 to 48 hours after injury), heat may be applied intermittently. Rotation exercises would likely be painful.

A patient who is being treated for pneumonia starts complaining of sudden shortness of breath. An arterial blood gas (ABG) is drawn. The ABG has the following values: pH 7.21, PaCO2 64 mm Hg, HCO3 = 24 mm Hg. What does the ABG reflect? A) Respiratory acidosis B) Metabolic alkalosis C) Respiratory alkalosis D) Metabolic acidosis

Ans:A Feedback: The pH is below 7.40, PaCO2 is greater than 40, and the HCO3 is normal; therefore, it is a respiratory acidosis, and compensation by the kidneys has not begun, which indicates this was probably an acute event. The HCO3 of 24 is within the normal range so it is not metabolic alkalosis. The pH of 7.21 indicates an acidosis, not alkalosis. The pH of 7.21 indicates it is an acidosis but the HCO3 of 24 is within the normal range, ruling out metabolic acidosis.

A nurse is planning preoperative teaching for a patient with hearing loss due to otosclerosis. The patient is scheduled for a stapedectomy with insertion of a prosthesis. What information is most crucial to include in the patient's preoperative teaching? A) The procedure is an effective, time-tested treatment for sensory hearing loss. B) The patient is likely to experience resolution of conductive hearing loss after the procedure. C) Several months of post-procedure rehabilitation will be needed to maximize benefits. D) The procedure is experimental, but early indications suggest great therapeutic benefits.

B

A nursing student has auscultated a patients abdomen and noted one or two bowel sounds in a 2-minute period of time. How would you tell the student to document the patients bowel sounds? A) Normal B) Hypoactive C) Hyperactive D) Paralytic ileus

B

A patient comes to the dermatology clinic requesting the removal of a port-wine stain on his right cheek. The nurse knows that the procedure especially useful in treating cutaneous vascular lesions such as port-wine stains is what? A) Skin graft B) Laser treatment C) Chemical face peeling D) Free flap

B

A patient has been diagnosed with glaucoma and the nurse is preparing health education regarding the patient's medication regimen. The patient states that she is eager to "beat this disease" and looks forward to the time that she will no longer require medication. How should the nurse best respond? A) "You have a great attitude. This will likely shorten the amount of time that you need medications." B) "In fact, glaucoma usually requires lifelong treatment with medications." C) "Most people are treated until their intraocular pressure goes below 50 mm Hg." D) "You can likely expect a minimum of 6 months of treatment."

B

A patient has been scheduled for a urea breath test in one months time. What nursing diagnosis most likely prompted this diagnostic test? A) Impaired Dentition Related to Gingivitis B) Risk For Impaired Skin Integrity Related to Peptic Ulcers C) Imbalanced Nutrition: Less Than Body Requirements Related to Enzyme Deficiency D) Diarrhea Related to Clostridium Dificile Infection

B

A patient has come to the clinic complaining of blood in his stool. A FOBT test is performed but is negative. Based on the patients history, the physician suggests a colonoscopy, but the patient refuses, citing a strong aversion to the invasive nature of the test. What other test might the physician order to check for blood in the stool? A) A laparoscopic intestinal mucosa biopsy B) A quantitative fecal immunochemical test C) Computed tomography (CT) D) Magnetic resonance imagery (MRI)

B

A patient has just returned to the surgical floor after undergoing a retinal detachment repair. The postoperative orders specify that the patient should be kept in a prone position until otherwise ordered. What should the nurse do? A) Call the physician and ask for the order to be confirmed. B) Follow the order because this position will help keep the retinal repair intact. C) Instruct the patient to maintain this position to prevent bleeding. D) Reposition the patient after the first dressing change

B

A patient has lost most of her vision as a result of macular degeneration. When attempting to meet this patient's psychosocial needs, what nursing action is most appropriate? A) Encourage the patient to focus on her use of her other senses. B) Assess and promote the patient's coping skills during interactions with the patient. C) Emphasize that her lifestyle will be unchanged once she adapts to her vision loss. D) Promote the patient's hope for recovery.

B

A patient has received a diagnosis of irritant contact dermatitis. What action should the nurse prioritize in the patients subsequent care? A) Teaching the patient to safely and effectively administer immunosuppressants B) Helping the patient identify and avoid the offending agent C) Teaching the patient how to maintain meticulous skin hygiene D) Helping the patient perform wound care in the home environment

B

A patient has undergone diagnostic testing and has been diagnosed with otosclerosis? What ear structure is primarily affected by this diagnosis? A) Malleus B) Stapes C) Incus D) Tympanic membrane

B

A patient is receiving ongoing nursing care for the treatment of Parkinsons disease. When assessing this patients gait, what finding is most closely associated with this health problem? A) Spastic hemiparesis gait B) Shuffling gait C) Rapid gait D) Steppage gait

B

A patient is scheduled for a bone scan to rule out osteosarcoma of the pelvic bones. What would be most important for the nurse to assess before the patients scan? A) That the patient completed the bowel cleansing regimen B) That the patient emptied the bladder C) That the patient is not allergic to penicillins D) That the patient has fasted for at least 8 hours

B

A patient is undergoing diagnostic testing for suspected Pagets disease. What assessment finding is most consistent with this diagnosis? A) Altered serum magnesium levels B) Altered serum calcium levels C) Altered serum potassium levels D) Altered serum sodium levels

B

A patient presents at the ED after receiving a chemical burn to the eye. What would be the nurse's initial intervention for this patient? A) Generously flush the affected eye with a dilute antibiotic solution. B) Generously flush the affected eye with normal saline or water. C) Apply a patch to the affected eye. D) Apply direct pressure to the affected eye.

B

A patient who presents for an eye examination is diagnosed as having a visual acuity of 20/40. The patient asks the nurse what these numbers specifically mean. What is a correct response by the nurse? A) "A person whose vision is 20/40 can see an object from 40 feet away that a person with 20/20 vision can see from 20 feet away." B) "A person whose vision is 20/40 can see an object from 20 feet away that a person with 20/20 vision can see from 40 feet away." C) "A person whose vision is 20/40 can see an object from 40 inches away that a person with 20/20 vision can see from 20 inches away." D) "A person whose vision is 20/40 can see an object from 20 inches away that a person with 20/20 vision can see from 40 inches away."

B

A patients sigmoidoscopy has been successfully completed and the patient is preparing to return home. Which of the following teaching points should the nurse include in the patients discharge education? A) The patient should drink at least 2 liters of fluid in the next 12 hours. B) The patient can resume a normal routine immediately. C) The patient should expect fecal urgency for several hours. D) The patient can expect some scant rectal bleeding.

B

During discharge teaching the nurse realizes that the patient is not able to read medication bottles accurately and has not been taking her medications consistently at home. How should the nurse intervene most appropriately in this situation? A) Ask the social worker to investigate alternative housing arrangements. B) Ask the social worker to investigate community support agencies. C) Encourage the patient to explore surgical corrections for the vision problem. D) Arrange for referral to a rehabilitation facility for vision training.

B

Results of a patients preliminary assessment prompted an examination of the patients carcinoembryonic antigen (CEA) levels, which have come back positive. What is the nurses most appropriate response to this finding? A) Perform a focused abdominal assessment. B) Prepare to meet the patients psychosocial needs. C) Liaise with the nurse practitioner to perform an anorectal examination. D) Encourage the patient to adhere to recommended screening protocols.

B

The nurse and a colleague are performing the Epley maneuver with a patient who has a diagnosis of benign paroxysmal positional vertigo. The nurses should begin this maneuver by performing what action? A) Placing the patient in a prone position B) Assisting the patient into a sitting position C) Instilling 15 mL of warm normal saline into one of the patient's ears D) Assessing the patient's baseline hearing by performing the whisper test

B

The nurse is caring for a patient who has undergone a mastoidectomy. In an effort to prevent postoperative infection, what intervention should the nurse implement? A) Teach the patient about the risks of ototoxic medications. B) Instruct the patient to protect the ear from water for several weeks. C) Teach the patient to remove cerumen safely at least once per week. D) Instruct the patient to protect the ear from temperature extremes until healing is complete.

B

The nurse is planning the care of a patient who is adapting to the use of a hearing aid for the first time. What is the most significant challenge experienced by a patient with hearing loss who is adapting to using a hearing aid for the first time? A) Regulating the tone and volume B) Learning to cope with amplification of background noise C) Constant irritation of the external auditory canal D) Challenges in keeping the hearing aid clean while minimizing exposure to moisture

B

The nurse is providing discharge education to an adult patient who will begin a regimen of ocular medications for the treatment of glaucoma. How can the nurse best determine if the patient is able to self-administer these medications safely and effectively? A) Assess the patient for any previous inability to self-manage medications. B) Ask the patient to demonstrate the instillation of her medications. C) Determine whether the patient can accurately describe the appropriate method of administering her medications. D) Assess the patient's functional status.

B

A patient is postoperative day 1 following gastrostomy. The nurse is planning interventions to address the nursing diagnosis of Risk for Infection Related to Presence of Wound and Tube. What intervention is most appropriate? A) Administer antibiotics via the tube as ordered. B) Wash the area around the tube with soap and water daily. C) Cleanse the skin within 2 cm of the insertion site with hydrogen peroxide once per shift. D) Irrigate the skin surrounding the insertion site with normal saline before each use.

B Feedback: Infection can be prevented by keeping the skin near the insertion site clean using soap and water. Hydrogen peroxide is not used, due to associated skin irritation. The skin around the site is not irrigated with normal saline and antibiotics are not administered to prevent site infection.

A nurse is writing a care plan for a patient with a nasogastric tube in place for gastric decompression. What risk nursing diagnosis is the most appropriate component of the care plan? A) Risk for Excess Fluid Volume Related to Enteral Feedings B) Risk for Impaired Skin Integrity Related to the Presence of NG Tube C) Risk for Unstable Blood Glucose Related to Enteral Feedings D) Risk for Impaired Verbal Communication Related to Presence of NG Tube

B Feedback: NG tubes can easily damage the delicate mucosa of the nose, sinuses, and upper airway. An NG tube does not preclude verbal communication. This patient's NG tube is in place for decompression, so complications of enteral feeding do not apply.

A patient's physician has determined that for the next 3 to 4 weeks the patient will require parenteral nutrition (PN). The nurse should anticipate the placement of what type of venous access device? A) Peripheral catheter B) Nontunneled central catheter C) Implantable port D) Tunneled central catheter

B Feedback: Nontunneled central catheters are used for short-term (less than 6 weeks) IV therapy. A peripheral catheter can be used for the administration of peripheral parenteral nutrition for 5 to 7 days. Implantable ports and tunneled central catheters are for long-term use and may remain in place for many years. Peripherally inserted central catheters (PICCs) are another potential option.

A nurse is initiating parenteral nutrition (PN) to a postoperative patient who has developed complications. The nurse should initiate therapy by performing which of the following actions? A) Starting with a rapid infusion rate to meet the patient's nutritional needs as quickly as possible B) Initiating the infusion slowly and monitoring the patient's fluid and glucose tolerance C) Changing the rate of administration every 2 hours based on serum electrolyte values D) Increasing the rate of infusion at mealtimes to mimic the circadian rhythm of the body

B Feedback: PN solutions are initiated slowly and advanced gradually each day to the desired rate as the patient's fluid and glucose tolerance permits. The formulation of the PN solutions is calculated carefully each day to meet the complete nutritional needs of the individual patient based on clinical findings and laboratory data. It is not infused more quickly at mealtimes.

A nursing educator is reviewing the care of patients with feeding tubes and endotracheal tubes (ET). The educator has emphasized the need to check for tube placement in the stomach as well as residual volume. What is the main purpose of this nursing action? A) Prevent gastric ulcers B) Prevent aspiration C) Prevent abdominal distention D) Prevent diarrhea

B Feedback: Protecting the client from aspirating is essential because aspiration can cause pneumonia, a potentially life-threatening disorder. Gastric ulcers are not a common complication of tube feeding in clients with ET tubes. Abdominal distention and diarrhea can both be associated with tube feeding, but prevention of these problems is not the primary rationale for confirming placement.

A nurse is aware of the high incidence of catheter-related bloodstream infections in patients receiving parenteral nutrition. What nursing action has the greatest potential to reduce catheter-related bloodstream infections? A) Use clean technique and wear a mask during dressing changes. B) Change the dressing no more than weekly. C) Apply antibiotic ointment around the site with each dressing change. D) Irrigate the insertion site with sterile water during each dressing change.

B Feedback: The CDC (2011) recommends changing CVAD dressings not more than every 7 days unless the dressing is damp, bloody, loose, or soiled. Sterile technique (not clean technique) is used. Irrigation and antibiotic ointments are not used.

A patient is concerned about leakage of gastric contents out of the gastric sump tube the nurse has just inserted. What would the nurse do to prevent reflux gastric contents from coming through the blue vent of a gastric sump tube? A) Prime the tubing with 20 mL of normal saline. B) Keep the vent lumen above the patient's waist. C) Maintain the patient in a high Fowler's position. D) Have the patient pin the tube to the thigh.

B Feedback: The blue vent lumen should be kept above the patient's waist to prevent reflux of gastric contents through it; otherwise it acts as a siphon. A one-way anti-reflux valve seated in the blue pigtail can prevent the reflux of gastric contents out the vent lumen. To prevent reflux, the nurse does not prime the tubing, maintain the patient in a high Fowler's position, or have the patient pin the tube to the thigh.

A nurse is caring for a patient with a subclavian central line who is receiving parenteral nutrition (PN). In preparing a care plan for this patient, what nursing diagnosis should the nurse prioritize? A) Risk for Activity Intolerance Related to the Presence of a Subclavian Catheter B) Risk for Infection Related to the Presence of a Subclavian Catheter C) Risk for Functional Urinary Incontinence Related to the Presence of a Subclavian Catheter D) Risk for Sleep Deprivation Related to the presence of a Subclavian Catheter

B Feedback: The high glucose content of PN solutions makes the solutions an idea culture media for bacterial and fungal growth, and the central venous access devices provide a port of entry. Prevention of infection is consequently a high priority. The patient will experience some inconveniences with regard to toileting, activity, and sleep, but the infection risk is a priority over each of these.

A nurse is caring for a patient who is receiving parenteral nutrition. When writing this patient's plan of care, which of the following nursing diagnoses should be included? A) Risk for Peripheral Neurovascular Dysfunction Related to Catheter Placement B) Ineffective Role Performance Related to Parenteral Nutrition C) Bowel Incontinence Related to Parenteral Nutrition D) Chronic Pain Related to Catheter Placement

B Feedback: The limitations associated with PN can make it difficult for patients to maintain their usual roles. PN does not normally cause bowel incontinence and catheters are not associated with chronic pain or neurovascular dysfunction.

A patient's enteral feedings have been determined to be too concentrated based on the patient's development of dumping syndrome. What physiologic phenomenon caused this patient's complication of enteral feeding? A) Increased gastric secretion of HCl and gastrin because of high osmolality of feeds B) Entry of large amounts of water into the small intestine because of osmotic pressure C) Mucosal irritation of the stomach and small intestine by the high concentration of the feed D) Acid-base imbalance resulting from the high volume of solutes in the feed

B Feedback: When a concentrated solution of high osmolality entering the intestines is taken in quickly or in large amounts, water moves rapidly into the intestinal lumen from fluid surrounding the organs and the vascular compartment. This results in dumping syndrome. Dumping syndrome is not the result of changes in HCl or gastrin levels. It is not caused by an acid-base imbalance or direct irritation of the GI mucosa.

A patient's NG tube has become clogged after the nurse instilled a medication that was insufficiently crushed. The nurse has attempted to aspirate with a large-bore syringe, with no success. What should the nurse do next? A) Withdraw the NG tube 3 to 5 cm and reattempt aspiration. B) Attach a syringe filled with warm water and attempt an in-and-out motion of instilling and aspirating. C) Withdraw the NG tube slightly and attempt to dislodge by flicking the tube with the fingers. D) Remove the NG tube promptly and obtain an order for reinsertion from the primary care provider.

B Feedback: When a tube is first noted to be clogged, a 30- to 60-mL syringe should be attached to the end of the tube and any contents aspirated and discarded. Then the syringe should be filled with warm water, attached to the tube again, and a back-and-forth motion initiated to help loosen the clog. Removal is not warranted at this early stage and a flicking motion is not recommended. The tube should not be withdrawn, even a few centimeters.

A patient has been admitted to the critical care unit with a diagnosis of thyroid storm. What interventions should the nurse include in this patient's immediate care? Select all that apply. A) Administering diuretics to prevent fluid overload B) Administering beta blockers to reduce heart rate C) Administering insulin to reduce blood glucose levels D) Applying interventions to reduce the patient's temperature E) Administering corticosteroids

B) Administering beta blockers to reduce heart rate D) Applying interventions to reduce the patient's temperature

A nurse is doing a shift assessment on a group of patients after first taking report. An elderly patient is having her second dose of IV antibiotics for a diagnosis of pneumonia. The nurse notices a new rash on the patients chest. The nurse should ask what priority question regarding the presence of a reddened rash? A) Is the rash worse at a particular time or season? B) Are you allergic to any foods or medication? C) Are you having any loss of sensation in that area? D) Is your rash painful?

B) Are you allergic to any foods or medication? The nurse should suspect an allergic reaction to the antibiotic therapy. Allergies can be a significant threat to the patients immediate health, thus questions addressing this possibility would be prioritized over those addressing sensation. Asking about previous rashes is important, but this should likely be framed in the context of an allergy assessment.

A gerontologic nurse is teaching a group of nursing students about integumentary changes that occur in older adults. How should these students best integrate these changes into care planning? A) By avoiding the use of moisturizing lotions on older adults skin B) By protecting older adults against shearing injuries C) By avoiding the use of ice packs to treat muscle pain D) By protecting older adults against excessive sweat accumulation

B) By protecting older adults against shearing injuries Cellular changes associated with aging include thinning at the junction of the dermis and epidermis, which creates a risk for shearing injuries. Moisturizing lotions can be safely used to address the increased dryness of older adults skin. Ice packs can be used, provided skin is assessed regularly and the patient possesses normal sensation. Older adults perspire much less than younger adults, thus sweat accumulation is rarely an issue.

A patient who has been taking corticosteroids for several months has been experiencing muscle wasting. The patient has asked the nurse for suggestions to address this adverse effect. What should the nurse recommend? A) Activity limitation to conserve energy B) Consumption of a high-protein diet C) Use of OTC vitamin D and calcium D) Passive range-of-motion exercises

B) Consumption of a high-protein diet

A patient with a diagnosis of syndrome of inappropriate antidiuretic hormone secretion (SIADH) is being cared for on the critical care unit. The priority nursing diagnosis for a patient with this condition is what? A) Risk for peripheral neurovascular dysfunction B) Excess fluid volume C) Hypothermia D) Ineffective airway clearance

B) Excess fluid volume

The physician has ordered a fluid deprivation test for a patient suspected of having diabetes insipidus. During the test, the nurse should prioritize what assessments? A) Temperature and oxygen saturation B) Heart rate and BP C) Breath sounds and bowel sounds D) Color, warmth, movement, and sensation of extremities

B) Heart rate and BP

While assisting with the surgical removal of an adrenal tumor, the OR nurse is aware that the patient's vital signs may change upon manipulation of the tumor. What vital sign changes would the nurse expect to see? A) Hyperthermia and tachypnea B) Hypertension and heart rate changes C) Hypotension and hypothermia D) Hyperthermia and bradycardia

B) Hypertension and heart rate changes

The nurse's assessment of a patient with thyroidectomy suggests tetany and a review of the most recent blood work corroborate this finding. The nurse should prepare to administer what intervention? A) Oral calcium chloride and vitamin D B) IV calcium gluconate C) STAT levothyroxine D) Administration of parathyroid hormone(PTH)

B) IV calcium gluconate

The nurse is caring for a patient at risk for an addisonian cirisis. For what associated signs and symptoms should the nurse monitor the patient? Select all that apply. A) Epitaxis B) Pallor C) Rapid respiratory rate D) Bounding pulse E) Hypotension

B) Pallor C) Rapid respiratory rate E) Hypotension

A patient with a suspected malignant melanoma is referred to the dermatology clinic. The nurse knows to facilitate what diagnostic test to rule out a skin malignancy? A) Tzanck smear B) Skin biopsy C) Patch testing D) Skin scrapings

B) Skin biopsy A skin biopsy is done to rule out malignancies of skin lesions. A Tzanck smear is used to examine cells from blistering skin conditions, such as herpes zoster. Patch testing is performed to identify substances to which the patient has developed an allergy. Skin scrapings are done for suspected fungal infections.

A wound care nurse is reviewing skin anatomy with a group of medical nurses. Which area of the skin would the nurse identify as providing a cushion between the skin layers, muscles, and bones? A) Dermis B) Subcutaneous tissue C) Epidermis D) Stratum corneum

B) Subcutaneous tissue The subcutaneous tissue, or hypodermis, is the innermost layer of the skin that is responsible for providing a cushion between the skin layers, muscles, and bones. The dermis is the largest portion of the skin, providing strength and structure. The epidermis is the outermost layer of stratified epithelial cells and composed of keratinocytes. The stratum corneum is the outermost layer of the epidermis, which provides a barrier to prevent epidermal water loss.

The nurse is caring for a patient with Addison's disease who is scheduled for discharge. When teaching the patient about hormone replacement therapy, the nurse should address what topic? A) The possibility of precipitous weight gain B) The need for lifelong steroid replacement C) The need to match the daily steroid dose to immediate symptoms D) The importance of monitoring liver function

B) The need for lifelong steroid replacement

Assessment of a patients leg reveals the presence of a 1.5-cm circular region of necrotic tissue that is deeper than the epidermis. The nurse should document the presence of what type of skin lesion? A) Keloid B) Ulcer C) Fissure D) Erosion

B) Ulcer An ulcer is skin loss extending past the epidermis with the involvement of necrotic tissue. Keloids lack necrosis and consist of scar tissue. A fissure is linear and erosions do not extend to the dermis.

What should the nurse teach the patient on corticosteroid therapy in order to reduce the patient's risk of adrenal insufficiency? A) take the medication late in the day to mimic the body's natural rhythms B) always have enough medication on hand to avoid running out C) skip up to 2 doses in cases of illness involving nausea D) take up 1 extra does per day during times of stress

B) always have enough medication on hand to avoid running out

A client is receiving pharmacologic therapy for treatment of hyperthyroidism and is prescribed propylthiouracil (PTU). when developing this clients plan of care, the nurse integrates understanding that this drug: A) suppresses release of thyroid hormone B) blocks synthesis of T3 and T4 C) reduces the amount of thyroid tissue D) destroys overactive thyroid cells

B) blocks synthesis of T3 and T4

A nurse is providing care to a client diagnosed with syndrome of inappropriate antidiuretic hormone (SIADH). The nurse understands that the primary problem involves the: A) anterior pituitary gland B) posterior pituitary gland C) thyroid gland D) adrenal gland

B) posterior pituitary gland

A nurse is reviewing the laboratory test results of a client diagnosed with SIADH. Which result would the nurse identify as reflecting this condition? select all that apply. A) sodium 140 mEq/L B) serum osmolality 260 mOsm/Kg C) urine sodium 28 mEq/L D) BUN 14 mg/dL (4.998 mmol/L) E) uric acid 2.5 mg/dL (148.7 Umol/L)

B) serum osmolality 260 mOsm/Kg C) urine sodium 28 mEq/L E) uric acid 2.5 mg/L (148.7 Umol/L)

A client with hyperthyroidism is being treated with radioactive iodine therapy. After receiving the dose of radioiodine, the nurse would assess the client for: A) hypothyroidism B) thyroid storm C) hypothermia D) agranulocytosis

B) thyroid storm

A nurse is creating a care plan for a patient who is receiving parenteral nutrition. The patient's care plan should include nursing actions relevant to what potential complications? Select all that apply. A) Dumping syndrome B) Clotted or displaced catheter C) Pneumothorax D) Hyperglycemia E) Line sepsis

B, C, D, E Feedback: Common complications of PN include a clotted or displaced catheter, pneumothorax, hyperglycemia, and infection from the venous access device (line sepsis). Dumping syndrome applies to enteral nutrition, not PN.

When caring for a patient with toxic epidermal necrolysis (TEN), the critical care nurse assesses frequently for high fever, tachycardia, and extreme weakness and fatigue. The nurse is aware that these findings are potential indicators of what? Select all that apply. A) Possible malignancy B) Epidermal necrosis C) Neurologic involvement D) Increased metabolic needs E) Possible gastrointestinal mucosal sloughing

B, D, E

A nurse is caring for a patient admitted with a suspected malabsorption disorder. The nurse knows that one of the accessory organs of the digestive system is the pancreas. What digestive enzymes does the pancreas secrete? Select all that apply. A) Pepsin B) Lipase C) Amylase D) Trypsin E) Ptyalin

B,C,D

A nurse is conducting a presentation about brain cancer for a local community group. During the presentation, one of the group members asks, "what causes brain tumors"? which response by the nurse would be most appropriate? A- "there is scientific evidence that cigarette smoking is high on the list of causes" B- "the cause of most brain tumors is still really not known" C- "its a known fact that using cell phones increases your risk for a tumor" D- "exposure to residential power lines is a definite cause of brain tumors"

B- "the cause of most brain tumors is still really not known"

The nurse is performing a neurologic assessment of a patient whose injuries have rendered her unable to follow verbal commands. How should the nurse proceed with assessing the patients level of consciousness (LOC)? A- assess the patients vital signs and correlate these with the patients baseline. B- assess the patients eye opening and response to stimuli C- document that the patient is currently lacks a level of consciousness. D- facilitate diagnostic testing in an effort to obtain objective data

B- assess the patients eye opening and response to stimuli

The nurse is planning the care of a patient with Parkinson's disease. The nurse should be aware that treatment will focus on what pathophysiological phenomenon? A- premature degradation of acetylcholine B- decreased availability of dopamine C- insufficient synthesis of epinephrine D- delayed reuptake of serotonin

B- decreased availability of dopamine

A trauma patient in the ICU has been declared brain dead. What diagnostic test is used in making the determination of brain death? A- magnetic resonance imaging (MRI) B- electroencephalography (EEG) C- electomyelography (EMG) D- computed tomography (CT)

B- electroencephalography (EEG)

The nurse has admitted a new patient to the unit. One of the patients admitting orders is for an adrenergic medication. The nurse knows that this medication will have what effect on the circulatory system? A- thin, watery saliva B- increased heart rate C- decreased BP D- constricted bronchioles

B- increased heart rate

A patient is brought to the ER following a motor vehicle accident in which he sustained head trauma. Preliminary assessment reveals a vision deficit in the patients left eye. The nurse should associate the abnormal finding with trauma to which of the following cerebral lobes? A- temporal B- occipital C- parietal D- frontal

B- occipital

The nurse is doing an initial assessment on a patient newly admitted to the unit with a diagnosis of cerebrovascular accident (CVA). The patient has difficulty copying a figure that the nurse has drawn and is diagnosed with visual-receptive aphasia. What brain region is primarily involved in this deficit? A- temporal lobe B- parietal-occipital area C- inferior posterior frontal areas D- posterior frontal area

B- parietal-occipital area

A gerontologic nurse planning the neurologic assessment of an older adult is considering normal, age-related changes. Of what phenomenon should the nurse be aware? A- hyperactive deep tendon reflexes B- reduction in cerebral blood flow C- increased cerebral metabolism D- hypersensitivity to painful stimuli

B- reduction in cerebral blood flow

A patient scheduled for magnetic resonance imaging (MRI) has arrived at the radiology department. The nurse who prepares the patient for the MRI should prioritize which of the following actions? A- withholding stimulants 24 to 48 hours prior to exam B- removing all metal-containing objects C- instructing the patient to void prior to the MRI D- initiating an IV line for administration of contrast

B- removing all metal-containing objects

A patient has been admitted to the critical care unit with a diagnosis of toxic hepatitis. When planning the patient's care, the nurse should be aware of what potential clinical course of this health problem? Place the following events in the correct sequence. 1. Fever rises 2. Hematemesis 3. Clotting abnormalities 4. Vascular collapse 5. Coma A. 1,2,5,4,3 B. 1,2,3,4,5 C. 2,3,1,4,5 D. 3,1,2,5,4

B. 1,2,3,4,5 Rationale: Recovery from acute toxic hepatitis is rapid if the hepatotoxin is identified early and removed or if exposure to the agent has been limited. Recovery is unlikely if there is a prolonged period between exposure and onset of symptoms. There are no effective antidotes. The fever rises; the patient becomes toxic and prostrated. Vomiting may be persistent, with the emesis containing blood. Clotting abnormalities may be severe, and hemorrhages may appear under the skin. The severe GI symptoms may lead to vascular collapse. Delirium, coma, and seizures develop, and within a few days the patient may die of fulminant hepatic failure unless he or she receives a liver transplant.

A patient with a history of injection drug use has been diagnosed with Hepatitis C. When collaborating with the care team to plan this patient's treatment, the nurse should anticipate what intervention? A. Administration of immune globulins B. A regimen of antiviral medications C. Rest and watchful waiting D. Administration of fresh-frozen plasma (FFP)

B. A regimen of antiviral medications Rationale: There is no benefit from rest, diet, or vitamin supplements in HCV treatment. Studies have demonstrated that a combination of two antiviral agents, Peg-interferon and ribavirin (Rebetol), is effective in producing improvement in patients with hepatitis C and in treating relapses. Immune globulins and FFP are not indicated.

A patient has been diagnosed with advanced stage breast cancer and will soon begin aggressive treatment. What assessment findings would most strongly suggest that the patient may have developed liver metastases? A. Persistent fever and cognitive changes B. Abdominal pain and hepatomegaly C. Peripheral edema unresponsive to diuresis D. Spontaneous bleeding and jaundice

B. Abdominal pain and hepatomegaly Rationale: The early manifestations of malignancy of the liver include pain, a continuous dull ache in the right upper quadrant, epigastrium, or back. Weight loss, loss of strength, anorexia, and anemia may also occur. The liver may be enlarged and irregular on palpation. Jaundice is present only if the larger bile ducts are occluded by the pressure of malignant nodules in the hilum of the liver. Fever, cognitive changes, peripheral edema, and bleeding are atypical signs.

A participant in a health fair has asked the nurse about the role of drugs in liver disease. What health promotion teaching has the most potential to prevent drug-induced hepatitis? A. Finish all prescribed courses of antibiotics, regardless of symptoms resolution. B. Adhere to dosing recommendations of OTC analgesics. C. Ensure that expired medications are disposed of safely. D. Ensure that pharmacists regularly review drug regimens for potential interactions.

B. Adhere to dosing recommendations of OTC analgesics. Rationale: Although any medication can affect liver function, use of acetaminophen (found in many OTC medications used to treat fever and pain) has been identified as the leading cause of acute liver failure. Finishing prescribed antibiotics and avoiding expired medications are unrelated to this disease. Drug interactions are rarely the cause of drug-induced hepatitis.

A nurse is caring for a patient who has been admitted for the treatment of advanced cirrhosis. What assessment should the nurse prioritize in this patient's plan of care? A. Measurement of abdominal girth and body weight B. Assessment for variceal bleeding C. Assessment for signs and symptoms of jaundice D. Monitoring of results of liver function testing

B. Assessment of variceal bleeding Rationale: Esophageal varices are a major cause of mortality in patients with uncompensated cirrhosis. Consequently, this should be a focus of the nurse's assessments and should be prioritized over the other listed assessments, even though each should be performed.

A patient with cirrhosis has experienced a progressive decline in his health; and liver transplantation is being considered by the interdisciplinary team. How will the patient's prioritization for receiving a donor liver be determined? A. By considering that patient's age and prognosis B. By objectively determining the patient's medical need C. By objectively assessing the patient's willingness to adhere to post-transplantation care D. By systematically ruling out alternative treatment options

B. By objectively determining the patient's medical need Rationale: The patient would undergo a classification of the degree of medical need through an objective determination known as the Model of End-Stage Liver Disease (MELD) classification, which stratifies the level of illness of those awaiting a liver transplant. This algorithm considers multiple variables, not solely age, prognosis, potential for adherence and the rejection of alternative options.

A patient with a diagnosis of esophageal varices has undergone endoscopy to gauge the progression of this complication of liver disease. Following the completion of this diagnostic test, what nursing intervention should the nurse perform? A. Keep patient NPO until the results of test are known B. Keep patient NPO until the patient's gag reflex returns C. Administer analgesia until post-procedure tenderness is relieved D. Give the patient a cold beverage to promote swallowing ability

B. Keep patient NPO until the patient's gag reflex returns Rationale: After the examination, fluids are not given until the patient's gag reflex returns. Lozenges and gargles may be used to relieve throat discomfort if the patient's physical condition and mental status permit. The result of the test is known immediately. Food and fluids are contraindicated until the gag reflex returns.

A nurse is caring for a patient with a blocked bile duct from a tumor. What manifestation of obstructive jaundice should the nurse anticipate? A. Watery, blood-streaked diarrhea B. Orange and foamy urine C. Increased abdominal girth D. Decreased cognition

B. Orange and foamy urine Rationale: If the bile duct is obstructed, the bile will be reabsorbed into the blood and carried throughout the entire body. It is excreted in the urine, which becomes deep orange and foamy. Bloody diarrhea, ascites, and cognitive changes are not associated with obstructive jaundice.

A nurse is caring for a patient with cirrhosis secondary to heavy alcohol use. The nurse's most recent assessment reveals subtle changes in the patient's cognition and behavior. What is the nurse's most appropriate response? A. Ensure that the patient's sodium intake does not exceed recommended levels B. Report this finding to the primary care provider due to the possibility of hepatic encephalopathy C. Inform the primary care provider that the patient should be assessed for alcoholic hepatitis. D. Implement interventions aimed at ensuring a calm and therapeutic care environment.

B. Report this finding to the primary care provider due to the possibility of hepatic encephalopathy Rationale: Monitoring is an essential nursing function to identify early deterioration in mental status. The nurse monitors the patient's mental status closely and reports changes so that treatment of encephalopathy can be initiated promptly. This change in status is likely unrelated to sodium intake and would not signal the onset of hepatitis. A supportive care environment is beneficial, but does not address the patient's physiologic deterioration.

A 35-year-old kidney transplant patient comes to the clinic exhibiting new skin lesions. The diagnosis is Kaposis sarcoma. The nurse caring for this patient recognizes that this is what type of Kaposis sarcoma? A) Classic B) AIDS-related C) Immunosuppression-related D) Endemic

C

A 56-year-old patient has come to the clinic for his routine eye examination and is told he needs bifocals. The patient asks the nurse what change in his eyes has caused his need for bifocals. How should the nurse respond? A) "You know, you are getting older now and we change as we get older." B) "The parts of our eyes age, just like the rest of us, and this is nothing to cause you to worry." C) "There is a gradual thickening of the lens of the eye and it can limit the eye's ability for accommodation." D) "The eye gets shorter, back to front, as we age and it changes how we see things."

C

A 6-month-old infant is brought to the ED by his parents for inconsolable crying and pulling at his right ear. When assessing this infant, the advanced practice nurse is aware that the tympanic membrane should be what color in a healthy ear? A) Yellowish-white B) Pink C) Gray D) Bluish-white

C

A client is prescribed intravenous antibiotic therapy that requires monitoring of peak and trough drug levels. the cient receives the drug at 8:30 am. at which time would the nurse anticipate that the peak level be drawn? A- 8:15 am B- 8:45 am C- 9:00 am D- 9:30 am

C

A clinic nurse is caring for a patient with a history of osteoporosis. Which of the following diagnostic tests best allows the care team to assess the patients risk of fracture? A) Arthrography B) Bone scan C) Bone densitometry D) Arthroscopy

C

A clinic patient has described recent dark-colored stools;the nurse recognizes the need for fecal occult blood testing (FOBT). What aspect of the patients current health status would contraindicate FOBT? A) Gastroesophageal reflux disease (GERD) B) Peptic ulcers C) Hemorrhoids D) Recurrent nausea and vomiting

C

A nurse is assessing the abdomen of a patient just admitted to the unit with a suspected GI disease. Inspection reveals several diverse lesions on the patients abdomen. How should the nurse best interpret this assessment finding? A) Abdominal lesions are usually due to age-related skin changes. B) Integumentary diseases often cause GI disorders. C) GI diseases often produce skin changes. D) The patient needs to be assessed for self-harm.

C

A nurse is caring for a patient who has a diagnosis of bullous pemphigoid and who is being treated on the medical unit. When providing hygiene for this patient, the nurse should perform which of the following actions? A) After washing, wipe lesions with sterile gauze to remove cellular debris. B) Apply antibiotic ointment to lesions after washing. C) Apply cornstarch to the patients skin after bathing to facilitate mobility. D) Avoid using water to cleanse the patients skin in order to maintain skin integrity.

C

A nurse is caring for a patient who has been scheduled for a bone scan. What should the nurse teach the patient about this diagnostic test? A) The test is brief and requires that you drink a calcium solution 2 hours before the test. B) You will not be allowed fluid for 2 hours before and 3 hours after the test. C) Youll be encouraged to drink water after the administration of the radioisotope injection. D) This is a common test that can be safely performed on anyone.

C

A nurse is caring for a patient with a diagnosis of cancer that has metastasized. What laboratory value would the nurse expect to be elevated in this patient? A) Bilirubin B) Potassium C) Alkaline phosphatase D) Creatinine

C

A nurse is leading a health promotion workshop that is focusing on cancer prevention. What action is most likely to reduce participants risks of basal cell carcinoma (BCC)?A) Teaching participants to improve their overall health through nutrition B) Encouraging participants to identify their family history of cancer C) Teaching participants to limit their sun exposure D) Teaching participants to control exposure to environmental and occupational radiation

C

A nurse is providing care for a patient who has psoriasis. The nurse is aware of the sequelae that can result from this health problem. Following the appearance of skin lesions, the nurse should prioritize what assessment? A) Assessment of the patients stool for evidence of intestinal sloughing B) Assessment of the patients apical heart rate for dysrhythmias C) Assessment of the patients joints for pain and decreased range of motion D) Assessment for cognitive changes resulting from neurologic lesions

C

A nurse is providing preprocedure education for a patient who will undergo a lower GI tract study the following week. What should the nurse teach the patient about bowel preparation? A) Youll need to fast for at least 18 hours prior to your test. B) Starting today, take over-the-counter stool softeners twice daily. C) Youll need to have enemas the day before the test. D) For 24 hours before the test, insert a glycerin suppository every 4 hours

C

A nurse is taking a health history on a new patient who has been experiencing unexplained paresthesia. What question should guide the nurses assessment of the patients altered sensations? A) How does the strength in the affected extremity compare to the strength in the unaffected extremity? B) Does the color in the affected extremity match the color in the unaffected extremity? C) How does the feeling in the affected extremity compare with the feeling in the unaffected extremity? D) Does the patient have a family history of paresthesia or other forms of altered sensation?

C

A patient asks the nursing assistant for a bedpan. When the patient is finished, the nursing assistant notifies the nurse that the patient has bright red streaking of blood in the stool. What is this most likely a result of? A) Diet high in red meat B) Upper GI bleed C) Hemorrhoids D) Use of iron supplements

C

A patient diagnosed with a stasis ulcer has been hospitalized. There is an order to change the dressing and provide wound care. Which activity should the nurse first perform when providing wound care? A) Assess the drainage in the dressing. B) Slowly remove the soiled dressing. C) Perform hand hygiene. D) Don non-latex gloves.

C

A patient has a diagnosis of seborrhea and has been referred to the dermatology clinic, where the nurse contributes to care. When planning this patients care, the nurse should include which of the following nursing diagnoses? A) Risk for Deficient Fluid Volume Related to Excess Sebum Synthesis B) Ineffective Thermoregulation Related to Occlusion of Sebaceous Glands C) Disturbed Body Image Related to Excess Sebum Production D) Ineffective Tissue Perfusion Related to Occlusion of Sebaceous Glands

C

A patient has been experiencing progressive increases in knee pain and diagnostic imaging reveals a worsening effusion in the synovial capsule. The nurse should anticipate which of the following? A) Arthrography B) Knee biopsy C) Arthrocentesis D) Electromyography

C

A patient has informed the home health nurse that she has recently noticed distortions when she looks at the Amsler grid that she has mounted on her refrigerator. What is the nurse's most appropriate action? A) Reassure the patient that this is an age-related change in vision. B) Arrange for the patient to have her visual acuity assessed. C) Arrange for the patient to be assessed for macular degeneration. D) Facilitate tonometry testing.

C

A patient has just arrived to the floor after an enucleation procedure following a workplace accident in which his left eye was irreparably damaged. Which of the following should the nurse prioritize during the patient's immediate postoperative recovery? A) Teaching the patient about options for eye prostheses B) Teaching the patient to estimate depth and distance with the use of one eye C) Assessing and addressing the patient's emotional needs D) Teaching the patient about his post-discharge medication regimen

C

A patient has just been told that he has malignant melanoma. The nurse caring for this patient should anticipate that the patient will undergo what treatment? A) Chemotherapy B) Immunotherapy C) Wide excision D) Radiation therapy

C

A patient is admitted to the intensive care unit with what is thought to be toxic epidermal necrolysis (TEN). When assessing the health history of the patient, the nurse would be alert to what precipitating factor? A) Recent heavy ultraviolet exposure B) Substandard hygienic conditions C) Recent administration of new medications D) Recent varicella infection

C

A patient is being discharged home after mastoid surgery. What topic should the nurse address in the patient's discharge education? A) Expected changes in facial nerve function B) The need for audiometry testing every 6 months following recovery C) Safe use of analgesics and antivertiginous agents D) Appropriate use of OTC ear drops

C

A patient who has been experiencing changes in his bowel function is scheduled for a barium enema. What instruction should the nurse provide for postprocedure recovery? A) Remain NPO for 6 hours postprocedure. B) Administer a Fleet enema to cleanse the bowel of the barium. C) Increase fluid intake to evacuate the barium. D) Avoid dairy products for 24 hours postprocedure.

C

A patient with low vision has called the clinic and asked the nurse for help with acquiring some low-vision aids. What else can the nurse offer to help this patient manage his low vision? A) The patient uses OTC NSAIDs. B) The patient has a history of stroke. C) The patient has diabetes. D) The patient has Asian ancestry.

C

A patient with squamous cell carcinoma has been scheduled for treatment of this malignancy. The nurse should anticipate that treatment for this type of cancer will primarily consist of what intervention? A) Chemotherapy B) Radiation therapy C) Surgical excision D) Biopsy of sample tissue

C

A public health nurse is participating in a health promotion campaign that has the goal of improving outcomes related to skin cancer in the community. What action has the greatest potential to achieve this goal? A) Educating participants about the relationship between general health and the risk of skin cancer B) Educating participants about treatment options for skin cancer C) Educating participants about the early signs and symptoms of skin cancer D) Educating participants about the health risks associated with smoking and assisting with smoking cessation

C

An adult patient is scheduled for an upper GI series that will use a barium swallow. What teaching should the nurse include when the patient has completed the test? A) Stool will be yellow for the first 24 hours post procedure. B) The barium may cause diarrhea for the next 24 hours. C) Fluids must be increased to facilitate the evacuation of the stool. D) Slight anal bleeding may be noted as the barium is passed.

C

An older adult patient has been diagnosed with macular degeneration and the nurse is assessing him for changes in visual acuity since his last clinic visit. When assessing the patient for recent changes in visual acuity, the patient states that he sees the lines on an Amsler grid as being distorted. What is the nurse's most appropriate response? A) Ask if the patient has been using OTC vasoconstrictors. B) Instruct the patient to repeat the test at different times of the day when at home. C) Arrange for the patient to visit his ophthalmologist. D) Encourage the patient to adhere to his prescribed drug regimen.

C

An older adult resident of a long-term care facility has been experiencing generalized pruritus that has become more severe in recent weeks. What intervention should the nurse add to this residents plan of care? A) Avoid the application of skin emollients. B) Apply antibiotic ointment as ordered following baths. C) Avoid using hot water during the patients baths. D) Administer acetaminophen 4 times daily as ordered.

C

Cytomegalovirus (CMV) is the most common cause of retinal inflammation in patients with AIDS. What drug, surgically implanted, is used for the acute stage of CMV retinitis? A) Pilocarpine B) Penicillin C) Ganciclovir D) Gentamicin

C

The human body is designed to protect its vital parts. A fracture of what type of bone may interfere with the protection of vital organs? A) Long bones B) Short bones C) Flat bones D) Irregular bones

C

The nurse is planning the care of a patient with a diagnosis of vertigo. What nursing diagnosis risk should the nurse prioritize in this patient's care? A) Risk for disturbed sensory perception B) Risk for unilateral neglect C) Risk for falls D) Risk for ineffective health maintenance

C

The nurse is providing health education to a patient scheduled for a colonoscopy. The nurse should explain that she will be placed in what position during this diagnostic test? A) In a knee-chest position (lithotomy position) B) Lying prone with legs drawn toward the chest C) Lying on the left side with legs drawn toward the chest D) In a prone position with two pillows elevating the buttocks

C

The nurse is teaching a patient to care for her new ocular prosthesis. What should the nurse emphasize during the patient's health education? A) The need to limit exposure to bright light B) The need to maintain a low Fowler's position when removing the prosthesis C) The need to perform thorough hand hygiene before handling the prosthesis D) The need to apply antiviral ointment to the prosthesis daily

C

The physiology instructor is discussing the GI system with the pre-nursing class. What should the instructor describe as a major function of the GI tract? A) The breakdown of food particles into cell form for digestion B) The maintenance of fluid and acid-base balance C) The absorption into the bloodstream of nutrient molecules produced by digestion D) The control of absorption and elimination of electrolytes

C

Which of the following nursing interventions would most likely facilitate effective communication with a hearing-impaired patient? A) Ask the patient to repeat what was said in order to evaluate understanding. B) Stand directly in front of the patient to facilitate lip reading. C) Reduce environmental noise and distractions before communicating. D) Raise the voice to project sound at a higher frequency.

C

You are caring for a patient who was admitted to have a low-profile gastrostomy device (LPGD) placed. How soon after the original gastrostomy tube placement can an LPGD be placed? A) 2 weeks B) 4 to 6 weeks C) 2 to 3 months D) 4 to 6 months

C Feedback: An alternative to the PEG device is a low-profile gastrostomy device (LPGD). LPGDs may be inserted 2 to 3 months after initial gastrostomy tube placement.

A nurse is providing care for a patient with a diagnosis of late-stage Alzheimer's disease. The patient has just returned to the medical unit to begin supplemental feedings through an NG tube. Which of the nurse's assessments addresses this patient's most significant potential complication of feeding? A) Frequent assessment of the patient's abdominal girth B) Assessment for hemorrhage from the nasal insertion site C) Frequent lung auscultation D) Vigilant monitoring of the frequency and character of bowel movements

C Feedback: Aspiration is a risk associated with tube feeding; this risk may be exacerbated by the patient's cognitive deficits. Consequently, the nurse should auscultate the patient's lungs and monitor oxygen saturation closely. Bowel function is important, but the risk for aspiration is a priority. Hemorrhage is highly unlikely and the patient's abdominal girth is not a main focus of assessment.

A patient receiving tube feedings is experiencing diarrhea. The nurse and the physician suspect that the patient is experiencing dumping syndrome. What intervention is most appropriate? A) Stop the tube feed and aspirate stomach contents. B) Increase the hourly feed rate so it finishes earlier. C) Dilute the concentration of the feeding solution. D) Administer fluid replacement by IV

C Feedback: Dumping syndrome can generally be alleviated by starting with a dilute solution and then increasing the concentration of the solution over several days. Fluid replacement may be necessary but does not prevent or treat dumping syndrome. There is no need to aspirate stomach contents. Increasing the rate will exacerbate the problem.

The management of the patient's gastrostomy is an assessment priority for the home care nurse. What statement would indicate that the patient is managing the tube correctly? A) "I clean my stoma twice a day with alcohol." B) "The only time I flush my tube is when I'm putting in medications." C) "I flush my tube with water before and after each of my medications." D) "I try to stay still most of the time to avoid dislodging my tube."

C Feedback: Frequent flushing is needed to prevent occlusion, and should not just be limited to times of medication administration. Alcohol will irritate skin surrounding the insertion site and activity should be maintained as much as possible.

A patient who suffered a stroke had an NG tube inserted to facilitate feeding shortly after admission. The patient has since become comatose and the patient's family asks the nurse why the physician is recommending the removal of the patient's NG tube and the insertion of a gastrostomy tube. What is the nurse's best response? A) It eliminates the risk for infection. B) Feeds can be infused at a faster rate. C) Regurgitation and aspiration are less likely. D) It allows caregivers to provide personal hygiene more easily.

C Feedback: Gastrostomy is preferred over NG feedings in the patient who is comatose because the gastroesophageal sphincter remains intact, making regurgitation and aspiration less likely than with NG feedings. Both tubes carry a risk for infection; this change in care is not motivated by the possibility of faster infusion or easier personal care.

A nurse is participating in a patient's care conference and the team is deciding between parenteral nutrition (PN) and a total nutritional admixture (TNA). What advantages are associated with providing TNA rather than PN? A) TNA can be mixed by a certified registered nurse. B) TNA can be administered over 8 hours, while PN requires 24-hour administration. C) TNA is less costly than PN. D) TNA does not require the use of a micron filter.

C Feedback: TNA is mixed in one container and administered to the patient over a 24-hour period. A 1.5-micron filter is used with the TNA solution. Advantages of the TNA over PN include cost savings. Pharmacy staff must prepare both solutions.

A critical care nurse is caring for a patient diagnosed with acute pancreatitis. The nurse knows that the indications for starting parenteral nutrition (PN) for this patient are what? A) 5% deficit in body weight compared to preillness weight and increased caloric need B) Calorie deficit and muscle wasting combined with low electrolyte levels C) Inability to take in adequate oral food or fluids within 7 days D) Significant risk of aspiration coupled with decreased level of consciousness

C Feedback: The indications for PN include an inability to ingest adequate oral food or fluids within 7 days. Weight loss, muscle wasting combined with electrolyte imbalances, and aspiration indicate a need for nutritional support, but this does not necessary have to be parenteral.

A nurse is assessing reflexes in a patient with hyperactive reflexes. When the patient's foot is abruptly dorsiflexed, it continues to "beat" two to three times before settling into a resting position. How should the nurse document this finding? A- rigidity B- flaccidity C- clonus D- ataxia

C- clonus

Prior to a patient's scheduled jejunostomy, the nurse is performing the preoperative assessment. What goal should the nurse prioritize during the preoperative assessment? A) Determining the patient's nutritional needs B) Determining that the patient fully understands the postoperative care required C) Determining the patient's ability to understand and cooperate with the procedure D) Determining the patient's ability to cope with an altered body image

C Feedback: The major focus of the preoperative assessment is to determine the patient's ability both to understand and cooperate with the procedure. Body image, nutritional needs, and postoperative care are all important variables, but they are not the main focuses of assessment during the immediate preoperative period.

The nurse is caring for a patient who is postoperative from having a gastrostomy tube placed. What should the nurse do on a daily basis to prevent skin breakdown? A) Verify tube placement. B) Loop adhesive tape around the tube and connect it securely to the abdomen. C) Gently rotate the tube. D) Change the wet-to-dry dressing.

C Feedback: The nurse verifies the tube's placement and gently rotates the tube once daily to prevent skin breakdown. Verifying tube placement and taping the tube to the abdomen do not prevent skin breakdown. A gastrostomy wound does not have a wet-to-dry dressing.

The nurse is assessing placement of a nasogastric tube that the patient has had in place for 2 days. The tube is draining green aspirate. What is the nurse's most appropriate action? A) Inform the physician that the tube may be in the patient's pleural space. B) Withdraw the tube 2 to 4 cm. C) Leave the tube in its present position. D) Advance the tube up to 8 cm.

C Feedback: The patient's aspirate is from the gastric area when the nurse observes that the color of the aspirate is green. Further confirmation of placement is necessary, but there is likely no need for repositioning. Pleural secretions are pale yellow.

The nurse caring for a patient with Cushing's syndrome is describing the dexamethasone suppression test scheduled for tomorrow. What does the nurse explain that this test will involve? A) Administration of dexamethasone orally, followed by a plasma cortisol level every hour for 3 hours B) Administration of dexamethasone IV, followed by an x-ray of the adrenal glands C) Administration of dexamethasone orally at 11PM, and a plasma cortisol level at 8AM the next morning D) Administration of dexamethasone intravenously, followed by a plasma cortisol level 3 hours after the drug is administered

C) Administration of dexamethasone orally at 11PM, and a plasma cortisol level at 8AM the next morning

The nurse is performing a comprehensive assessment of a patients skin surfaces and intends to assess moisture, temperature, and texture. The nurse should perform this component of assessment in what way? A) By examining the patient under a Woods light B) By inspecting the patients skin in direct sunlight C) By palpating the patients skin D) By performing percussion of major skin surfaces

C) By palpating the patients skin Inspection and palpation are techniques commonly used in examining the skin. A patient would only be examined under a Woods light if there were indications it could be diagnostic. The patient is examined in a well-lit room, not in direct sunlight. Percussion is not a technique used in assessing the skin.

A nurse is reviewing gerontologic considerations relating to the care of patients with dermatologic problems. What vulnerability results from the age-related loss of subcutaneous tissue? A) Decreased resistance to ultraviolet radiation B) Increased vulnerability to infection C) Diminished protection of tissues and organs D) Increased risk of skin malignancies

C) Diminished protection of tissues and organs Loss of the subcutaneous tissue substances of elastin, collagen, and fat diminishes the protection and cushioning of underlying tissues and organs, decreases muscle tone, and results in the loss of the insulating properties of fat. This age-related change does not correlate to an increased vulnerability to sun damage, infection, or cancer.

A 30-year-old female patient has been diagnosed with Cushing's syndrome. What psychosocial nursing diagnosis should the nurse most likely prioritize when planning the patient's care? A) Decisional conflict r/t treatment options B) Spiritual distress r/t changes in cognitive function C) Disturbed body image r/t changes in physical appearance D) Powerlessness r/t disease progression

C) Disturbed body image r/t changes in cognitive function

A nurse caring for a patient with diabetes insipidus is reviewing laboratory results. What is an expected uninalysis finding? A) Glucose in the urine B) Albumin in the urine C) Highly dilute urine D) Leukocytes in the urine

C) Highly dilute urine

A patient has returned to the floor after having a thyroidectomy for thyroid cancer. The nurse knows that sometimes during the surgery the parathyroid glands can be injured or removed. What laboratory finding may be an early indication of parathyroid gland injury or removal? A) Hyponatremia B) Hypophosphatemia C) Hypocalcemia D) Hypokalemia

C) Hypocalcemia

The nurse is caring for a patient with a diagnosis of Addison's disease. What sign or symptom is most closely associated with this health problem? A) Truncal obesity B) Hypertension C) Muscle weakness D) Moon face

C) Muscle weakness

A nurse is preparing to perform the physical assessment of a newly admitted patient. During which of the following components of the assessment should the nurse wear gloves? Select all that apply. A) Palpation of the patients scalp B) Palpation of the patients upper extremities C) Palpation of a rash on the patients trunk D) Palpation of a lesion on the patients upper back E) Palpation of the patients fingers

C) Palpation of a rash on the patients trunk D) Palpation of a lesion on the patients upper back oves are worn during skin examination if a rash or lesions are to be palpated. It is not normally necessary to wear gloves to palpate a patients scalp, extremities, or fingers unless contact with body fluids is reasonably foreseeable.

A new patient has come to the dermatology clinic to be assessed for a reddened rash on his abdomen. What diagnostic test would most likely be ordered to identify the causative allergen? A) Skin scrapings B) Skin biopsy C) Patch testing D) Tzanck smear

C) Patch testing Patch testing is performed to identify substances to which the patient has developed an allergy. Skin scrapings are done for suspected fungal lesions. A skin biopsy is completed to rule out malignancy and to establish an exact diagnosis of skin lesions. A Tzanck smear is used to examine cells from blistering skin conditions, such as herpes zoster.

A nurse in a dermatology clinic is reading the electronic health record of a new patient. The nurse notes that the patient has a history of a primary skin lesion. What is an example of a primary skin lesion? A) Crust B) Keloid C) Pustule D) Ulcer

C) Pustule A pustule is an example of a primary skin lesion. Primary skin lesions are original lesions arising from previously normal skin. Crusts, keloids and ulcers are secondary lesions.

A patient has been admitted to the post-surgical unit following a thyroidectomy. To promote comfort and safety, how should the nurse best position the patient? A) Side-lying (lateral) with one pillow under the head B) Head of the bed elevated 30 degrees and no pillows placed under the head C) Semi-Fowler's with the head supported on two pillows D) Supine, with a small roll supporting the neck

C) Semi-Fowler's with the head supported on two pillows

The nurse is teaching a patient that the body needs iodine for the thyroid to function. What food would be the best source of iodine for the body? A) Eggs B) Shellfish C) Table salt D) Red meat

C) Table salt

When planning the skin care of a patient with decreased mobility, the nurse is aware of the varying thickness of the epidermis. At what location is the epidermal layer thickest? A) The scalp B) The elbows C) The palms of the hands D) The knees

C) The palms of the hands The epidermis is the thickest over the palms of the hands and the soles of the feet.

A patient is prescribed corticosteriod therapy. What would be priority information for the nurse to give the patient who is prescribed long-term corticosteroid therapy? A) The patient's diet should be low protein with ample fat. B) The patient may experience short-term changes in cognition. C) The patient is at an increased risk for developing infection. D) The patient is at a decreased risk for development of thrombophelbitis and thromboembolism.

C) The patient is at an increased risk for developing infection.

A patient has experienced burns to his upper thighs and knees. Following the application of new wound dressings, the nurse should perform what nursing action? A) instruct the patient to keep the wound site in a dependent position B) administer PRN analgesia as ordered C) assess the patients peripheral pulses distal to the dressing D) assist with passive range of motion exercises to "set" the new dressing

C) assess the patients peripheral pulses distal to the dressing

A patient with severe burns is admitted to the intensive care unit to stabilize and begin fluid resuscitation before transport to the burn center. The nurse should monitor the patient closely for what signs of the onset of burn shock? A) confusion B) high fever C) decreased blood pressure D) sudden agitation

C) decreased blood pressure

A patient has been assessed for aldosteronism and has recently begun treatment. What are priority areas for assessment that the nurse should frequently address? Select all that apply. A) Pupillary response B) Creatine and BUN levels C) Potassium level D) Peripheral pulses E) BP

C) potassium level E) BP

A patient experienced a 33% TBSA burn 72 hours ago. The nurse observes that the patient's hourly urine output has been steadily increasing over the past 24 hours. How should the nurse best respond to this finding? A) obtain an order to reduce the rate of the patient's IV fluid infusion B) report the patients early signs of acute kidney injury (AKI) C) recognize that the patient is experiencing an expected onset of diuresis D) administer sodium chloride as ordered to compensate for this fluid loss

C) recognize that the patient is experiencing an expected onset of diuresis

A 28-year-old pregnant woman is spilling sugar in her urine. The physician orders a glucose tolerance test, which reveals gestational diabetes. The patient is shocked by the diagnosis, stating that she is conscientious about her health, and ask the nurse what causes gestational diabetes. The nurse should explain that gestational diabetes is a result of what etiologic factor? A) increased caloric intake during the first trimester B) changes in osmolality and fluid balance C) the effects of hormonal changes during pregnancy D) overconsumption of carbohydrates during the first two trimesters

C) the effects of hormonal changes during pregnancy

The nurse is caring for a patient with a duodenal ulcer and is relating the patients symptoms to the physiologic functions of the small intestine. What do these functions include? Select all that apply. A) Secretion of hydrochloric acid (HCl) B) Reabsorption of water C) Secretion of mucus D) Absorption of nutrients E) Movement of nutrients into the bloodstream

C,D,E

An older adult with a recent history of mixed hearing loss has been diagnosed with a cholesteatoma. What should this patient be taught about this diagnosis? Select all that apply A) Cholesteatomas are benign and self-limiting, and hearing loss will resolve spontaneously. B) Cholesteatomas are usually the result of metastasis from a distant tumor site. C) Cholesteatomas are often the result of chronic otitis media. D) Cholesteatomas, if left untreated, result in intractable neuropathic pain. E) Cholesteatomas usually must be removed surgically

C,E

A patient is scheduled for CT scanning of the head because of a recent onset of neurologic deficits. What should the nurse tell the patient in preparation for this test? A- "No metal objects can enter the procedure room" B- "You need to fast for 8 hours prior to the test" C- "You will need to lie still throughout the procedure" D- "There will be a lot of noise during the test"

C- "You will need to lie still throughout the procedure"

The nurse caring for an 80 year old patient knows that she has a pre-existing history of dulled tactile sensation. The nurse should first consider what possible cause for this patients diminished tactile sensation? A- damage to cranial nerve VIII B- adverse medication effects C- age related neurologic changes D- an undiagnosed cerebrovascular accident in early adulthood

C- age related neurologic changes

A patient in the OR goes into malignant hyperthermia due to an abnormal reaction to the anesthetic. The nurse knows that the area of the brain that regulates body temperature is which of the following? A- cerebellum B- thalamus C- hypothalamus D- midbrain

C- hypothalamus

A patient is having a "fight or flight response" after receiving bad news about his prognosis. What affect will this have on the patients sympathetic nervous system? A- constriction of blood vessels in the heart muscle B- constriction of bronchioles C- increase in the secretion of sweat D- constriction of pupils

C- increase in the secretion of sweat

When caring for a patient with an altered level of consciousness, the nurse is preparing to test cranial nerve VII. What assessment technique would the nurse use to elicit a response from cranial nerve VII. A- palpate trapezius muscle while patient shrugs should against resistance B- administer the whisper or watch tick test C- observe for facial movement symmetry such as a smile D- not any hoarseness in the patient voice

C- observe for facial movement symmetry, such as a smile

A patient had a lumbar puncture performed at the outpatient clinic and the nurse has phoned the patient and family that evening. What does this phone call enable the nurse to determine? A- what are the patient and family's expectations of the test B- whether the patients family had any questions about why the test was necessary C- whether the patient has had any complications of the test D- whether the patient understood accurately why the test was done

C- whether the patient has had any complications of the test

A nurse is caring for a patient with severe hemolytic jaundice. Laboratory tests show free bilirubin to be 24 mg/dL. For what complication is this patient at risk? A. Chronic jaundice B. Pigment stones in portal circulation C. Central nervous system damage D. Hepatomegaly

C. Central nervous system damage Rationale: Prolonged jaundice, even if mild, predisposes to the formation of pigment stones in the gallbladder, and extremely severe jaundice (levels of free bilirubin exceeding 20 to 25 mg/dL) poses a risk for CNS damage. There are not specific risks of hepatomegaly or chronic jaundice resulting from high bilirubin

A patient with portal hypertension has been admitted to the medical floor. The nurse should prioritize which of the following assessments related to the manifestations of this health problem? A. Assessment of blood pressure and assessment for headaches and visual changes. B. Assessments for signs and symptoms of venous thromboembolism. C. Daily weights and abdominal girth measurement. D. Blood glucose monitoring q4h.

C. Daily weights and abdominal girth measurement Rationale: Obstruction to blood flow through the damaged liver results in increased blood pressure (portal hypertension) throughout the portal venous system. This can result in varices and ascites in the abdominal cavity. Assessments related to ascites are daily weights and abdominal girths. Portal hypertension is not synonymous with cardiovascular hypertension and does not create a risk for unstable blood glucose of VTE.

A patient with a diagnosis of cirrhosis has developed variceal bleeding and will imminently undergo variceal banding. What psychosocial nursing diagnosis should the nurse most likely prioritize during this phase of the patient's treatment? A. Decisional Conflict B. Deficient Knowledge C. Death Anxiety D. Disturbed Thought Processes

C. Death Anxiety Rationale: The sudden hemorrhage that accompanies variceal bleeding is intensely anxiety-provoking. The nurse must address the patient's likely fear of death, which is a realistic possibility. For most patients, anxiety is likely to be a more acute concern than lack of knowledge or decisional conflict. The patient may or may not experience disturbances in thought processes.

A nurse has entered the room of a patient with cirrhosis and found the patient on the floor. The patient stated that she fell when transferring to the commode. The patient's vital signs are within reference ranges and the nurse observes no apparent injuries. What is the nurse's most appropriate action? A. Remove the patient's commode and supply a bedpan B. Complete an incident report and submit it to the unit supervisor C. Have the patient assessed by the physician due to the risk of internal bleeding D. Perform a focused abdominal assessment in order to rule out injury.

C. Have the patient assessed by the physician due to the risk of internal bleeding Rationale: A fall would necessitate thorough medical assessment due to the patient's risk of bleeding. The nurse's abdominal assessment is an appropriate action, but is not wholly sufficient to rule out internal injury. Medical assessment is a priority over removing the commode or filling out an incident report, even though these actions are appropriate.

A previously healthy adult's sudden and precipitous decline in health has been attributed to fulminant hepatic failure, and the patient has been admitted to the intensive care unit. The nurse should be aware that the treatment of choice for this patient is what? A. IV administration of immune globulins B. Transfusion of packed red blood cells and fresh-frozen plasma (FFP) C. Liver transplantation D. Lobectomy

C. Liver transplantation Rationale: Liver transplantation carries the highest potential for the resolution of fulminant hepatic failure. This is preferred over other interventions, such as pharmacologic treatments, transfusions, and surgery

Diagnostic testing has revealed that a patient's hepatocellular carcinoma (HCC) is limited to one lobe. The nurse should anticipate that this patient's plan of care will focus on what intervention? A. Cryosurgery B. Liver transplantation C. Lobectomy D. Laser hyperthermia

C. Lobectomy Rationale: Surgical resection is the treatment of choice when HCC is confined to one lobe of the liver and the function of the remaining liver is considered adequate for postoperative recovery. Removal of a lobe of the liver (lobectomy) is the most common surgical procedure for excising a liver tumor. While cryosurgery and liver transplantation are other surgical options for management of liver cancer, these procedures are not performed at the same frequency as a lobectomy. Laser hyperthermia is a nonsurgical treatment for liver cancer.

A nurse is performing an admission assessment of a patient with a diagnosis of cirrhosis. What technique should the nurse use to palpate the patient's liver? A. Place hand under the right lower abdominal quadrant and press down lightly with the other hand. B. Place the left hand over the abdomen and behind the left side at the 11th rib. C. Place hand under right lower rib cage and press down lightly with the other hand. D. Hold hand 90 degree to right side of the abdomen and push down firmly.

C. Place hand under right lower rib cage and press down lightly with the other hand. Rationale: To palpate the liver, the examiner places one hand under the right lower rib cage and presses downward with light pressure with the other hand. The liver is not on the left side or in the right lower abdominal quadrant.

A nurse is caring for a patient with hepatic encephalopathy. The nurse's assessment reveals that the patient exhibits episodes of confusion, is difficult to arouse from sleep and has rigid extremities. Based on these clinical findings, the nurse should document what stage of hepatic encephalopathy? A. Stage 1 B. Stage 2 C. Stage 3 D. Stage 4

C. Stage 3 Rationale: Patients in the third stage of hepatic encephalopathy exhibit the following symptoms: stuporous, difficult to arouse, sleeps most of the time, exhibits marked confusion, incoherent in speech, asterixis, increased deep tendon reflexes, rigidity of extremities, marked EEG abnormalities. Patients in stages 1 and 2 exhibit clinical symptoms that are not as advanced as found in stage 3, and patients in stage 4 are comatose. In stage 4, there is an absence of asterixis, absence of deep tendon reflexes, flaccidity of extremities, and EEG abnormalities.

A patient who has undergone liver transplantation is ready to be discharged home. Which outcome of health education should the nurse prioritize? A. The patient will obtain measurement of drainage from the T-tube. B. The patient will exercise three times a week. C. The patient will take immunosuppressive agents as required. D. The patient will monitor for signs of liver dysfunction.

C. The patient will take immunosuppressive agents as required. Rationale: The patient is given written and verbal instructions about immunosuppressive agent doses and dosing schedules. The patient is also instructed on steps to follow to ensure that an adequate supply of medication is available so that there is no change of running out of the medication or skipping a dose. Failure to take medications as instructed may precipitate rejection. The nurse would not teach the patient to measure drainage from a T-tube as the patient would;t go home with a T-tube. The nurse may teach the patient about the need to exercise or what the signs of liver dysfunction are, but the nurse would not stress these topics over the immunosuppressive drug regimen.

A 65-year-old man presents at the clinic complaining of nodules on both legs. The man tells the nurse that his son, who is in medical school, encouraged him to seek prompt care and told him that the nodules are related to the fact that he is Jewish. What health problem should the nurse suspect? A) Stasis ulcers B) Bullous pemphigoid C) Psoriasis D) Classic Kaposis sarcoma

D

A nurse is caring for a patient with recurrent hematemesis who is scheduled for upper gastrointestinal fibroscopy (UGF). How should the nurse in the radiology department prepare this patient? A) Insert a nasogastric tube. B) Administer a micro Fleet enema at least 3 hours before the procedure. C) Have the patient lie in a supine position for the procedure. D) Apply local anesthetic to the back of the patients throat.

D

A nurse is performing a musculoskeletal assessment of a patient with arthritis. During passive range-of- motion exercises, the nurse hears an audible grating sound. The nurse should document the presence of which of the following? A) Fasciculations B) Clonus C) Effusion D) Crepitus

D

A nurse is performing an abdominal assessment of an older adult patient. When collecting and analyzing data, the nurse should be cognizant of what age-related change in gastrointestinal structure and function? A) Increased gastric motility B) Decreased gastric pH C) Increased gag reflex D) Decreased mucus secretion

D

A patient is being assessed for a suspected deficit in intrinsic factor synthesis. What diagnostic or assessment finding is the most likely rationale for this examination of intrinsic factor production? A) Muscle wasting B) Chronic jaundice in the absence of liver disease C) The presence of fat in the patients stool D) Persistently low hemoglobin and hematocrit

D

A patient is being discharged home from the ambulatory surgical center after cataract surgery. In reviewing the discharge instructions with the patient, the nurse instructs the patient to immediately call the office if the patient experiences what? A) Slight morning discharge from the eye B) Any appearance of redness of the eye C) A "scratchy" feeling in the eye D) A new floater in vision

D

A patient is exploring treatment options after being diagnosed with age-related cataracts that affect her vision. What treatment is most likely to be used in this patient's care? A) Antioxidant supplements, vitamin C and E, beta-carotene, and selenium B) Eyeglasses or magnifying lenses C) Corticosteroid eye drops D) Surgical intervention

D

A patient is ready to be discharged home after a cataract extraction with intraocular lens implant and the nurse is reviewing signs and symptoms that need to be reported to the ophthalmologist immediately. Which of the patient's statements best demonstrates an adequate understanding? A) "I need to call the doctor if I get nauseated." B) "I need to call the doctor if I have a light morning discharge." C) "I need to call the doctor if I get a scratchy feeling." D) "I need to call the doctor if I see flashing lights."

D

A patient with a diagnosis of retinal detachment has undergone a vitreoretinal procedure on an outpatient basis. What subject should the nurse prioritize during discharge education? A) Risk factors for postoperative cytomegalovirus (CMV) B) Compensating for vision loss for the next several weeks C) Non-pharmacologic pain management strategies D) Signs and symptoms of increased intraocular pressure

D

A patient with a recent history of intermittent bleeding is undergoing capsule endoscopy to determine the source of the bleeding. When explaining this diagnostic test to the patient, what advantage should the nurse describe? A) The test allows visualization of the entire peritoneal cavity. B) The test allows for painless biopsy collection. C) The test does not require fasting. D) The test is noninvasive

D

A patient with a sudden onset of hearing loss tells the nurse that he would like to begin using hearing aids. The nurse understands that the health professional dispensing hearing aids would have what responsibility? A) Test the patient's hearing promptly. B) Perform an otoscopy. C) Measure the width of the patient's ear canal. D) Refer the patient to his primary care physician.

D

A patient with chronic open-angle glaucoma is being taught to self-administer pilocarpine. After the patient administers the pilocarpine, the patient states that her vision is blurred. Which nursing action is most appropriate? A) Holding the next dose and notifying the physician B) Treating the patient for an allergic reaction C) Suggesting that the patient put on her glasses D) Explaining that this is an expected adverse effect

D

A patient with cystic fibrosis takes pancreatic enzyme replacements on a regular basis. The patients intake of trypsin facilitates what aspect of GI function? A) Vitamin D synthesis B) Digestion of fats C) Maintenance of peristalsis D) Digestion of proteins

D

An inpatient has returned to the medical unit after a barium enema. When assessing the patients subsequent bowel patterns and stools, what finding should the nurse report to the physician? A) Large, wide stools B) Milky white stools C) Three stools during an 8-hour period of time D) Streaks of blood present in the stool

D

An older adult patient has come to the clinic for a regular check-up. The nurses initial inspection reveals an increased thoracic curvature of the patients spine. The nurse should document the presence of which of the following? A) Scoliosis B) Epiphyses C) Lordosis D) Kyphosis

D

On otoscopy, a red blemish behind the tympanic membrane is suggestive of what diagnosis? A) Acoustic tumor B) Cholesteatoma C) Facial nerve neuroma D) Glomus tympanicum

D

Probably the most widely used in-office or at-home occult blood test is the Hemoccult II. The patient has come to the clinic because he thinks there is blood in his stool. When you reviewed his medications, you noted he is on antihypertensive drugs and NSAIDs for early arthritic pain. You are sending the patient home with the supplies necessary to perform 2 hemoccult tests on his stool and mail the samples back to the clinic. What instruction would you give this patient? A) Take all your medications as usual. B) Take all your medications except the antihypertensive medications. C) Dont eat highly acidic foods 72 hours before you start the test. D) Avoid vitamin C for 72 hours before you start the test.

D

Several residents of a long-term care facility have developed signs and symptoms of viral conjunctivitis. What is the most appropriate action of the nurse who oversees care in the facility? A) Arrange for the administration of prophylactic antibiotics to unaffected residents. B) Instill normal saline into the eyes of affected residents two to three times daily. C) Swab the conjunctiva of unaffected residents for culture and sensitivity testing. D) Isolate affected residents from residents who have not developed conjunctivitis.

D

The nurse is administering eye drops to a patient with glaucoma. After instilling the patient's first medication, how long should the nurse wait before instilling the patient's second medication into the same eye? A) 30 seconds B) 1 minute C) 3 minutes D) 5 minutes

D

The nurse is caring for a patient who has a diagnosis of AIDS. Inspection of the patients mouth reveals the new presence of white lesions on the patients oral mucosa. What is the nurses most appropriate response? A) Encourage the patient to gargle with salt water twice daily. B) Attempt to remove the lesions with a tongue depressor. C) Make a referral to the units dietitian. D) Inform the primary care provider of this finding.

D

The nurse is discharging a patient home after mastoid surgery. What should the nurse include in discharge teaching? A) "Try to induce a sneeze every 4 hours to equalize pressure." B) "Be sure to exercise to reduce fatigue." C) "Avoid sleeping in a side-lying position." D) "Don't blow your nose for 2 to 3 weeks."

D

The public health nurse is addressing eye health and vision protection during an educational event. What statement by a participant best demonstrates an understanding of threats to vision? A) "I'm planning to avoid exposure to direct sunlight on my next vacation." B) "I've never exercised regularly, but I'm going to start working out at the gym daily." C) "I'm planning to talk with my pharmacist to review my current medications." D) "I'm certainly going to keep a close eye on my blood pressure from now on."

D

While performing an initial assessment of a patient admitted with appendicitis, the nurse observes an elevated blue-black lesion on the patients ear. The nurse knows that this lesion is consistent with what type of skin cancer? A) Basal cell carcinoma B) Squamous cell carcinoma C) Dermatofibroma D) Malignant melanoma

D

A patient with dysphagia is scheduled for PEG tube insertion and asks the nurse how the tube will stay in place. What is the nurse's best response? A) Adhesive holds a flange in place against the abdominal skin. B) A stitch holds the tube in place externally. C) The tube is stitched to the abdominal skin externally and the stomach wall internally. D) An internal retention disc secures the tube against the stomach wall.

D Feedback: A PEG tube is held in place by an internal retention disc (flange) that holds it against the stomach wall. It is not held in place by stitches or adhesives.

A patient has been brought to the emergency department by EMS after telling a family member that he deliberately took an overdose of NSAIDs a few minutes earlier. If lavage is ordered, the nurse should prepare to assist with the insertion of what type of tube? A) Nasogastric tube B) Levin tube C) Gastric sump D) Orogastric tube

D Feedback: An orogastric tube is a large-bore tube inserted through the mouth with a wide outlet for removal of gastric contents; it is used primarily in the emergency department or an intensive care setting. Nasogastric, Levin, and gastric sump tubes are not used for this specific purpose.

A nurse is preparing to administer a patient's scheduled parenteral nutrition (PN). Upon inspecting the bag, the nurse notices that the presence of small amounts of white precipitate are present in the bag. What is the nurse's best action? A) Recognize this as an expected finding. B) Place the bag in a warm environment for 30 minutes. C) Shake the bag vigorously for 10 to 20 seconds. D) Contact the pharmacy to obtain a new bag of PN.

D Feedback: Before PN infusion is administered, the solution must be inspected for separation, oily appearance (also known as a "cracked solution"), or any precipitate (which appears as white crystals). If any of these are present, it is not used. Warming or shaking the bag is inappropriate and unsafe.

A nurse has obtained an order to remove a patient's NG tube and has prepared the patient accordingly. After flushing the tube and removing the nasal tape, the nurse attempts removal but is met with resistance. Because the nurse is unable to overcome this resistance, what is the most appropriate action? A) Gently twist the tube before pulling. B) Instill a digestive enzyme solution and reattempt removal in 10 to 15 minutes. C) Flush the tube with hot tap water and reattempt removal. D) Report this finding to the patient's primary care provider.

D Feedback: If the tube does not come out easily, force should not be used, and the problem should be reported to the primary provider. Enzymes are used to resolve obstructions, not to aid removal. For safety reasons, hot water is never instilled into a tube. Twisting could cause damage to the mucosa.

A nurse is caring for a patient with a nasogastric tube for feeding. During shift assessment, the nurse auscultates a new onset of bilateral lung crackles and notes a respiratory rate of 30 breaths per minute. The patient's oxygen saturation is 89% by pulse oximetry. After ensuring the patient's immediate safety, what is the nurse's most appropriate action? A) Perform chest physiotherapy. B) Reduce the height of the patient's bed and remove the NG tube. C) Liaise with the dietitian to obtain a feeding solution with lower osmolarity. D) Report possible signs of aspiration pneumonia to the primary care provider.

D Feedback: The patient should be assessed for further signs of aspiration pneumonia. It is unnecessary to remove the NG tube and chest physiotherapy is not indicated. A different feeding solution will not resolve this complication.

A nurse is creating a care plan for a patient with a nasogastric tube. How should the nurse direct other members of the care team to check correct placement of the tube? A) Auscultate the patient's abdomen after injecting air through the tube. B) Assess the color and pH of aspirate. C) Locate the marking made after the initial x-ray confirming placement. D) Use a combination of at least two accepted methods for confirming placement.

D Feedback: There are a variety of methods to check tube placement. The safest way to confirm placement is to utilize a combination of assessment methods.

The nurse is caring for a patient with hyperparathyroidism. What level of activity would the nurse expect to promote? A) Complete bed rest B) Bed rest with bathroom privileges C) Out of bed (OOB) to the chair twice a day D) Ambulation and activity as tolerated

D) Ambulation and activity as tolerated

A patients health assessment has resulted in a diagnosis of alopecia areata. What nursing diagnosis should the nurse most likely associate with this health problem? A) Chronic Pain B) Impaired Skin Integrity C) Impaired Tissue Integrity D) Disturbed Body Image

D) Disturbed Body Image Alopecia areata causes hair loss in smaller defined areas. As such, it is common for the patient to experience a disturbed body image. Hair loss does not cause pain and does not affect skin or tissue integrity.

A patient with pheochromocytoma has been admitted for an adrenalectomy to be performed the following day. To prevent complications, the nurse should anticipate preoperative administration of which of the following? A) IV antibiotics B) Oral antihypertensives C) Parenteral nutrition D) IV corticosteroids

D) IV corticosteroids

The home care nurse is conducting patient teaching with a patient on corticosteroid therapy. To achieve consistency with the body's natural secretion of cortisol, when would the home care nurse instruct the patient to take his or her corticosteroids? A) Un the evening between 4PM and 6PM B) Prior to going to sleep at night C) At noon every day D) In the morning between 7AM and 8AM

D) In the morning between 7AM and 8AM

An 82-year-old patient is being treated in the hospital for a sacral pressure ulcer. What age-related change is most likely to affect the patients course of treatment? A) Increased thickness of the subcutaneous skin layer B) Increased vascular supply to superficial skin layers C) Changes in the character and quantity of bacterial skin flora D) Increased time required for wound healing

D) Increased time required for wound healing Wound healing becomes slower with age, requiring more time for older adults to recover from surgical and traumatic wounds. There are no changes in skin flora with increased age. Vascular supply and skin thickness both decrease with age.

A nurse is providing an educational presentation addressing the topic of Protecting Your Skin. When discussing the anatomy of the skin with this group, the nurse should know that what cells are responsible for producing the pigmentation of the skin? A) Islets of Langerhans B) Squamous cells C) T cells D) Melanocytes

D) Melanocytes Melanocytes are the special cells of the epidermis that are primarily responsible for producing the pigment melanin. Islets of Langerhans are clusters of cells in the pancreas. Squamous cells are flat, scaly epithelial cells. T cells function in the immune response.

A nurse is aware that the outer layer of the skin consists of dead cells that contain large amounts of keratin. The physiologic functions of keratin include which of the following? Select all that apply. A) Producing antibodies B) Absorbing electrolytes C) Maintaining acid-base balance D) Physically repelling pathogens E) Preventing fluid loss

D) Physically repelling pathogens E) Preventing fluid loss The dead cells of the epidermis contain large amounts of keratin, an insoluble, fibrous protein that forms the outer barrier of the skin. Keratin has the capacity to repel pathogens and prevent excessive fluid loss from the body. It does not contribute directly to antibody production, acidbase balance, or electrolyte levels.

An African American is admitted to the medical unit with liver disease. To correctly assess this patient for jaundice, on what body area should the nurse look for yellow discoloration? A) Elbows B) Lips C) Nail beds D) Sclerae

D) Sclerae Jaundice, a yellowing of the skin, is directly related to elevations in serum bilirubin and is often first observed in the sclerae and mucous membranes.

A patient has been taking prednisone for several weeks after experiencing a hypersensitivity reaction. To prevent adrenal insufficiency, the nurse should ensure that the patient knows to do which of the following? A) Take the drug concurrent with levothyroxine (Synthroid) B) Take each dose of prednisone with a dose of calcium C) Gradually replace the prednisone with an OTC alternative D) Slowly taper down the dose of prednisone, as ordered

D) Slowly taper down the dose of prednisone, as ordered

A patient on corticosteroid therapy needs to be taught that a course of corticosteroids of 2 weeks' duration can suppress the adrenal cortex for how long? A) Up to 4 weeks B) Up to 3 months C) Up to 9 months D) Up to 1 year

D) Up to 1 year

While assessing a dark-skinned patient at the clinic, the nurse notes the presence of patchy, milky white spots. The nurse knows that this finding is characteristic of what diagnosis? A) Cyanosis B) Addisons disease C) Polycythemia D) Vitiligo

D) Vitiligo With cyanosis, nail beds are dusky. With polycythemia, the nurse notes ruddy blue face, oral mucosa, and conjunctiva. A bronzed appearance, or external tan, is associated with Addisons disease. Vitiligo is condition characterized by destruction of the melanocytes in circumscribed areas of skin and appears in light or dark skin as patchy, milky white spots, often symmetric bilaterally.

A patient with hypo-function of the adrenal cortex has been admitted to the medical unit. What would the nurse likely find when assessing this patient? A) increased body temperature B) jaundice C) copious urine output D) decreased BP

D) decreased BP

A patient's burns are estimated at 36% of total body surface area; fluid resuscitation has been ordered in the emergency department. After establishing intravenous access, the nurse should anticipate the administration of what fluid? A) 0.45% NaCl with 20 mEq/L KCl B) 0.45% NaCl with 40 mEq/L KCl C) Normal saline D) Lactated Ringer's

D) lactated ringers

The neurologic nurse is testing the function of a patients cerebellum and basal ganglia. What action will most accurately test these structures? A- have the patient identify the location of a cotton swab on his or her skin with the eyes closed B- elicit the patients response to a hypothetical problem C- ask the patient to close his or her eyes and discern between hot and colt stimuli D- guide the patient through the performance of rapid, alternating movements

D- guide the patient through the performance of rapid, alternating movements

A nurse is caring for a patient diagnosed with Meniere's disease. While completing a neurologic examination on the patient, the nurse assesses cranial nerve VII. The nurse would be correct in identifying the function of this nerve as what? A- movement of the tongue B- visual acuity C- sense of smell D- hearing and equilibrium

D- hearing and equilibrium

The nursing students are learning how to assess function of cranial nerve VIII. To assess the function of cranial nerve VIII the students would be correct in completing which of the following assessment techniques? A- have the patient identify familiar odors with the eyes closed B- assess papillary reflex C- utilize the snellen chart D- test for air and bone conduction (Rinne test)

D- test for air and bone conduction (Rinne test)

A nurse is performing an admission assessment for an 81-year-old patient who generally enjoys good health. When considering normal, age-related changes to hepatic function, the nurse should anticipate what finding? A. Similar liver size and texture as in younger adults B. A nonpalpable liver C. A slightly enlarged liver with palpably hard edges D. A slightly decreased size of the liver

D. A slightly decreased size of the liver Rationale: The most common age-related change in the liver is a decrease in size and weight. The liver is usually still palpable, however, it is not expected to have hardened edges.

A nurse is amending a patient's plan of care in light of the fact that the patient has recently developed ascites. What should the nurse include in this patient's care plan? A. Mobilization with assistance at least 4 times daily B. Administration of beta-adrenergic blockers as ordered C. Vitamin B12 injections as ordered D. Administration of diuretics as ordered

D. Administration of diuretics as ordered Rationale: Use of diuretics along with sodium restriction is successful in 90% of patients with ascites. Beta-blockers are not used to treat ascites and bed rest is often more beneficial than increased mobility. Vitamin B23 injections are not necessary.

A local public health nurse is informed that a cook in a local restaurant has been diagnosed with hepatitis A. What should the nurse advise individuals to obtain who ate at this restaurant and have never received the hepatitis A vaccine? A. The hepatitis A vaccine B. Albumin infusion C. The hepatitis A and B vaccines D. An immune globulin injection

D. An immune globulin injection Rationale: For people who have not been previously vaccinated, hepatitis A can be prevented by the intramuscular administration of immune globulin during the incubation period, if given within 2 weeks of exposure. Administration of the hepatitis A vaccine will not protect the patient exposed to hepatitis A, as protection will take a few weeks to develop after the first dose of the vaccine. The hepatitis B vaccine provides protection against the hepatitis B virus, but plays no role in protection for the patient exposed to hepatitis A. Albumin confers no therapeutic benefit.

A patient with liver cancer is being discharged home with a hepatic artery catheter in place. The nurse should be aware that this catheter will facilitate which of the following? A. Continuous monitoring for portal hypertension B. Administration of immunosuppressive drugs during the first weeks after transplantation C. Real-time monitoring of vascular changes in the hepatic system D. Delivery of a continuous chemotherapeutic dose

D. Delivery of a continuous chemotherapeutic dose Rationale: In most cases, the hepatic artery catheter has been inserted surgically and has a prefilled infusion pump that delivers a continuous chemotherapeutic dose until completed. The hepatic artery catheter dose not monitor portal hypertension, deliver immunosuppressive drugs, or monitor vascular changes in the hepatic system

A nurse is participating in the emergency care of a patient who has just developed variceal bleeding. What intervention should the nurse anticipate? A. Infusion of intravenous heparin B. IV administration of albumin C. STAT administration of vitamin K by the intramuscular route D. IV administration of octreotide (Sandostatin)

D. IV administration of octreotide (Sandostation) Rationale: Octreotide (Sandostatin) a synthetic analog of the hormone somatostatin is effective in decreasing bleeding from esophageal varices, and lacks the vasoconstrictive effects of vasopressin. Because of this safety and efficacy profile, octreotide is considered the preferred treatment regimen for immediate control of variceal bleeding. Vitamin K and albumin are not administered and heparin would exacerbate, not alleviate, bleeding.

A nurse is caring for a patient with liver failure and is performing an assessment in the knowledge of the patient's increased risk of bleeding. The nurse recognizes that this risk is related to the patient's inability to synthesize prothrombin in the liver. What factor most likely contributes to this loss of function? A. Alterations in glucose metabolism B. Retention of bile salts C. Inadequate production of albumin by hepatocytes D. Inability of the liver to use vitamin K.

D. Inability of the liver to use vitamin K. Rationale: Decreased production of several clotting factors may be partially due to deficient absorption of vitamin K from the GI tract. This probably is caused by the inability of liver cells to use vitamin K to make prothrombin. This bleeding risk is unrelated to the roles of glucose, bile salts, or albumin.

A patient with esophageal varices is being cared for in the ICU. The varices have begun to bleed and the patient is at risk for hypovolemia. The patient has Ringer's Lactate at 150 cc/hr infusing. What else might the nurse expect to have ordered to maintain volume for this patient? A. Arterial line B. Diuretics C. Foley catheter D. Volume expanders

D. Volume expanders Rationale: Because patients with bleeding esophageal varices have intravascular volume depletion and are subject to electrolyte imbalance, IV fluids with electrolytes and volume expanders are provided to restore fluid volume and replace electrolytes. Diuretics would reduce vascular volume. An arterial line and Foley catheter are likely to be ordered, but neither actively maintains the patient's volume.

A nurse suspects that an older adult client may be experiencing hearing loss. which finding would support the nurse's suspicion? select all that apply. a- dropping of word endings b- disinterest in conversations c- social withdrawal d- domination of conversation e- quick decision making

a, b, c, d


संबंधित स्टडी सेट्स

Marketing Chp. 15, 16, 21, Marketing Ch 9 10 12, MKTG 3433 Exam 2 UARK, MGMT 3433 UARK Exam 2, Marketing 3433 Lezon Test 2, MKTG 3433 UARK Test 2, MKTG 3433 Chapters 9,10,12,13,14,15,16,17,18 EXAM 2 (SUMMER), MKT 291 Chapter 17, MKTG Exam 4: Chapter....

View Set

Fundamentals of Nursing Chapter 30 Review PP problem slides

View Set

Chapter 25 - Fluid, Electrolyte, and Acid-Base Homeostasis

View Set

Patho Immunity, Infection and Cancer

View Set

OST-249 - Medical Coding Prep - Mock Exams

View Set

PF VET113 / VET114 Anatomy & Physiology Terms

View Set

metabolism, nutrition, and energetics

View Set